You are on page 1of 167

MED2.

CP: SAMPLEX BASED RATIO CARDIO PULMO

SE 1
SHIFTING EXAM OCT 04 2021

2021 SHIFTING EXAM – 3A


USE AT YOUR OWN RISK!
QUESTION ANSWER RATIONALE
1. A 25y female, known case of RHD consulted her cardiologist due to shortness B
of breath and easy fatigability every time she would hurriedly climb 2 flights of
stairs of their house, which she used to do without difficulty. What is the
functional capacity of our patient?
A. Class I
B. Class II
C. Class III
D. Class IV

Harrison’s Principles of Internal Medicine, 20th edition


2. A 55-year-old male presented with progressive shortness of breath. He B Central in the pharmacologic treatment of COPD and in symptoms reduction
stopped smoking 10 years ago. You are considering COPD. On initial would be the bronchodilators. The role of pulmonary rehabilitation should also be
assessment, the patient has mMRC 3 and history of 2 exacerbations for the emphasized.
past year. Which of the following should also be considered?
A. Oral bronchodilator COPD Management Goals:
B. Pulmonary rehabilitation ● Diagnosis ● Reduce symptoms
C. Tetanus vaccination → Spirometry → Pharmacotherapy
D. Lung transplantation ● Reduce Risk → Bronchodilators
→ Smoking cessation → Pulmonary rehabilitation
→ Immunize (Influenza and ● Reduce complications
pneumococcal) → Pharmacotherapy
● Reduce other exposure

A-TG8, A-TG12
The patient is in Group D of the COPD Severity Group considering his history of 2
exacerbations for the past year and mMRC score of 3. Pharmacologic treatment
for Group D patients includes a combination of bronchodilators.

[MED.PULMO.1.03. CHRONIC OBSTRUCTIVE PULMONARY DISEASE] p.5 2023 TRANS

Pulmonary Rehabilitation
This refers to a comprehensive treatment program that incorporate exercise,
education, and psychosocial and nutritional counseling. In COPD, pulmonary
rehabilitation has been demonstrated to improve health-related quality of life,
dyspnea, and exercise capacity. It has also been shown to reduce rates of
hospitalization over a 6- to 12-month period.
Chapter 286, p. 1997 Harrison’s Principles of Internal Medicine, 20th edition

Lung Transplantation
COPD is the second leading indication for lung transplantation. Current
recommendations are that candidates for lung transplantation should have very
severe airflow limitation, severe disability despite maximal medical therapy, and
be free of significant comorbid conditions such as liver, renal, or cardiac disease.
Chapter 286, p. 1997 Harrison’s Principles of Internal Medicine, 20th edition
3. A 20-year-old previously well male, smoker, came in the emergency room with C According to Dra. Visperas, the patient is already saturating in the emergency
sudden onset of left-sided chest pain and dyspnea. Vital signs are as follows: room so we can do a simple aspiration.
BP 140/90, HR 95, RR 29, T 36.8, O2 sat 88% at room air. Pulmonary physical Shifting Exams Review/Dra. Visperas
examination is compatible with left-sided pneumothorax, confirmed by chest
radiograph. What is the initial recommended treatment for this patient?
A. Thoracostomy with stapling of blebs
B. Tube thoracostomy
C. Simple aspiration
D. Pleurodesis
4. A 42-year-old female consulted because of progressive shortness of breath. D A flow volume loop that is narrow points to a restrictive ventilatory defect whereas
The volume flow loop of her spirometry last appeared narrow. Which of the a short volume loop suggests an obstructive ventilatory defect.
following should be considered?
A. Asthma
B. Chronic bronchitis
C. COPD
D. Interstitial lung disease

A-TG8, A-TG12
Restrictive vs. Obstructive Ventilatory Defect
OBSTRUCTIVE RESTRICTIVE
Disease: Airways Disease: Thoracic pump/Lungs
Hindrance to passage of air Inability to expand alveoli
P: Pleura
A: Alveoli
I: Interstitial
N: Neuromuscular
T: Thoracic Cage
Low FEV1/FVC Low FVC
*Lung volume measured as TLC;
seen as FVC in spirometry
[MED2.PULMO.1.05. SPIROMETRY] 2023 TRANS
5. One of the following drugs may be used for pharmacologic cardioversion of Amiodarone is an anti-arrhythmic drug that can be used for pharmacologic
AF: B cardioversion. Digoxin, Metoprolol, and Verapamil are drugs used as an acute and
A. Digoxin long-term treatment for rate control.
B. Amiodarone
C. Metoprolol ● Acute Treatment: Rate Control
D. Verapamil → Control ventricular rate
■ Beta blockers (e.g. Metoprolol) – first-line
■ Calcium channel blocking agents: verapamil or diltiazem
- Amlodipine is not used because it is a dihydropyridine CCB; NO effect
in AV node
■ Digoxin – uncommonly used as a stand-alone agent, usually given in
combination with BBs or CCBs
→ Anticoagulation (if with indications)
■ IV heparin – if >48 h
● Acute Treatment: Rhythm Control
→ Pharmacologic Cardioversion – antiarrhythmics
■ Amiodarone – IV or oral (available in the PH)
■ Flecainide – IV or oral (available in the PH)
■ Propafenone – IV or oral
■ Dofetilide – oral
■ Ibutilide – IV
→ Electrical Direct Current Cardioversion
■ Treatment of choice in Acute AF with severe symptoms and
hemodynamic instability

A-TG8, A-TG12
- → Unstable hypotension and/or heart failure;
altered sensorium
- → DC Cardioversion (200 J biphasic synchronized shock)
○ ■ Establishes normal sinus rhythm in < 90% of patients
○ ■ Must sedate patient first (!!) - very painful intervention
● Long-term Management: Rate Control
■ Beta blocker
■ Calcium channel blocker: verapamil, diltiazem
■ Digoxin
■ Combination
● Long-term Management: Rhythm Control – antiarrhythmics
■ Amiodarone (Available in the PH)
■ Flecainide (Available in the PH)
■ Disopyramide
■ Dofetilide
■ Propanefone
■ Sotalol

[MED2.CP.1.08. DISORDERS OF RHYTHM] p.16 2023 TRANS


6. Which is the most frequent CXR findings in acute PE? D According to Dra. Visperas, although the first 3 choices are more or less the
A. Pleural-based density (Hampton’s hump) “classic” findings, meaning we could have these findings, the more common is the
B. Focal oligemia (Westermark’s sign) normal x-ray.
C. Enlarged right descending pulmonary artery (Palla’s sign) Shifting Exams Review/Dra. Visperas
D. Normal chest x-ray
7. Which of the following ECG findings suggests transmural ischemia? C Ischemia characteristic changes in the ECG:
A. Horizontal ST segment depression ● Repolarization abnormalities: T-wave inversions, displacement of ST
B. Down sloping ST segment depression segment (more severe)
C. ST segment elevation ● Transient T-wave inversion: Non-transmural, intramyocardial ischemia
D. T wave inversion ● Transient ST-segment depression: Patchy subendocardial ischemia
● ST-segment elevation: More severe transmural ischemia
[MED2.CP.1.06.1. ISCHEMIC HEART DISEASE] p.3 2023 TRANS
8. A 77-year-old female had cough productive of whitish sputum and fever with B According to Dra. Visperas, it is a high-risk CAP since the patient has had previous
highest temperature of 39 degrees Celsius. She self medicated with antibiotic treatment, multilobar, CAP gram (+) cocci in pairs, and comorbidities
Paracetamol and was given unrecalled antibiotics. At the emergency room, her (UTI and diabetes). We need to give a better or broader spectrum antibiotics and
oxygen saturation was 60% at room air, BP 120/80, CR 105 Temp. 38.6 degrees cover for Pseudomonas as well.
Celsius, RR 32/minute. She is conscious, coherent, GCS 15 in supine position. Shifting Exams Review/Dra. Visperas
Crackles were heard from T8 down bilateral, no wheezes. She is diabetic and
has had previous admission for UTI 3 months ago. Chest Xray showed
infiltrates multilobar on bilateral lung fields. Sputum GS yielded Gram + cocci
A-TG8, A-TG12
in pairs. SARS COV-2 RT PCR was negative. Which of the following treatment
regimen should be empirically given?
A. Azithromycin 500mg/IV OD + Levofloxacin 500 mg /IV OD
B. Piperacillin tazobactam 4.5 g/IV q 6h + Azithromycin 500 mg/IV OD
C. Cefuroxime 500 mg/tab BID + Levofloxacin 750 mg/tablet OD
D. Amikacin 500 mg/IV q 12hours + Azithromycin 500 mg/tab OD

[MED2.PULMO.1.06. COMMUNITY ACQUIRED PNEUMONIA] p.4 2023 TRANS


9. Best antihypertensive medication given in a patient with BP 180/100 and acute D “For acute dissection, unless contraindicated, β-adrenergic blockers should be
aortic dissection. administered parenterally, using intravenous propranolol, metoprolol, or the
A. Hydralazine IV short=acting esmolol to achieve a heart rate of ~60 beats/min. This should be
B. Nicardipine IV accompanied by sodium nitroprusside infusion to lower systolic blood pressure to
C. Phentolamine IV <120 mmHg. Labetalol, a drug with both β- and α-adrenergic blocking
D. Labetalol IV properties may be used as a parenteral agent in acute therapy for
dissection.”
Chapter 274, p. 1921 Harrison’s Principles of Internal Medicine, 20th edition
10. What is the respiratory disturbance index of a patient who had 10 apnea and B AHI (Apnea Hypopnea Index) is the averaged frequency of apnea and hypopnea
10 hypopnea during sleep study where he slept for 5 hours? events per hour of sleep
A. 2
B. 4 AHI = (Apneas + hypopneas)/ hours of sleep
C. 15 = (10 + 10)/ 5
D. 50 =4
[MED.PULMO.1.09.b. SLEEP DISORDERED BREATHING] p.6 2023 TRANS
11. A 77-year-old female had cough productive of whitish sputum and fever with D • According to Dra. Visperas, “We want to know if there’s hypoxemia because of
highest temperature of 39 degrees Celsius. She self-medicated with the O2 saturation (60%)”, therefore the answer is ABG.
Paracetamol and was given unrecalled antibiotics. At the emergency room, her Shifting Exams Review/Dra. Visperas
oxygen saturation was 60% at room air, BP 120/80 mmHg, CR 105 bpm,
Temperature 38.6 degrees Celsius, RR 32/minute. She is conscious, coherent,
GCS 15 in supine position. Crackles were heard from T8 down bilateral, no
wheezes. Which of the following should one request for at the emergency
room?
A. Chest Xray
B. Peak flow
C. CBC
D. ABG
12. Which finding most likely requires the addition of positive end expiratory C ● According to Dra. Visperas, if still not improving after 100% FiO2 with widened
pressure after challenging with 100% FiO2? P(A-a) O2, this is indicated for PEEP.
A. Decreased P(A-a)O2 and unimproved Pa O2 ● In most clinical settings, positive end-expiratory pressure (PEEP) is adjusted to
B. Decreased P(A-a) O2 and improved Pa O2 minimize FiO2 and provide adequate PaO2 without causing alveolar
C. Widened P(A-a) O2 and unimproved Pa O2 overdistention.
D. Widened P(A-a) O2 and improved Pa O2 ● Normal P(A-a) O2 ratio is 10-20 torr

A-TG8, A-TG12
→ There should be a difference or else gas will not transfer from alveolar to
arterial
● Widened P(A-a) O2 is >20 torr
→ Alveolar oxygen is not transferred to arteries due to hypoventilation or
another mechanism, which is identified by giving 100% oxygen
■ Does not improve: Shunt (collapsed alveoli in atelectasis, filled alveoli in
pneumonia)
■ Improves: V/Q mismatch (airway diseases such as asthma, COPD, ILD,
alveolar disease, pulmonary vascular disease)
[MED 2-PULMO] 1.10 – Arterial Blood Gas p.7
Harrison’s Principles of Internal Medicine, 20th edition
13. A 40-year-old female presents with bipedal edema and ascites. A PA lift and C ● According to Dra. Garcia, “The case is ASD hence echo is the best diagnostic
murmur was heard at the pulmonic valve area. The second sound was also modality”.
widely split. The best diagnostic procedure to request is: ● The classic PE finding in ASD is a wide, fixed splitting of S2, which is due to
A. Cardiac computerized tomography prolonged RV ejection and increased PA capacitance, which, in turn, delay
B. Myocardial perfusion scintigraphy pulmonary valve closure.
C. 2D Echocardiography with Doppler ● ASDs occur most commonly in the region of the fossa ovalis, referred to as
D. Coronary angiography secundum-type defects. Additional ASDs include defects of the sinus venosus
and atrium primum.
● Color flow Doppler echocardiography is usually sufficient for diagnosis of a
secundum-type ASD, but agitated saline is generally needed for the diagnosis
of other types of ASD.
Harrison’s Principles of Internal Medicine, 20th edition
14. In a patient with hypertensive emergency with acute cerebral infarction for C Criteria for Thrombolytic Therapy
thrombolysis, what is the target BP to minimize the risk for intracerebral Cerebral Infarct not candidate to Cerebral Infarct candidate to
hemorrhage to occur? thrombolytic therapy thrombolytic therapy
A. Reduce SBP to < 220 and DBP to < 130 mmHg
Institute anti-hypertensives Goal blood pressure
B. Reduce SBP to < 200 and DBP to < 120 mmHg
• SBP > 220 mmHg • < 185 mmHg SBP
C. Reduce SBP to < 185 and DBP to < 110 mmHg
D. Reduce SBP to < 160 and DBP to < 100 mmHg • DBP > 130 mmHg • < 110 mmHg DBP
MED.CARDIO.CCC3.CASE WITH RIGHT-SIDED WEAKNESS 2023 Trans
15. What is the best way to confirm the presence of pleural effusion if ultrasound C ● If there is no access to ultrasound, to be able to know if it’s pleural effusion, get
is not available? the CXR of a patient in a side lying (lateral decubitus) position relative to the
A. Lateral upright film affected lung
B. Apicolordotic view ● If the fluid flows to the dependent area, it is pleural effusion (Layering). If
C. Lateral decubitus view layering is present, it means:
D. Antero-posterior film → It is pleural effusion and not thickening of the pleura
→ No loculation or compartmentalization of the effusion and fluid is freely
moving in the pleural space
→ If you measure from the inside of the chest wall up to the upper level of the
fluid, if the measured distance is >1 cm, then there is enough fluid to indicate
at least a diagnostic thoracentesis
[MED 2-PULMO] 1.01 – Chest Imaging p.11
16. A 42-year-old male with lung cancer on room air, was referred for preop risk A Oxygenation Interpretation for < 60 y/o
assessment prior to lung resection surgery. ABG results are as follows: pH
PaO2 < 80 Hypoxemic
7.43, PaCO2 36, HCO3 23, PaO2 91, O2 sat 98%, AaDO2 14, a/ O2 0.87, PF 433.
PaO2 of 80-100 Normal oxygenation
What is the oxygenation status of the patient?
A. Normal oxygenation
Oxygenation Interpretation for > 60 y/o
B. Inadequate oxygenation
PaO2 < 80 – years above 60 Hypoxemic
C. More than adequate oxygenation
PaO2 > 80 – years above 60 Normal oxygenation
D. With hypoxemia
[MED 2-PULMO] 1.10 – Arterial Blood Gas p.5
17. A 79-year-old hypertensive diabetic male with previous history of atrial C
fibrillation presents with palpitations and shortness of breath. He had history
of stroke 2 years ago and heart failure 1 year prior. On examination, BP 150/100

A-TG8, A-TG12
mmHg, HR 97 bpm irregularly irregular, RR 15 cpm. There were no murmurs.
Neurologic exam is normal. Your advice would be:
A. His blood pressure is acceptable for his age and no antihypertensive drugs is
needed since he had a stroke 2 years ago
B. He needs additional workup with a cardiac MRI to evaluate cardiac structure
and function
C. He has significant risk for stroke and will need an anticoagulant, preferably a
non-vitamin K oral anticoagulant
D. He will definitely benefit from rhythm control through cardioversion since he’s
symptomatic despite a heart rate of 97 bpm

Stroke Prevention in AF (ESC 2016 Guidelines)


st
● 1 thing to ask: Is the patient with mechanical/prosthetic heart valves or
moderate/severe mitral stenosis?
● 2nd thing to do: Get the CHA2DS2 – VASc Score
→ Score of >/= 2 warrants anticoagulation with preference for NOAC over
warfarin
→ Score of 1 = Consider anticoagulation
→ Score of 0 = Antiplatelet or anticoagulant not needed
CHA2DS2 – VASc Score
Risk Factors Points
C – Congestive Heart Failure 1
H – Hypertension 1
A2 – Age >/= 75 years old 2
D – Diabetes Mellitus 1
S2 – Stroke/TIA 2
V – Vascular Disease 1
A – Age 65-74 years old 1
Sc – Sex category (Female) 1
• For this question, the patient has a CHA2DS2 – VASc score of 8. Therefore, the
risk of this patient in developing another stroke is HIGH. Anticoagulation should
be given.
MED.CARDIO.CCC3.CASE WITH RIGHT-SIDED WEAKNESS 2023 Trans
18. A 30-year-old female pregnant patient with Marfan’s syndrome is seen at the D • The diagnosis of aortic dissection can be established by noninvasive techniques
ER because of chest pain. Her BP was noted to be unstable and there are such as echocardiography, CT, and MRI. Aortography is used less commonly
bibasal crackles present. Your working diagnosis is aortic dissection. What because of the accuracy of these noninvasive techniques. Transthoracic
diagnostic test is best AVOIDED? echocardiography can be performed simply and rapidly and has an overall
A. Electrocardiogram sensitivity of 60–85% for aortic dissection.
B. Chest radiograph • The relative utility of transesophageal echocardiography, CT, and MRI depends
C. Transesophageal echocardiography on the availability and expertise in individual institutions as well as on the
D. CT aortography
A-TG8, A-TG12
hemodynamic stability of the patient, with CT and MRI obviously less suitable
for unstable patients (The patient in this question is unstable).
• From journal: Because CT contributes to much higher fetal radiation, it is
essential always to be considerate of other options when contemplating utilizing
CT on pregnant patients. Other imaging modalities, including MRI, plain
radiograph, ultrasound, and nuclear medicine studies, should be considered first
before performing CT
Harrison’s Principles of Internal Medicine, 20th edition
Yoon, I. & Slesinger, T. 2021. Radiation exposure in pregnancy. Retrieved from
https://www.ncbi.nlm.nih.gov/books/NBK551690/
19. A 45-year-old male patient is referred because of loss of consciousness. Vital C ● Remember “SCREAM”
signs of the patient showed BP 0, HR 0, RR 0. Chest compression is → Shock à CPR (2 minutes) à Rhythm check à Epinephrine (1 mg/IV every 3-
immediately initiated. Upon arrival of the ECG, tracing showed: 5 minutes) à AMiodarone (Antiarrhythmic medication)
● For VF and Pulseless VT, EARLY DEFIBRILLATION IS KEY.
→ 200 J for biphasic defibrillator
→ 360 J for monophasic defibrillator
● Always do chest compressions immediately after defibrillation.
● Lidocaine IV – alternative to amiodarone
[MED 2-CARDIO] 1.08 – Disorders of Rhythm p.19

What is the next appropriate thing to do?


A. Insert an intravenous line, give Epinephrine 1 mg/IV every 3-5 minutes
B. Intubate the patient and hook to mechanical ventilator
C. Immediately get a defibrillator and defibrillate
D. Insert an intravenous line, give Atropine 0.5 mg/IV every 3-5 minutes
20. Which is the correct method in taking the BP? D 7 Simple Tips to Get an Accurate BP Reading:
A. The cuff length should encircle at least 40% of the arm circumference ● Correct cuff size (too small adds 2-10 mmHg)
B. The width of the cuff should be at least 80% of the arm circumference → Length: 80% of the arm’s circumference
C. The manometer should be at the level of the heart → Width: 40% of the arm’s circumference
D. The patient seated comfortably with the arm and feet both supported → Small cuff – marked overestimation of true BP
→ Large cuff – underestimation of true BP
● Cuff on bare arm (over clothes adds 10-40 mmHg to the SBP)
● Support arm so that the center of the cuff is at heart level: brachial artery level
(unsupported adds 10 mmHg)
● Support back/feet (unsupported adds 5-10 mmHg)
● Legs should be flat and uncrossed (crossed legs add 2-8 mmHg)
● Empty bladder (full bladder adds 10-15 mmHg)
● Don’t talk (distracting with conversation adds 10-15 mmHg)
[MED 2-CARDIO] 1.10 – Systemic Arterial Hypertension p.4
21. A 52-year-old male suspected to have IHD came in for treadmill exercise test. D Contraindications of Stress Testing:
He has a history of exertional dyspnea and intermittent fever. Which of the ● Rest angina within 48 hrs
following is a contraindication to the said condition? ● Unstable rhythm
A. Attacks of angina a week before the test ● Severe aortic stenosis
B. Pulmonary artery pressure of 30 mmHg ● Acute myocarditis
C. Aortic valve area of 3.0 cm2 ● Uncontrolled HF
D. Mitral valve vegetations ● Severe pulmonary hypertension
● Active infective endocarditis
→ Lesions due to infective endocarditis occur along the lining of heart valves
called “vegetations”
[MED 2-CARDIO] 1.06.1 – Ischemic Heart Disease p.5
22. A 35-year-old male with a history of asthma presents to the emergency C According to Dra. Visperas, letter C is the answer because: “The patient has fever
department because of difficulty of breathing. He has had 2 days of fever to for 2 days, it may be viral, we just need to give medications to control the
38.7°C (101.7°F), cough, and coryza. On exam, he is tachypneic, saturating 98% exacerbations.”
on room air, has 5-word conversational dyspnea, accessory muscle use, Shifting Exams Review/Dra. Visperas

A-TG8, A-TG12
diffuse wheezing, and bibasilar crackles. What would be the best steps in
managing this patient?
A. Obtain CBC and Chest x-ray. Start treatment with ampicillin+sulbactam and
azithromycin, salbutamol/ipratropium. Repeat exams after treatment to assess
for improvement.
B. Obtain CBC and Chest x-ray. Start treatment with salbutamol/ipratropium and
steroids. Repeat exams after treatment to assess for improvement.
C. Give salbutamol/ipratropium and steroids. Repeat exams after treatment to
assess for improvement.
D. Start treatment with ampicillin+sulbactam and azithromycin,
salbutamol/ipratropium. Repeat exams after treatment to assess for
improvement.
23. An 80-year old man, balikbayan from the US consulted for interpretation of C “Segmental pressures measure the systolic arterial pressure in multiple segments
laboratory findings done in the US before his trip to Manila. He was not able to of the lower extremities. A blood pressure cuff typically is applied in four segments:
go back as scheduled to the US because of the pandemic. He got hold of the high thigh, low thigh, calf, and ankle. It may not be possible to apply two thigh cuffs
results of a procedure where sequential pneumatic cuffs were placed in both to shorter or obese patients. Each cuff above the ankle is inflated to a pressure at
his legs and readings taken. The diagnosis was PAD according to the official which a Doppler signal is lost in the dorsalis pedis. The ankle cuff is inflated to
result, but the patient is skeptical. How would you explain the result based on pressures that result in the loss of signal in the dorsalis pedis and posterior tibial
the placement of sequential pneumatic cuffs? artery, and these values are recorded separately. The systolic pressure in mm Hg
A. Enables assessment of blood flow along the legs using Doppler is compared to the higher value of the upper extremity systolic brachial pressures,
B. Pulse volume contour amplitude is measured and is absent in PAD and a ratio is generated. The ratio of ankle pressure, using the greater of the
C. Presence of pressure gradients between cuffs provides evidence of presence dorsalis pedis or posterior tibial, is the ankle-brachial index (ABI). A drop in
and location of hemodynamically significant stenosis. pressure of 20 mm Hg between any two levels is considered suspicious for
D. None of the above are true a hemodynamically significant lesion in that segment.”
Mittleider D. (2018). Noninvasive Arterial Testing: What and When to Use. Seminars in
interventional radiology, 35(5), 384–392. https://doi.org/10.1055/s-0038-1676328
24. You are about to start medical treatment on a patient with stable ischemic A Beta blockers are contraindicated for asthma as it may cause bronchospasm.
heart disease. Before initiating beta blocker therapy, the presence of which
finding would make you proceed with caution?
A. History of night cough and wheezing
B. PR interval 0.18 sec
C. Heart rate of 65 bpm
D. Family history of depressive disorder
25. In the management of atrial fibrillation, which of the following is the most B ● Based on the ABC Management of AF, the priority is to avoid stroke or
appropriate? anticoagulation if indicated.
A. Priority is to acutely convert to sinus rhythm through electrical cardioversion ● A is wrong because the priority should be rate control
B. Anticoagulants is recommended for patients with risk for stroke ● C is wrong because clinical trials show no difference in mortality or stroke rate
C. Rate control is associated with higher mortality and risk of stroke compared to between rate and rhythm control
rhythm control ● D is wrong because thromboembolic prophylaxis should be given priority in AF
D. Treatment of the cardiovascular comorbidity takes precedence over [MED 2-CARDIO] 2.09 – Disorders of Rhythm pp.15-16
thromboembolic prophylaxis

26. NTM (Non-Tuberculosis Mycobacteria) infection is: D ● D is correct because NTM is considered one of the major etiologies for
A. Diagnosed by sputum microscopy bronchiectasis through bacterial infection.
B. Eradicated following 4HRZE/2HR ● A is wrong because CT scan is the gold standard for detecting NTM
C. Linked in bronchiectasis because of poor mucociliary clearance ● B is wrong because NTM cannot be easily eradicated by anti-TB medications
D. Emerging as an important etiologic agent in Non-CF bronchiectasis ● C is wrong because NTM is linked in bronchiectasis through bacterial infection
[MED 2-PULMO] 1.14 – Bronchiectasis
27. Beta blockers reduce myocardial oxygen demand by: D Beta blockers reduce myocardial oxygen demand by reductions in heart rate,
A. Decreasing heart rate and arterial pressure and increasing myocardial arterial pressure, and myocardial contractility.
contractility [MED 2-CARDIO] 1.06.1 – Ischemic Heart Disease p.7
B. Decreasing heart rate, increasing arterial pressure and myocardial contractility
C. Decreasing heart rate and myocardial contractility and increasing arterial
pressure
D. Decreasing heart rate, arterial pressure, and myocardial contractility
A-TG8, A-TG12
28. A patient with ischemic heart disease experiences symptoms of angina. His A CCBs are usually given if beta blockers are contraindicated
blood pressure readings and heart rates reach as high as 160/100 mmHg and
102 bpm, respectively. He has been recommended to take beta blockers, but a ● A: Verapamil (CCB-Nondihydropyridines)
history of asthma attacks precludes their use. Which drug can be given next? ● B: Metoprolol (BB)
A. Verapamil ● C: Aspirin (Antiplatelet)
B. Metoprolol ● D: Isosorbide Mononitrate (Nitrates)
C. Aspirin [MED 2-CARDIO] 1.06.1 – Ischemic Heart Disease p.7
D. Isosorbide mononitrate
29. Which of the following is a risk factor for sudden cardiac death in hypertrophic A
cardiomyopathy?
A. Left ventricular interventricular septal thickness of 3.0 cm
B. ECG showing premature ventricular complexes occurring singly
C. Blood pressure of 190/90 mmHg at peak exercise
D. A second degree cousin who died of acute coronary syndrome at age 66

[MED 2-CARDIO] 1.03 – Cardiomyopathy p.8


30. A 34-year-old female with chronic cough, weight loss, and night sweats. His B Regimen 2: 2HRZE/10HR
sputum AFB was positive. She also presented with altered sensorium. You are ● EPTB of CNS, bones, joints whether new or retreatment, with final Xpert result
suspecting TB meningitis. The most appropriate management is? → MTB, RIF sensitive
A. 2HRZE/6HRE → MTB, RIF inderteminate
B. 2HRZE/10HR ● New EPTB of CNS with positive SM/TB or clinically diagnosed and:
C. 2HRZES/1HRZE/5HRE → Xpert not done
D. 2HR/4HR → Xpert result is MTB not detected
[MED 2-PULMO] 1.13 – Tuberculosis p.4
31. What is the expected finding in a patient with fully compensated respiratory B Respiratory acidosis – low pH, high PCO
acidosis?
A. pH greater than 7.4 In fully compensated respiratory acidosis:
B. PaCO2 of 48 pH: 7.35-7.40
C. HCO3 of 20 HCO3: >28 (increased)
D. PaO2 of 90 PaCO2: >45
Page 4 of MED.PULMO.1.10.Arterial Blood Gas
32. On scheduled follow-up, a COPD patient had a chest x-ray. What radiologic A Tubular or slender/narrow heart is expected in patients with hyperinflation or
finding may be expected? emphysema
A. Tubular heart Page 1 of MED.PULMO.WS1.CXR and ABG
B. Focal oligemia
C. Globular heart
D. Narrowed ICS
33. What is the expected P(A-a)O2 in a sixty year old patient? C Arterial-Alveolar Oxygen Difference: p(A-a) O2 ratio
A. Greater than 0.75
B. 15 Formula for more accurate estimation of A-a difference:
C. 24 = 15 + (# of decades above age 30 x 3)
D. 400 or greater = 15 + (3x3)= 24
Page 6 of MED.PULMO.1.10 Arterial Blood Gas
34. A 22-year-old female with asthma exacerbation comes in the emergency room D A silent chest at auscultation means that airways are severely constricted, not
at 1AM. Which of the following PE findings is life-threatening? allowing air to pass through. This is life-threatening. This is why an absence of

A-TG8, A-TG12
A. Talks in words wheezing on auscultation does not rule out asthma. Upon therapy, wheezing may
B. In tripod position be appreciated due to air already passing through the airways.
C. Presence of intercostal retractions Page 8 of 2022.MED2.PULMO.1.04.Asthma (COMBINED PREVIOUS TRANS + LEC)
D. Silent chest on auscultation
35. A patient had pleural effusion with multiple loculations seen on chest C Do not do thoracentesis when there are multiple loculations seen since you
ultrasound. What is the next best step? cannot drain all loculations. Refer to TCVS for more permanent drainage.
A. Aspirate the fluid from the most superficial locule Page 4 of MED.PULMO.1.03.PLEURAL DISEASES
B. Increase the dose of antibiotics
C. Refer to surgery for drainage
D. Submit fluid for Gram stain and bacterial C/S
36. An 80-year old man is seen in the ER for syncope and difficulty of breathing. B “The obstruction to LV outflow produces a systolic pressure gradient between the
He is a known case of AS diagnosed 2 years ago after which he was lost to LV and aorta. When severe obstruction is suddenly produced experimentally, the
follow up. His ECG revealed LVH with evidence of strain pattern. Why did this LV responds by dilation and reduction of stroke volume. However, in some
patient develop recent symptoms of heart failure despite diagnosis of AS years patients, the obstruction may be present at birth and/or increase gradually over
before? the course of many years, and LV contractile performance is maintained by the
A. AS progresses slowly over the course of many years, LV contractile presence of concentric LV hypertrophy. Initially, this serves as an adaptive
performance is maintained by the development of increased cardiac output mechanism because it reduces toward normal the systolic stress developed by the
B. LVH initially serves as an adaptive mechanism by reducing systolic stress myocardium, as predicted by the Laplace relation (S = Pr/h, where S = systolic
developed by the myocardium toward normal; irreversible myocardial fibrosis wall stress, P = pressure, r = radius, and h = wall thickness). A large transaortic
develops eventually and is followed by systolic dysfunction valve pressure gradient may exist for many years without a reduction in cardiac
C. Large transaortic valve pressure gradients may exist for many years without output (CO) or the development of LV dilation. Ultimately, however, excessive
cardiac remodeling hypertrophy becomes maladaptive, LV systolic function declines because of
D. Initially in AS, even if the aortic gradient is high normal, excessive hypertrophy afterload mismatch, abnormalities of diastolic function progress, and irreversible
becomes maladaptive, abnormalities of systolic function progresses, and myocardial fibrosis develops.”
irreversible myocardial fibrosis develops Pathophysiology of Aortic Stenosis on Page 1803 of Harrison’s Principles of Internal Medicine
(20th edition)

● Choice A is incorrect: LV contractile performance is maintained by the


presence of concentric LV hypertrophy NOT by the development of
increased cardiac output.
● Choice C is incorrect: A large transaortic valve pressure gradient may exist for
many years without a reduction in CO NOT without cardiac remodeling
● Choice D is incorrect: Abnormalities of diastolic function progress, NOT
systolic function.
37. What is the drug of choice for the prevention of recurrent acute rheumatic A Best antibiotic for secondary prophylaxis in rheumatic fever: Benzathine
fever? Penicillin G
A. Benzathine PCN G Page 12 of MED.CARDIO.CCC1.CASE ON DYSPNEA
B. Penicillin V
C. Erythromycin
D. Azithromycin
38. In a STEMI patient with severe chest pain in need of immediate reperfusion, D ● A BP >180/100 is a relative contraindication. The patient’s BP is even less than
which of the following would contraindicate the use of streptokinase? 180/100.
A. Systolic blood pressure of 160mmHg ● Familial history of cerebral hemorrhage is not considered. Instead, the patient’s
B. Familial history of cerebral hemorrhage history of cerebral hemorrhage AT ANY TIME IN THE PAST should be asked
C. Active menstrual bleeding à absolute contraindication
D. Back pain and widened mediastinum on chest X-ray ● Active menstrual bleeding is not a contraindication.
● Back pain and a widened mediastinum on CXR suggests a dissecting aortic
aneurysm à absolute contraindication
Table 5 on page 8 of MED.CARDIO.1.06.2.ACS AND PRINZMETAL ANGINA

39. An 88-year-old patient consults because of syncope. ECG showed the C 3rd Degree AV Block or Complete Heart Block
following tracing. ● Also known as complete atrioventricular block
● Results to atrioventricular dissociation
→ Regular QRS complexes that are slow with regular P waves in between
→ QRS complexes and P waves have no relation to each other
A-TG8, A-TG12
Page 5 of MED.CARDIO.1.05.1.ABNORMAL ECG

Which of the following is the most appropriate statement?


A. The patient has sinus bradycardia which is a benign rhythm and does not
warrant treatment
B. He has first degree AV block which require close observation but no intervention
at this point
C. The tracking shows complete heart block and the patient needs a permanent
pacemaker
D. The ECG is normal and the patient must be worked up for other causes of
syncope

40. A 73-year-old male farmer, diagnosed case of PAD after complaining of C Treatment of PAD
claudication and now undergoing medical treatment inquired if (he) can go ● “Patients with claudication should be encouraged to exercise regularly and at
back to his farm and resume previous activity. He was instructed to start a progressively more strenuous levels.” (Choice A is false)
regimen of exercise training towards that goal. What principle of PAD ● “Supervised exercise training programs for 30- to 45-min sessions, three to five
management will you base your medical advice regarding this issue? times per week for at least 12 weeks, prolong walking distance.” (Choice B
A. Patients with claudication should be encouraged to perform regular low intensity is false)
exercise only ● “The beneficial effect of supervised exercise training on walking performance
B. Studies have shown that supervised exercise training programs for 30- to 45- in patients with claudication often is similar to or greater than that realized
min sessions, 3-5x / week for at least 12 weeks was ineffective in prolonging after a revascularization procedure.” (Choice C is true)
walking distance ● “Structured home and community-based exercise programs are also effective.”
C. Beneficial effect of supervised exercise training on walking performance in (Choice D is false)
patients with claudication often is similar to or greater than that realized after a Page 1924 of Harrison’s Principles of Internal Medicine (20th edition)
revascularization procedure
D. Structured home and community-based exercise programs are not cost effective
41. A 77 year old female had cough productive of whitish sputum and fever with C Oxygen supplementation should be given ASAP to treat hypoxemia.
highest temperature of 39 degrees Celsius. She self medicated with
Paracetamol and was given unrecalled antibiotics. At the emergency room, her
oxygen saturation was 60% at room air. BP 120/80, CR 105, Temp 38.6 degrees
Celcius, PR 32/minute. She is conscious, coherent, GCS 15 in supine position.
Crackles were heard from T8 down bilateral, no wheezes, SARS COV2 RT PCR
was negative. Which of the following should be given ASAP?
A. Furosemide
B. D50 water
C. Oxygen supplementation
D. Intubation
42. You were asked to monitor a patient suspected of having ischemic heart Suggestive The ischemic ST-segment response generally is defined as flat or downsloping
disease during a treadmill exercise stress test. Results showed downsloping of a positive depression of the ST segment >0.1 mV below baseline (i.e., the PR segment) and
depression of the ST segment >0.1 mV below the baseline which lasted longer exercise lasting longer than 0.08 s
than 0.08 seconds. These findings are: stress test for Harrison’s Principles of Internal Medicine, 20th edition (p.1854)
Note: This item was not discussed/included in the answer key during the SE review myocardial
ischemia
43. A 35 year old male was admitted due to intermittent difficulty of breathing. HE C Massive Pulmonary Embolism
is being treated for Lung cancer and is on his 3rd cycle of chemotherapy. The ● Characterized by extensive thrombosis affecting at least half of the pulmonary
pretest probability for VTE is high and CT PA showed filling defect on the right vasculature
and left pulmonary artery and was already on enoxaparin 0.6 ml/SC BID. While ● Hallmarks
→ Dyspnea

A-TG8, A-TG12
admitted, patient developed sudden drop in BP at 70/40 and CR at 40. What is → Syncope
the most likely diagnosis? → Hypotension
A. Acute Decompensated Lung Disease → Cyanosis
B. Acute Pulmonary Embolism ● Pretest probability for VTE is high
C. Massive Pulmonary Embolism Harrison’s Principles of Internal Medicine, 20th edition (p.1911)
D. Pulmonary Infarct
44. Which arterial blood gas finding and oxygenation index is expected of a patient D ● Patient is hypoxemic, hence should check for hypoventilation
with abdominal paradox and pulmonary edema on chest radiograph? → Increased PaCO2 = hypoventilation
A. Normal PaCO2 and P(A-a)O2 → Widened P(A-a)O2 = hypoventilation plus another mechanism (for this
B. Normal PaCO2 and widened P(A-a)O2 patient, it might be due to the shunt caused by intraalveolar filling or
C. Increased PaCO2 and normal P(A-a)O2 presence of pulmonary edema)
D. Increased PaCO2 and widened P(A-a)O2 [MED 2-PULMO] 1.10 – Arterial Blood Gas p.7
45. A 42-year-old female Filipino, non-smoker consulted because of an incidental A Lung Adenocarcinoma
finding of a peripherally located mass. She was asymptomatic and her physical ● Most common:
examination was essentially normal. If the mass turns out to be malignant, → Lung CA
what is most likely cell type based on his clinical profile? → in non-smokers
A. Adenocarcinoma → histologic type in women and young adults
B. Carcinoid tumor ● Peripheral lesion
C. Mesothelioma ● Metastasize early
D. Squamous Cell Carcinoma
● B is wrong because Carcinoid tumor is neuroendocrine tumor and is associated
with smokers
● C is wrong because Mesothelioma is related to asbestos exposure
● D is wrong because SCC is common in smokers and has a central lesion
[MED 2-PULMO] 2.07 – Lung cancer p.6
46. Along with an elevated cardiac troponin, which diagnostic finding fulfills the B Acute Myocardial Infarction
criteria for an acute myocardial infarction? ● ECG findings:
A. Pain localized below the umbilicus → ST segment elevation of at least 1mm in contiguous leads except in VII and
B. QRS duration > 120 ms with dominant S wave in V1 and broad monophasic R VIII
wave in the lateral leads ■ VII and VIII = ST segment elevation should be at least 1.5mm (female) or
C. Development of pathologic U waves 2.0 mm (male)
D. Absence of hypokinetic segments → Left bundle branch block = QRS duration > 120 ms with dominant S wave in
V1 and broad monophasic R wave in the lateral leads

● A is wrong because symptom coming from the heart never goes into the
umbilicus and above the jaw
● C is wrong because it should be development of new Q waves
● D is wrong because if the patient is known to have a normal wall motion before,
then suddenly there’s a presence of hypokinetic segments, then the patient
might be suffering from AMI
MED 2 Cardio-Pulmo SE Review (Ratio by Dra. Clarissa Mendoza)
47. A 66 year old hypertensive diabetic male presents with sudden onset of D Atrial fibrillation
slurring of speech and left sided weakness. On examination, he was conscious ● HR 95 bpm irregularly irregular
and coherent. BP 150/100 mmHg, HR 95 bpm irregularly irregular, RR 16. JVP ● Variable amplitude of carotid impulse
3 cm at 30 degrees with absent a wave. Variable amplitude of carotid impulse ● Varying intensity of S1
and varying intensity of S1. However, there were no murmurs noted. He had a
shallow left nasolabial fold, with MMT 3/5 on left upper and lower extremities, Cardioembolic stroke
(+) Babinski reflex on left. The neurologic diagnosis of this patient is most ● Most significant cause in most of the world is nonrheumatic (often called
likely: nonvalvular) atrial fibrillation.
A. Intracerebral hemorrhage ● The presumed stroke mechanism is thrombus formation in the fibrillating atrium
B. Subarachnoid hemorrhage or atrial appendage, with subsequent embolization.
C. Atherothrombotic stroke Harrison’s Principles of Internal Medicine, 20th edition (p. 3082-3083)
D. Cardioembolic stroke

A-TG8, A-TG12
48. Which patient data is in favour of a diagnosis of obstructive sleep apnea? D STOP BANG
A. 35 year old female ● Snoring
B. Neck circumference of 15 inches ● Tiredness
C. STOP BANG score of 1 ● Obstruction
D. BP of 140/90 mmHg ● Pressure (High BP)
● BMI >30
● Age >50
● Neck Circumference
→ 17 in for Men
→ 16 in for Women
● Gender (Male>Female)
● Total = 8 points
→ If >2 points = higher risk for Sleep disordered breathing
[MED 2-PULMO] 1.09.B – Sleep Disordered Breathing p.6
49. HR an 80-year-old male was seen in the ER for back pain. His vital signs were B Treatment of Aortic Dissection
noted to be stable. His ECG showed sinus rhythm with non-specific ST T wave Medical therapy should be initiated as soon as the diagnosis is considered. The
changes. His lateral chest x ray showed a dilated descending aorta. A CT patient should be admitted to an intensive care unit for hemodynamic monitoring.
aortogram showed dissection of the thoracic aneurysm. The following is true Unless hypotension is present, therapy should be aimed at reducing cardiac
of the treatment of HR. contractility and systemic arterial pressure, and thus shear stress. For acute
A. For uncomplicated and stable distal dissections and intramural hematomas dissection, unless contraindicated, β-adrenergic blockers should be administered
(type B), surgery is still the preferred treatment parenterally, using intravenous propranolol, metoprolol, or the short-acting
B. Long-term therapy for patients with aortic dissection (with or without surgery) esmolol to achieve a heart rate of ~60 beats/min. This should be accompanied by
requires antihypertensives and agents that reduce cardiac contractility sodium nitroprusside infusion to lower systolic blood pressure to ≤120 mmHg.
C. The long-term prognosis for patients with treated dissections is generally dim Labetalol, a drug with both β- and α-adrenergic blocking properties, also may be
with careful follow-up; the 10-year survival rate is ~10% used as a parenteral agent in acute therapy for dissection.
D. The in-hospital mortality rate of medically treated patients with type B
dissection is ~50% The calcium channel antagonists verapamil and diltiazem may be used
intravenously if nitroprusside or β-adrenergic blockers cannot be employed. The
addition of a parenteral angiotensin-converting enzyme (ACE) inhibitor such as
enalaprilat to a β-adrenergic blocker also may be considered.
Harrison’s Principles of Internal Medicine, 20th edition (p. 1921)
50. A 60-year-old patient with bronchiectasis followed up in your clinic. On C Bronchiectasis Severity Index score of 3
assessment, her Bronchiectasis Severity Index (BSI) score is 3. Management ● Low/Mild risk of hospitalization and mortality
consists of: ● If mild, the recommended treatment is daily physiotherapy
A. Macrolide therapy for 6 months → Maneuvers that will help the patient relieve dyspnea
B. Inhaled corticosteroids for 6 months → Breaks the cycle of inflammation and infection
C. Airway clearance physiotherapy → To clear up the secretions, do postural positioning or do chest physiotherapy
D. Inhaled antibiotics against Pseudomonas → Other maneuvers
■ Exercise
■ PEP
■ PEP with oscillation
■ High-frequency chest wall oscillation

A and B are for management of moderate type of bronchiectasis


D is for the management of severe type of bronchiectasis
[MED 2-PULMO] 1.14 – Bronchiectasis p.7
51. After 1 week of anti-TB meds, a 28-year old female developed psychosis and A Adverse effects:
convulsion. Which of the following is likely culprit? ● Isoniazid: psychosis and convulsion
A. Isoniazid ● Rifampicin: oliguria, albuminuria due to renal disorder, thrombocytopenia,
B. Rifampicin anemia and shock
C. Pyrazinamide ● Pyrazinamide: arthralgia due to hyperuricemia
D. Ethambutol ● Ethambutol: impairment of visual acuity and color vision due to renal disorder
MED.PULMO 1.13. Tuberculosis 2023 trans

A-TG8, A-TG12
52. What is the ideal management of a sleep apnea patient with BMI of 32 and an
apnea-hypopnea index of 30/hour?
A. Lifestyle change alone C CPAP à Gold standard treatment for sleep apniea
B. Mandibular appliance ● Breathing of patients with CPAP will be better after use
C. Continuous positive airway pressure ● Pushes positive pressure that will kick the airway open and splint it,
D. Bariatric surgery preventing airways from collapsing
MED.PULMO 1.09 B Sleep disordered breathing
53. Which of the following cardiomyopathy phenotype has predominantly right C Congestive symptoms:
sided congestive features? ● Restrictive: right often dominates
A. Dilated cardiomyopathy ● Dilated: left before right, except right prominent in young adults
B. Ischemic cardiomyopathy ● Hypertrophic: left sided congestion at rest may develop late
C. Restrictive cardiomyopathy
D. Hypertrophic cardiomyopathy MED.CARDIO 03 Cardiomyopathy 2022 trans
54. Which of the following class of drugs relieves congestion in acute B Acute decompensated heart failure à most common sign is fluid retention
decompensated heart failure ● Oral beta blockers: decrease heart rate and decrease inotropy
A. Oral beta blockers ● IV loop diuretics: treats congestion, decrease preload
B. Intravenous loop diuretic ● Dobutamine infusion: inotropic agent
C. Dobutamine infusion ● Oral digoxin: patients with resistant symptoms on the usual drug algorithm
D. Oral digoxin MED.CARDIO 02 Heart failure 2022 trans
55. A 55 year old male was recently diagnosed with extensive stage small cell lung A Recommended therapy for small cell carcinoma
cancer. What is the recommended first line treatment assuming that he has a ● Limited stage
good performance status? → chemo + radiotherapy
A. Chemotherapy → clinical stage 1: surgery + chemo
B. Surgery ● Extensive stage
C. Radiotherapy → chemotherapy
D. Immunotherapy
MED Pulmo 07 Lung cancer 2022 trans
56. In the management of atrial fibrillation, which of the following is true? C Rate control vs Rhythm control strategies in AF à clinical trial shows NO
A. Rhythm control is superior in decreasing mortality compared to rate control DIFFERENCE in mortality or stroke rate in between patients assigned to rate
B. Rate control is superior in decreasing mortality compared to Rhythm control control vs rhythm control
C. Stroke prophylaxis with anticoagulation reduces morbidity and mortality
D. Stroke prophylaxis with antiplatelets is superior to anticoagulants Anticoagulants are preferred over antiplatelets as stroke prophylaxis

MED. CARDIO 08 Rhythm disorders 2022 trans


57. Which of the following has acute rheumatic fever in a previously healthy 18 D Rheumatic heart disease
year old female living in a squatters area in the Philippines? ● Patient lives in the squatter’s area: moderate / high risk population
● There should be an antecedent group A strep infection = request for ASO titer
A. ASO titer 180 IU/ml, T 37.8C, ESR 25mm, PR interval of 0.12sec with multiple ● Titer should at least be 200
joint pains involving the lumbosacral region
B. ASO titer 150 IU/ml, T 38C, CRP 2md/dL, PR interval of 0.16 sec with multiple Carditis: hallmark in RF is endocarditis, affects mitral valve usually.
erythematous rashes on both cheeks for the past 5 days ● Present with MR +/- AR, prolongation of PR interval
C. ASO titer 250 IU/ml, T 37.8, ESR 20mm, PR interval of 0.20 sec with swollen ● If pericarditis: pericardial friction rub
and erythematous rashes on both cheeks for the past 5 days
D. ASO titer 200 IU/ml, T 37C, CRP 2.5mg/dL, PR interval of 0.18 sec with Arthritis: affect big joints, not the fingers
emotional lability and purposeless movements ● Joints affected: ankle, knee, hip joint, elbow joint
● Asymmetrical, migrating
● If moderate to high-risk population: may not be polyarthralgia

Erythema marginatum: trunk and extremities, never on the face


● Macules that expand with central clearing

Subcutaneous nodules: painless and movable


● Over extensor surfaces (backbone/vertebra, occiput, bony prominences)

A-TG8, A-TG12
Chorea: among the major criteria, chorea is the ONLY ONE that may appear
ALONE
● Commonly occur in the absence of other manifestations
● Purposeless movements
● May appear as emotional lability

Among the choices:


● A & B: ASO titer is not elevated
● C: T should at least be 38, ESR should at least be 30, normal PR interval,
fingers and toes are not affected

Shifting exams review/Dr. Garcia


58. A 72 year old chronic heavy smoker consulted because of chronic cough with A Appearance of a central tumor, blocking the airways
hemoptysis and progressive dyspnea for the past 3 months. On PE, there were ● Cough
wheezes appreciated over the left lung field. If you are considering a malignant ● Dyspnea
process, his manifestations are likely due to: ● Hemoptysis
● Wheeze
A. Direct effect of primary tumor ● Post-obstructive pneumonitis
B. Tumor spread to the thorax ● Chest discomfort
C. Extrathoracic metastases
D. Release of cytokines by tumor cells
MED.PULMO 07 Lung cancer 2022 trans

A-TG8, A-TG12
59. DL was told to have a murmur at 3 years old. She consults because of easy ASD causing volume overload to the RV must always be corrected either surgically
fatigue and progressive abdominal enlargement. On PE, she has an RV heave, or transcatheter. Since A is surgical which the patient does not want, then B is the
a PA lift, an ejection murmur at the 2nd left intercostal space and a widely spit B answer.
S2. Offhand she tells you that she does not want any surgery. The best
management to offer is?
A. Patch closure of the ASD
B. Transcatheter closure of the ASD
C. Comprehensive medical management
D. Symptomatic medical management Shifting exams review/Dr. Garcia
60. A 45-year old male, a case of PTB, bacteriologically confirmed, new. Which of A New BCTB or retreatment
the following is most appropriate first follow-up sputum examination? ● Ff up 1: end of extensive phase/2nd month
A. End of intensive phase ● Ff up 2: end of 5th month
B. End of maintenance phase ● Ff up 3: end of treatment/6th month
C. End of treatment
D. End of week 2
MED.PULMO 1.13 tuberculosis 2023 trans
61. Which of the following PE findings would suggest the presence of ischemic C Apex Beat found at the 6th ICS LAAL would indicate Hypertrophy or enlargement
heart disease? of the heart which would increase the work rate of the heart eventually leading to
ischemia once compensatory mechanism fail.
A. Sharp Disk margins on fundoscopy
B. JVP at 3cm at 30deg Clues suggestive of IHD
C. Apex Beat at 6th ICS, LAAL ● Repolarization abnormalities (ST-T changes)
D. Closure of semilunar valves louder at the base ● Disturbances of rhythm (bradyarrhythmias, tachyarrhythmias)
● Left ventricular hypertrophy
● Interventricular conduction delay

MED.PULMO.1.06.1 ISCHEMIC HEART DISEASE 2023 Trans


62. You were attending to a 65 year old female patient with decompensated heart C “Sympathetic system stimulation is thought to lead to ventricular tachycardia”
failure presenting with easy fatigability, PND, orthopnea, and bipedal edema.
ON PE, BP 100/70, HR 118, RR 22, with cold clammy extremities, distended
neck veins, bibasal crackles, and grade 2 bipedal edema. Which sign and/or
symptom points to sympathetic hyperactivity in heart failure?

A. Paroxysmal nocturnal dyspnea Franciosi, S., Perry, F., Roston, T. M., Armstrong, K. R., Claydon, V. E., & Sanatani, S. (2017).
B. Bipedal Edema The role of the autonomic nervous system in arrhythmias and sudden cardiac death. Autonomic
neuroscience : basic & clinical, 205, 1–11. https://doi.org/10.1016/j.autneu.2017.03.005
C. HR of 118
D. Respiratory Rate of 22
63. A 65 year old male known hypertensive comes for follow-up, bringing the work- D Normal values of LDL-C is <130mg and Micral Test is <10mg/DL. Patient’s LDL-
ups you requested. Creatinine 0.90mg/dL, FBS 110 mg/dL, total cholesterol C and Micral Test are both increased therefore it is Dyslipidemia and
183.7 mg/dL, HD-C 34.3 mg/dL, LDL-C 133.4 mg/dL, Triglyceride 131.3 mg/dL, Microalbuminuria
Na 138.00 mmol/L, K 5.0 mmol/L, SGPT 26 U/L, Micral test 55.20 mg/L. What are
the comorbidities that you have identified?

A. Diabetes and Dylipedima


B. Dyslipedima and Azotemia
C. Diabetes and Hyperkalemia
D. Dyslipedima and Microalbuminuria
Jensen, J. E., Nielsen, S. H., Foged, L., Holmegaard, S. N., & Magid, E. (1996). The MICRAL test
for diabetic microalbuminuria: predictive values as a function of prevalence. Scandinavian journal
of clinical and laboratory investigation, 56(2), 117–122.
https://doi.org/10.3109/00365519609088597

Lee Y, Siddiqui WJ. Cholesterol Levels. [Updated 2021 Jul 26]. In: StatPearls [Internet]. Treasure
Island (FL): StatPearls Publishing; 2021 Jan

A-TG8, A-TG12
64. Which of the following radiographic features of a pulmonary mass favor a D In a Chest CT scan Features Suggestive of Malignant Lesions
malignant process? 1. Spiculated border
2. Presence of Corona radiata pattern
A. Central Calcification
B. Doubling Time of less than 30 days
C. Doubling time of more than 400 days MED.PULMO.2.07 .LUNG CANCER 2023 Trans
D. Spiculated Border
65. A 65 year old male patient came to consult at the clinic due to PND, orthopnea, C Answer: Spironolactone 45mg
and intermittent bipedal edema. On PE, BP 150/90, HR 59, RR 22. Distended
neck veins, AB at 7th LICS AAL, (+) heave and lifts, (+) grade 3/6 MR murmur, A. Ivabradine 5mg BID – HR 59bpm
grade 3/6 AR murmur, occasional bibasal crackles, grade 1 bipedal edema. B. Furosemide 40mg TID – Not a disease modifying drug
Current medications Valsartan 160mg BID, Carvedilol 25mg BID, ASA 100mg D. Sacubitril Valsartan 100mg BID – Patient is already on Valsartan
OD, and Rosuvastatin 20mg/tab OD. What will you give next?

A. Ivabradine 5mg BID


B. Furosemida 40mg TID
C. Spironolactione 25mg OD
D. Sacubitril Valsartan 100mg BID
MED 2 Cardio-Pulmo SE Review (Ratio by Dra. Julie Visperas)
66. A 68 year old female, married, consulted due to elevated BP and knee & ankle C
joint pains. She is diagnosed hypertensive since she was 40 years old,
maintained on losartan/HCTZ 50/12.5mg 1 tab qD. She has also been diagnosed
with gouty arthritis since last year and is taking etorixocib 30mg 1 tab qD. PE
showed BP 160/100, CR 65 bpm, RR 20cpm. What is the appropriate
management for our patient’s problems?

A. Increase Losartan/HCTZ to 100/25 1tab qD and Increase etoricoxib to 60mg 1


tab qD
B. Increase Losartan/HCTZ to 100/25 1tab qD and shift Paracetamol/Ibuprofen
325/200mg 1tab q6H
C. Shift Losartan/HCTZ to 100/25 1tab qD to Losartan/Amlodipine 100/10 1Tab
qd and shift etoricoxib to Paracetamol/Tramadol 325/200mg 2tabs initially
then 1tab prn for pain
D. Maintain Losartan/HCTZ to 50/ 12.5 1 tab qD, add metoprolol succinate
100mg 1 tab qD and increase etoricoxib to 60mg 1tab qD

MED.PULMO.1,10 SYSTEMIC ARTERIAL HYPERTENSION 2023 Trans

A-TG8, A-TG12
67. A 45-year old female consulted because of shortness of breath. Her spirometry B Reversibility (SPIROMETRY)
test showed the following: What is the most likely diagnosis? ● Increase in FEV1 > 12% and > 200 mL, 10-15 mins after an inhaled short-acting
B2-agonist (SABA: such as inhaled albuterol/salbutamol 200-400 ug) or
LLN Actual Actual %change equivalent
PRE POST
FEV/FVC 63 57 60 MED.PULMO.1.04 ASTHMA 2023 Trans
FVC 1.98 2.44 2.54 4%
FEV1 1.22 1.39 1.56 12%
Since there was no significance response to the bronchodilator as indicated by the
A. Asthma
12% in FEV1 and 4% change in FVC and since in the FEV/FVC is the Actual POST
B. COPD
C. PTB (60) is lower than the LLN (63), we can say that the cause of the Patient’s SOB is
D. Heart Failure Obstructive

MED.PULMO. 1.02a SPIROMETRY 2023 Trans

68. A 35 year old male was admitted due to intermittent difficulty of breathing. He B Suggested treatment algorithm for use of fibrinolytics to treat acute pulmonary
is being treated for Lung cancer and is on his 3rd cycle of chemotherapy. The embolism. Probability of Pulmonary Embolism is dictated by hemodynamic
pretest probability for VTE is high and CT PA showed filling defect on the right parameters. Patients with low risk PE (Submassive without RV strain) are given
and left pulmonary artery and was already on enoxaparin 0.6 ml/SC BID. While only heparin anticoagulation. If the patient has an RV strain, heparin
admitted, patient developed sudden drop in BP at 70/40 and CR at 40. Which anticoagulation is initiated and fibrinolytics are indicated if there are evidences of
treatment should be instituted? increased severity. Alteplase is the preferred fibrinolytic agent.

A. Continue LMWH
B. Institute Alteplase
C. Catheter Directed Thrombolysis MED.PULMO. 1.12c ER MASSIVE VTE 2023 Trans
D. Surgical Embolectomy
69. One of the following ECG findings is consisent with Sick Sinus Syndrome? B Sick Sinus Syndrome is a conduction system disease that is a cause for Sinus
Bradycadia
A. Frequent Sinus tachycardia
B. Frequent Long sinus pauses Sinus Bradycardia
C. Frequent Ventricular tachycardia ● Impulses come from the sinus node and travel down the normal conducting
D. Frequent Ventricular Fibrillation pathways
● Rate is < 60 beats per minute
● Regularly occurring PQRST

MED.CARDIO.1.05.1.ABNORMAL-ECG 2023 Trans


70. A 35 year old patient living with HIV came in due to dyspnea, fatigability, PND, A Laboratory Evaluation for Myocarditis
and orthopnea. He has no other co-morbid conditions and is compliant with ● 12 L ECG
his antiretroviral agent. On PE, patient was hypostenic, conversant, BP 120/80, ● Echocardiogram
HR100, RR 28 cpm, T 37.9 celsius. AB 6th LICS AAL, normal S1/S2, (+) S3, (+) ● Serum Cardiac Biomarkers (Troponins) - usually elevated
grade 3/6 murmur at the apex. Clear breath sounds and has grade II bipedal ● Cardiac MRI
edema. What diagnostic exam will you request? → Typical mid-wall location;
→ increased tissue edema and gadolinium enhancement
A. Cardiac MRI with gadolinium contrast to rule out myocarditis
B. CD4 count and viral serology to determine disease status
C. Serum BUN and creatinine ratio to determine deterioration of renal fxn MED.CARDIO.1.03 CARDIOMYOPATHY AND RELATED DISEASES 2023 Trans
D. Cardiac troponin levels to rule in cardiac cause of dyspnea
71. A 55 year old call teacher had hemoptysis of approximately 670 mL in 6 B In acute management of massive hemoptysis, the initial priorities include “ensuring
hours. BP 90/70, CR 121, altered sensorium. What is the next best step? adequate airway protection, ventilation, and preserving cardiovascular function”

A. Transfuse Blood
B. Protect the airway
C. Give tranexamic acid
D. Start aspirin MED.PULMO.1.12.B.ER2.HEMOPTYSIS 2023 Trans

A-TG8, A-TG12
72. A 48 year old male with COPD presented at the ER on tripod position with RR B According to Dr. Visperas, you can intubate, but since there is no altered
32, SpO2 83%, with use of accessory muscles of respiration. He is not sensorium yet, it’s better to do NIV.
improving with the usual oxygen supplementation. What is the next best step?
A. Intubate the patient Non-invasive Mechanical Ventilation
B. Start on NIV Patient Inclusion Criteria:
C. Continue to observe ● Patient Cooperation
D. Nebulize with ICS every 15 minutes x 3 doses → Essential component: excludes agitated, belligerent or comatose patient
● Dyspnea
→ Moderate to severe, but short of respiratory failure
● Tachypnea
→ > 24 breaths/min
● Increased work of breathing
→ Accessory muscle use, pursed-lips breathing
● Hypercapnic respiratory acidosis
→ pH range 7.10 – 7.35
● Hypoxemia
→ PaO2/FiO2 < 200 mmHg
→ Best for rapidly reversible cause of hypoxemia like CHF

Absolute Contraindications:
● Coma
● Cardiac arrest
● Respiratory arrest
● Any condition requiring immediate intubation
→ Altered sensorium
→ Failure to protect airway

MED.PULMO.1.12.A.ER1.ACUTE RESPIRATORY FAILURE 2023 Trans


MED 2 Cardio-Pulmo SE Review (Ratio by Dra. Julie Visperas)
73. True of preoperative assessment of an 85-year old male heavy smoker, A “Exclusion of coronary artery disease by coronary angiography is important in all
diabetic with severe aortic stenosis slated for valve replacement surgery patients older than 35 years who are being considered for valve surgery. Coronary
angiography should also be performed in patients younger than 35 years if they
A. Preoperative coronary angiography should be performed routinely when have LV systolic dysfunction, symptoms or signs suggestive of coronary artery
appropriate, as determined by age, symptoms, and coronary risk factors. disease, or 2 or more risk factors for premature coronary artery disease, excluding
B. Favorable response to maximal medical therapy is a prerequisite gender. Generally, the incidence of associated coronary artery disease has been
C. Calcium scoring of the aortic valve is routinely performed reported to be 50% in patients with aortic stenosis who are older than 50 years.
D. Dobutamine stress echocardiography is a less expensive and less invasive Coronary angiography need not be performed in young patients with no
modality to rule out concomitant CAD in this patient. atherosclerotic risk factors and in circumstances where the risk involved outweighs
the benefits.”

[Guideline] Vahanian A, Alfieri O, Andreotti F, et al. Guidelines on the management of valvular


heart disease (version 2012): The Joint Task force on the Management of Valvular Heart Disease
of the European Society of Cardiology (ESC) and the European Association for Cardio-Thoracic
Surgery (EACTS). Eur Heart J. 2012 Oct. 33(19):2451-96.
74. A 70 year old male, previous smoker came in the ER with sudden dyspnea. If D PE findings in Pneumothorax:
this patient has pneumothorax on the right, what is the expected finding on ● Inspection: Asymmetrical; Lagging on the affected side
pulmonary PE? ● Palpation: Decreased tactile fremitus
A. (+) Egophony, right ● Percussion: Hyperresonant
B. Lagging on the left ● Auscultation: Decreased breath sounds
C. Increased tactile fremiti on the right ● Shift of mediastinal structures away from the affected side
D. Shifting of mediastinal structures to the left On CXR: Contralateral deviation of mediastinal structures

MED.PULMO.1.01.CHEST IMAGING 2023 Trans

A-TG8, A-TG12
75. In the scoring system for stroke risk in nonvalvular atrial fibrillation, one of B CHADS-VASc score
the following carries the highest risk of stroke: ● Each risk factor corresponds to a certain score
A. Hypertension ● Sum of scores corresponds to a certain stroke rate percentage
B. Previous stroke
C. Heart failure
D. Diabetes

MED.CARDIO.1.08.DISORDERS OF RYTHM 2023 Trans


76. SB, A 75-year old male smoker with hypercholesterolemia was seen in the D Thrombosis, embolism, and vasculitis are all associated with peripheral artery
outpatient clinic for intermittent claudication. He has a history of coronary disease (PAD) which is the most common cause of intermittent claudication.
artery disease and is a heavy alcoholic beverage drinker. Name the LEAST
probable pathologic cause of our patient’s condition.
A. Thrombosis
B. Embolism
C. Vasculitis
D. Fibromuscular dysplasia
Cassar K. (2006). Intermittent claudication. BMJ (Clinical research ed.), 333(7576), 1002–1005.
https://doi.org/10.1136/bmj.39001.562813.DE
77. A patient on regular hemodialysis complained of severe chest heaviness with C Cardiac Troponin I and T:
diaphoresis at 5AM. He was immediately rushed to the ER and was diagnosed → Peak: 24 hrs
with acute coronary syndrome. Initial quantitative troponin I was taken at → Baseline (persists): 7-10 days
7:30AM was only slightly elevated. If this was acute myocardial infarction,
when would you expect his cardiac biomarker to be at its peak?
A. 12:00nn of the same day
B. 5:00 PM of the same day
C. Around 5:00 AM the following day
D. At least 48 hours after the initial result

MED.CARDIO.CCC2.CASE ON CHEST PAIN 2023 Trans

A-TG8, A-TG12
78. A 35 year old male was referred for hypertension after a BP 160/100 was taken D All choices are features of congenital anomalies of the aorta
during a routine company’s annual physical. The patient is a nonsmoker
nondiabetic with good functional capacity. During PE, it was noted that the BP
in the lower extremities were normal. The working impression is coarctation of
the aorta. This is true of congenital anomalies of the aorta?
A. Usually involve the aortic arch and its branches
B. Most congenital anomalies of the aorta are asymptomatic
C. Symptoms of dysphagia, stridor, and cough arise if an anomaly causes a ring
around or otherwise compresses the esophagus or trachea. Hanneman, K., Newman, B., & Chan, F. (2017). Congenital Variants and Anomalies of the Aortic
D. All of the above Arch. RadioGraphics, 37(1), 32–51. https://doi.org/10.1148/rg.2017160033
79. A 58 year old male, came in at the ER because of repeated episodes of minimal B According to Dr. Visperas, if there is previous TB and structural damage, you are
hemoptysis. He had previous PTB treatment 10 years ago. You are suspecting thinking of bronchiectasis. Patients with history of previous infection such as
structural lung damage secondary to the previous TB infection. On PE, what pneumonia or TB may develop bronchiectasis. Crackles are heard in
will be commonly expected? bronchiectasis.
A. Decreased breath sounds
B. Crackles
C. Friction rub
D. Increased breath sounds MED 2 Cardio-Pulmo SE Review (Ratio by Dra. Julie Visperas)
80. A 56 year old housewife consulted due to progressive dyspnea. She is A Heart Failure:
frequently awakened due to bolus of coughing and drowning sensation ● Decrease preload (diuretics, nitrates)
accompanied by palpitations. PE revealed BP 110/70 , PR 112 bpm irregularly ● Decrease afterload (vasodilators, ACE inhibitors, ARBs) -
irregular, RR 26 cpm. JVP 6cm at 45 degree with absent A-wave. Hyperdynamic ● Increase contractility (digoxin, inotropic agents)
precordium with strong pulsations noted over the left parasternum and mid left ● Patients who are acutely decompensated will eventually have chronic heart
parasternum, (+) diastolic thrill at the apex, (+) bibasal crackles. 2D Echo- failure; thus, we must:
Doppler revealed mitral stenosis with mitral valve area of 0.8cm2, Wilkin score → Attenuate the adverse effect of chronic neurohumoral stimulation (BBs, ACE
of 10 with moderate mitral regurgitation and moderate pulmonary HTN. How inhibitors, ARBs, mineralocorticoid receptor antagonists, ARNIs)
are you going to manage this patient?
Systemic embolism:
A. Start the patient on Furosemide 40 mg 1-tab qD, Spironolactone 25 mg 1 tab ● Threat is high if patient has AF due to stasis of blood in the LA
qD, Digoxin 0.25mg 1 tab qD, Warfarin 2.5mg 1 tab qD to achieve INR of 2-3 ● Goal: Use anticoagulants that decrease the viscosity of the blood and decrease
and prepare patient for Mitral valve replacement with mechanical valve its tendency to clot
B. Start the patient on Hydrochlorothiazide 25 mg 1 tab BID, Spironolactone 25 → For the patient: Warfarin
mg 1 tab BID, Metoprolol 50mg 1-tab qD, ASA + clopidogrel and prepare
patient for Mitral valve replacement with tissue valve Atrial fibrillation:
C. Start the patient on Diltiazem 30mg 1-tab TID, Spironolactone 25 mg 1 tab qD, ● Goal: Control the rate
Nebivolol 5mg 1 tab qD, Warfarin 2.5mg 1 tab qD to achieve INR of 2-3 and → Cannot aim for conversion to sinus rhythm because LA is very dilated -
prepare patient for Mitral valve preplacement with mechanical valve permanent AF
D. Start the patient on Rivaroxaban 15mg 1-tab qD, Spironolactone 25 mg 1 tab ● Digoxin
qD, Candesartan 5mg 1 tab qD, and prepare patient for Percutaneous trans ● Beta blockers (if no contraindications)
venous mitral valve balloon commissurotomy ● Non-dihydropyridine calcium antagonists

Pulmonary congestion:
● Diuretic therapy (loop diuretics)

Mitral Valve Replacement:


● Indications
→ Patients with MS and significant associated MR
■ Severe MS (orifice <1.5cm)
■ NYHA III-IV
→ Patients whose MV has been severely distorted by previous transcatheter or
operative manipulation
→ Surgeon does not find it possible to improve valve function significantly with
valvotomy
● Depends on the age and future plans regarding pregnancy

A-TG8, A-TG12
MED.CARDIO.CCC1.CASE ON DYSPNEA 2023 Trans
81. A 76 year old hypertensive diabetic male presents in the ER with sudden onset B Patient is already showing signs of affectation of the right cerebral hemisphere
left sided weakness of 1 hour duration. On examination, BP 150/100 mmHg, HR (MMT 3/5 on LU and LL extremities). Pathophysiology of atrial fibrillation (HR
127 bpm irregularly irregular, HR 99 bpm irregularly irregular, RR 15. irregularly irregular) can produce cardioembolic stroke. Thus, intracerebral
Auscultation revealed varying intensity of S1. There was no murmur. MMT 3/5 hemorrhage must be ruled out immediately with the use of CT scan to allow
on left upper and lower extremities. Which of the following is the most thrombolytic therapy.
appropriate statement?
A. Transthoracic 2D echocardiogram will help rule out left atrial thrombus and MED.CARDIO.CCC3.CASE-WITH-RIGHT-SIDED-WEAKNESS
negate the need for anticoagulation
B. Plain cranial CT scan will aid in ruling out intracerebral hemorrhage and allow
thrombolytic therapy
C. Electrocardiogram must be done immediately in order to assess the need for
immediate coronary intervention
D. Carotid ultrasound must be immediately done to rule out carotid stenosis

A-TG8, A-TG12
82. A 62 year old patient presented with pulmonary embolism and deep vein D According to Dra. Visperas, we can only give direct thrombin inhibitors to patients
thrombosis in a setting of heparin-induced thrombocytopenia, what is the with heparin-induced thrombocytopenia.
appropriate step after initiating treatment with parenteral anticoagulation?
A. Bridge to warfarin According to Harrison’s 20e (Chapter 111, page 825), “The direct thrombin
B. Bridge to NOAC inhibitor argatroban is effective in HITT”.
C. Switch to NOAC
D. Use direct thrombin inhibitors Harrison’s Principles of Internal Medicine, 20th edition
MED 2 Cardio-Pulmo SE Review (Ratio by Dra. Julie Visperas)
83. In which situation would fibrinolysis be favorable over percutaneous A ● Availability of a skilled PCI laboratory is definitely an indication for PCI.
coronary intervention as the mode of reperfusion? ● If the symptoms have been present for 12 hours, the clot cannot be lysed using
fibrinolytic agents. PCI is already indicated.
A. Early presentation ● Cardiogenic shock is a Class I indication for PCI.
B. Availability of a skilled PCI laboratory
C. Symptoms have been present for 12 hours MED 2 Cardio-Pulmo SE Review (Ratio by Dra. Marivic Garcia)
D. Cardiogenic shock
84. What ultrasound sign will identify pneumothorax? C ● Bar code signifies pneumothorax. During M-mode, there is no speckled
A. Absence of A lines appearance, meaning there is no lung sliding.
B. Comet tails ● A lines will tell us there is air in the lungs. Its absence may point to presence of
C. Bar code fluid or pulmonary edema.
D. Lung pulse ● Comet tails mean that the lung is expanded and that the visceral pleura is
rubbing against the parietal pleura.
● Lung pulse – “absent lung sliding with the perception of heart activity at the
pleural line”

Lichtenstein, D. A., Lascols, N., Prin, S., & Mezière, G. (2003). The "lung pulse": an early
ultrasound sign of complete atelectasis. Intensive care medicine, 29(12), 2187–2192.
https://doi.org/10.1007/s00134-003-1930-9
MED.PULMO.1.01.CHEST IMAGING

85. The presence of systemic arterial hypertension in a patient with AS as they A Hypertension causes hypertrophy and diastolic dysfunction that favors an
age can result to: increase in the left to right shunt.
A. Increase in the left to right shunt
B. Decrease in the left to right shunt MED 2 Cardio-Pulmo SE Review (Ratio by Dra. Clarissa Mendoza)
C. No effect in the left to right shunt
D. Unknown effect, still must be studied
86. A 28 year old accountant from Bicol, presents with episodic dyspnea, chest B
tightness, and cough. She has a history of atopy as a child. She was seen
several months ago for these symptoms at the ER and was given an inhaled
corticosteroid, which provided relief. However, since moving to Manila 4 weeks
ago, she has been using salbutamol over 4x a week at daytime when she has
symptoms. She claims to have night symptoms and has dyspnea on daily
activities. How would you assess her level of asthma control?
A. Acute exacerbation
B. Uncontrolled
C. Partly controlled
D. Controlled

Patient has night symptoms, uses salbutamol 4x a week, and has dyspnea on
daily activities. Having experienced 3 of the 4 symptoms, this signifies
uncontrolled asthma.

MED.PULMO.1.04.Asthma-COMBINED-PREVIOUS-TRANS-LEC

A-TG8, A-TG12
87. What is the most suitable support for an awake and cooperative patient with C Since the patient is still awake, we can give non-invasive ventilation.
type 2 respiratory failure with a pH of 7.2?
A. Face mask See ratio for #72.
B. High flow nasal cannula
C. Non-invasive ventilation
D. Intubation and ventilatory support
88. A 77 year old female had cough productive of whitish sputum and fever with D CURB 65
highest temperature of 39 deg Celsius. She self-medicated with Paracetamol ● Confusion – which the patient does not have (still conscious and coherent)
and was given unrecalled antibiotics. At the emergency room, her oxygen ● BUN >19mg/dL – patient has 35 mg/dL
saturation was 60% at room air, BP 90/50, CR 105, Temp 38.6 def C, RR 32/min. ● RR >30/min – patient has RR 32
She is conscious, coherent, GCS 15 in supine position in respiratory distress. ● BP <90 SBP, <60 DBP – patient’s BP is 90/50
Crackles were heard from T8 down bilateral, no wheezes. If CURB 65 score will ● Age >65 – patient is 77 y/o
be done given a BUN of 35 mg/dL, what will be the disposition for the patient?

A. Send the patient home


B. Observe at the emergency room for a few hours
C. Admit at ward
D. Admit at the ICU

Since the patient’s score is 4, she must be admitted to the ICU.


MED.PULMO.1.06.Community Acquired Pneumonia
89. A 32 year old smoker consulted because of episodic dyspnea for the past A Spirometry features of asthma show significant response to bronchodilators
year. He has a history of childhood atopy. Which of the following spirometric (reversibility)
features confirms asthma? ● Look at FEV1 or FVC.
● >/= 12% increase from pre-to post-bronchodilator
A. Change from pre- to post-bronchodilator FEV1 is 220mL, percent change is ● >/= 200mL increase from pre- to post-bronchodilator
14%
B. Pre-bronchodilator FEV1/FVC ratio is below the lower limit of normality
C. Post-bronchodilator FEV1 is 60% of predicted value MED.PULMO.1.05.SPIROMETRY
D. Pre-bronchodilator FVC is below the limit of normality
90. Which of the following PE findings serves as a clue for the presence of C All are normal PE findings except for cotton wool spots (hypertensive
atherosclerosis? retinopathy).
A. BP >90/60 mmHg but <120/80 mmHg
B. Apex beat 5th ICS, left MCL
C. Cotton wool spots on fundoscopy
D. +2 peripheral pulses
91. A 36 year old male had fever and cough for 1 month and had pleural effusion C Tuberculous Pleuritis
by CXR. What pleural fluid exam best correlates with the clinical ● Most common cause of exudative pleural effusion in many parts of the world:
manifestations? Tuberculosis
A. (+) Gram stain ● Clinical manifestations: Fever, weight loss, dyspnea, and/or pleuritic chest
B. pH 7.2 pain
C. predominance of lymphocytes ● Pleural fluid: Exudate with predominantly small lymphocytes
D. amber colored, slightly turbid
[MED 2-PULMO] 1.02. Pleural Diseases p.2
92. As a primary care physician what initial investigations will you request for A Primary Care Diagnosis
patients whom you suspect to have bronchiectasis? ● Investigations
A. Chest x-ray, spirometry, sputum culture → Chest x-ray
B. Chest x-ray, HRCT, sputum culture ■ May be normal
C. ABG, aspergillus antigen testing, chest x-ray ■ May be used to exclude other differential diagnoses
D. HRCT, spirometry, ABG ■ Limited sensitivity in patients with mild disease
A-TG8, A-TG12
→ Diagnostic spirometry
■ May be obstructive, normal or rarely restrictive
■ Look for other existing conditions such as COPD
→ Sputum culture
■ To reach lab on day of production
■ To identify the presence of persistent pathogens

According to Dra. Visperas:


● HRCT – should specifically be chest CT scan
● ABG – may be done, but not priority

[MED 2-PULMO] 1.14 – Bronchiectasis p. 4


MED 2 Cardio-Pulmo SE Review (Ratio by Dra. Julie Visperas)
93. What is the best treatment approach for a patient who is hypotensive with A Absence of lung sliding, hypotensive: TENSION PNEUMOTHORAX
absence of lung sliding and no A lines on lung ultrasound? A lines: Air inside the lungs, rules out pneumothorax
A. Needle decompression Tx: Needle decompression
B. Intravenous fluid infusion
C. Vasopressors
D. Thoracentesis MED.PULMO.1.01.CHEST IMAGING 2023 Trans
94. A 75 year old male was seen because of fever and cough. The patient’s D A. A. Aortic stenosis, one of the most common forms of valvular heart
daughter also mentioned his father was diagnosed with moderate aortic disease, most often occurs because of gradual progression of valvular
stenosis 2 years ago but was lost to follow up. A CXR was requested to confirm calcification in both normal and congenitally abnormal valves thus,
the diagnosis of pneumonia and also to look for radiologic signs of aortic absence of Absence of valvular calcification in an adult suggests that
stenosis. The following is/are expected radiologic finding(s) in this patient with severe valvular AS is absent.
AS Harrison’s Principles of Internal Medicine, 20th edition

A. Absence of valvular calcification in an adult suggests that severe valvular AS is B. LV dilatation, pulmonary congestion left atrial, pulmonary arterial, and
absent right-sided heart chamber enlargement are seen in the later stages of
B. LV dilatation, pulmonary congestion left atrial, pulmonary arterial, and right- the disease
sided heart chamber enlargement are seen in the later stages of the disease MED.CARDIO.CCC2.CASE ON CHEST PAIN 2023 Trans
C. A dilated proximal ascending aorta may be seen along the upper right heart
border in the frontal view as an indication of post stenotic dilatation C. A dilated proximal ascending aorta may be seen along the upper right
D. All of the above are correct heart border in the frontal view as an indication of post stenotic dilatation

95. A 21 year old male nurse with asthma presents to your primary care clinic as D Assessment of Asthma in Adults
a new patient. On review of his symptoms, he appears to be doing well using ● Assess Asthma Control
low-dose ICS-formoterol as needed (track 1 step 1). He has not been to the → From the case give, the patient is “doing well”, so choices A and C can be
ER or hospitalized, nor has he required oral corticosteroid in the past year. eliminated.
His prior records are not available. Which of the following is recommended as ● Assess treatment issues
the best next step in the management of this patient? → Check of the patient has a written asthma action plan
A. Initiate montelukast ● Assess comorbidities
B. Obtain allergy testing to identify triggers
C. Change to Track 2 step-1 MED.PULMO.1.04 ASTHMA 2023 Trans
D. Provide an individualized written asthma action plan
96. What is the most likely cause of hypoventilation in a patient with massive C Causes of Hypoventilation
pleural effusion? ● Low Tidal Volume
A. Decreases dead space → Anything that restricts lung expansion (PAINT) – P for pleura (pleural
B. Decreased respiratory rate effusion, pneumothorax)
C. Decreased tidal volume
D. Increased alveolar ventilation [MED 2-PULMO] 1.10. Arterial Blood Gas p. 2

A-TG8, A-TG12
97. A 77 year old female had cough productive whitish sputum and fever with A Signs of Respiratory Failure:
highest temperature of 39 deg Celsius. She self-medicated with Paracetamol ● Central cyanosis
and was given unrecalled antibiotics. At the ER, he oxygen saturation was 60% ● Abdominal paradox
at room air, BP 120/80, CR 105, Temp 38.6deg Celsius, RR 32/minute. She is ● Altered sensorium
conscious, coherent, GCS 15 in supine position. Crackles were heard from T8
down bilateral, no wheezes. If patient will develop decrease in sensorium and Invasive Mechanical Ventilation
abdominal paradox, which of the following should be the next step? ● Required in conditions that require intubation (ex. Altered sensorium, failure to
protect the airway)
A. Intubate the patient ● Procedure wherein an endotracheal tube is inserted with the usage of
B. Refer to Neurology service laryngoscope
C. Hook to Noninvasive mechanical ventilator
D. Request for blood culture MED.PULMO.1.12.A.ER1.ACUTE RESPIRATORY FAILURE 2023 Trans

98. A 38 year old male has been under DOTS treatment for TB for 3 months already. A
Because of COVID lockdown, he has to go to the province and missed the
continuation of his treatment. He returns to your clinic after 6 months. The
Xpert results is positive for MTB, RIF sensitive. I will be treating him as:

A. Category I
B. Category Ia
C. Category II
D. Regimen for Drug Resistant

Category Ia is seen in the old treatment management.


The organism is not drug resistant since it is still RIF sensitive.

A-TG8, A-TG12
[MED 2-PULMO] 1.13 – Tuberculosis p. 4-5

99. If you are managing a 55 year old female COPD patient and he is still short of B
breath with exacerbations. She is currently on LAMA + LAMA with eosinophil
count of 350 cells/uL. Which of the following is an option?

A. Shift to LAMA monotherapy


B. Shift to LABA + ICS
C. Shift to LABA monotherapy
D. Shift to PDE4 inhibitor plus azithromycin

Addition of ICS is considered due to high eosinophil count.

MED.PULMO 06 Chronic Obstructive Pulmonary Disease 2022 trans


100. Based on the 2020 Philippine HTN Clinical Practice Guidelines, an C
individual is labeled hypertensive when on home BP monitoring is:
A. SBP is at least 120mmHg and or the DBP is at least 70mmHg
B. SBP is at least 130mmHg and or the DBP is at least 80mmHg
C. SBP is at least 135mmHg and or the DBP is at least 85mHg
D. SBP is at least 140mmHg and or the DBP is at least 90mmHg

MED 2 Cardio-Pulmo SE Review (Ratio by Dra. Marivic Garcia

A-TG8, A-TG12
MED2.CP: SAMPLEX BASED RATIO

SHIFTING EXAM
CARDIO PULMO

SE 2
ALL CARDIO PULMO TOPICS 13 DEC 2021

2021 SHIFTING EXAM - 3B


USE AT YOUR OWN RISK! (Choices in some numbers were not discussed)
QUESTION ANSWER RATIONALE
1. Which non-invasive procedure is best used to document the PHYSIOLOGIC According to Dr. Garcia “B, C, and D would all check for the ANATOMIC
EFFECT of a hemodynamically significant obstruction without vessel EFFECT while transcutaneous oximetry would check for the physiologic effect
collateralization in an 85 year old male patient who refuses bypass surgery for of obstruction and collateralization”
significant PAD? According to Harrisons:
A. Transcutaneous Oximetry B: Vessel angiography: revascularization procedures, including catheter-based
B. Vessel Angiography and surgical interventions are usually indicated for patients with disabling,
C. Arterial Duplex Scan progressive, or severe symptoms of intermitten claudication despite medical
D. Segmental Pulse Volume therapy in order to improve claudication despite medical therapy. Conventional
angiography should be performed to assess CARDIOVASCULAR ANATOMY
in patients who are being considered for revascularization.

C: Arterial Duplex Scan: Shows the arteries and measures flow velocity as a
means of assessing the severity of obstruction or stenosis (anatomic)
A
D: Segmental Pulse volume recording is a noninvasive test wherein there is
placement of pneumatic cuffs which enables assessment of systolic pressure
along the legs. The presence of pressure gradients between sequential cuffs
provides evidence of the presence and location of hemodynamically
significant stenoses (anatomic)

Transcutaneous oximetry is a non-invasive test wherein the amount of oxygen


that reaches the skin through blood circulation is measured. It is used to assess
the severity of artery disease in the legs or, much less commonly, in the arms or
hands.
Dr Maria Victoria D. Garcia “Review of Cardio-Pulmo Shifting Exam” (1/22/22)
Harrison’s Principles of Internal Medicine, 20th Ed. (2018), p. 1923
2. A 21 year-old male presented at the OPD with the 4 cardinal symptoms of 4 cardinal symptoms of TB: cough, unexplained fever, unexplained weight
PTB. What will be the best diagnostic tool? loss, and nights sweats.
A. CXR
B. Sputum GeneXpert If TB is suspected, start first with the cardinal signs and symptoms.
C. Chest CT • If the patient presents with cough, unexplained fever, unexplained
D. IGRA weight loss, and nights sweats for ≥ 2 weeks, then the patient is tagged
as a case of presumptive TB.
B • Plan of action is to request for microbiological confirmation via GENE
X-PERT (if readily available).

GeneXpert
• Primary microbiological confirmatory testing tool especially in NCR
wherein the incidence of drug-resistant TB cases is quite high.
MED.PULMO.1.13.Tuberculosis
BMED 2023 TC: LICLICAN, C
3. A 65 year-old COPD case has more symptoms and at high risk of The case describes Group D patients who are those with most/more symptoms
exacerbation, which of the following is accurate? and at high risk for exacerbations. These patients experience more than or equal
A. mMRC is 1 to 2 exacerbations per year or more than or equal to 1 hospitalization per year.
B. CAT score is 4 This is defined as a frequent exacerbator under the COPD phenotypes.
C. Frequent exacerbator
D. Should be on SABA alone

A is not the answer since Group D patients have mMRC scores of more than or
equal to 2
B is not the answer since Group D patients have CAT scores of more than or
equal to 10
D is not the answer since Group D patients require drugs such as LAMA, LABA,
and ICS
MED 2-PULMO 1.03 – COPD pg.2, pg.6

BMED 2023 TC: LICLICAN, C


4. In which of the following circumstances should revascularization be Patients with three-vessel disease (or two-vessel disease that includes the
considered in a patient with ischemic heart disease? C proximal left descending coronary artery) and impaired global LV function (LV
A. HbA1c 5.7% ejection fraction <50%) or diabetes mellitus and those with left main CAD or
B. Ejection fraction 50% other lesions unsuitable for catheter-based procedures should be considered for
C. 80% stenosis of the left main coronary artery CABG as the initial method of revascularization.
D. Symptoms resolving with medical therapy Harrison’s Principles of Internal Medicine, 20th Ed. (2018), p. 1864
Revascularization should be considered in the presence of unstable phases of
the disease, intractable symptoms, severe ischemia or high-risk coronary
anatomy, diabetes, and impaired LV function.
Harrison’s Principles of Internal Medicine, 20th Ed. (2018), p. 1862-1863

A FPG ≥7.0 mmol/L (126 mg/dL), a glucose ≥11.1 mmol/L (200 mg/dL) 2 h after
an oral glucose challenge, or an HbA1c ≥6.5% meets the criteria for the
diagnosis of DM
Harrison’s Principles of Internal Medicine, 20th Ed. (2018), p. 2852
5. A 35-year-old male was seen for hypertension. On PE, his BP was 160/100. Peripheral Artery disease
The patient is a nonsmoker, nondiabetic, with good functional capacity. During D - This is defined as a clinical disorder which there is a stenosis or
PE it was noted that the lower extremities were significantly less muscular than occlusion in the aorta or the arteries of the limbs.
the upper extremities. Upon further examination it was noted that the BP in the - There is an increased risk of developing in cigarette smokers and in
lower extremities were 120/80. What is your diagnosis? persons with DM, hypercholesterolemia, hypertension, and renal
A. Peripheral artery disease insufficiency.
B. Subclavian artery steal phenomenon - It was stated in the case that the patient is a nonsmoker and
C. Aortic dissection nondiabetic, in which both factors contradict the risk factors stated
D. Aortic coarctation above.
- The most common symptom for patients suffering with PAD is
intermittent claudication. In the case, no claudication was noted which
makes PAD less likely to be our answer.
o Claudication
§ Pain, ache, cramp, numbness, or a sense of
fatigue in the muscles; it occurs during exercise
and is relieved by rest.
Harrison’s Principles of Internal Medicine, 20th Ed. (2018), p. 1923
Subclavian artery steal phenomenon
- If the subclavian artery is occluded proximal to the origin of the
vertebral artery, there is a reversal in the direction of blood flow in the
ipsilateral vertebral artery. Exercise of the ipsilateral arm may increase
demand on vertebral flow, producing posterior circulation TIAs, or
subclavian steal
- With the phenomenon’s definition, this choice can be immediately
crossed out since a transient ischemic attack shall be documented
upon PE.
Harrison’s Principles of Internal Medicine, 20th Ed. (2018), p. 3075
Aortic dissection
- This occurrence involves a tear in the aortic intima, resulting in
separation of the media and creation of a separate “false” lumen.
- The subtypes of acute aortic syndrome discussed in Harrison’s
typically present with chest discomfort that is often severe, sudden in
onset, and sometimes described as “tearing” in quality.
- There was no pain documented in this case which makes aortic
dissection be eliminated from our differential diagnoses.
Harrison’s Principles of Internal Medicine, 20th Ed. (2018), p. 76

BMED 2023 TC: LICLICAN, C


Aortic Coarctation
- On PE, the lower extremity blood pressure and pulses are lower
than (and delayed in timing, in contrast to) the upper extremity
values, unless significant aortic collaterals have developed.
- Among the choices, aortic coarctation is the diagnosis that fits our
case the most. Therefore, this is the correct answer
Harrison’s Principles of Internal Medicine, 20th Ed. (2018), p. 1837
6. A 50 y/o obese male, with DM and Hypertension (controlled) presents to the Treatment of Pulmonary Embolism
emergency room with sudden dyspnea and chest pain. There are no leg signs. A Anticoagulation
ECG showed sinus tachycardia. ABG revealed respiratory alkalosis. Chest ray - Effective anticoagulation is the foundation for successful
was essentially normal. A diagnosis of pulmonary embolism is strongly treatment of DVT and PE.
entertained. Which among the following is the best choice regarding initial - There are 3 major strategies
treatment of PE? o The classical but wanning strategy of parenteral
A. Unfractionated heparin with initial bolus of 80 U/kg anticoagulation with unfractionated heparin (UFH), low-
B. Warfarin titrated-empirically molecular -weight heparin (LMWH), or fondaparinux
C. Oral anti-coagulation monotherapy with apixaban “bridged to warfarin
D. Fondaparinux monotherapy o Parenteral therapy switched after 5 days to a novel oral
anticoagulant such as dabigatran (a direct thrombin
inhibitor) or edoxaban (anti-Xa agent)
o Oral anticoagulation monotherapy with rivaroxaban or
apixaban (both are anti Xa agents) with a 3 week or 1 week
loading dose, respectively, followed by a maintenance dose
without parenteral anticoagulation

Unfractionated Heparin
- UFH coagulates by binding to and accelerating the activity of anti-
thrombin, thus preventing additional thrombus formation
- UFH is dosed to achieve a target activated partial thromboplastin
time (aPTT) of 60-80 s.
- The most popular nomogram uses an initial bolus of 80 U/kg,
followed by an initial infusion rate of 18 U/kg per Hour in patients with
normal liver function
- The major advantage of UFH is its short half-life, which is especially
useful in patients in whom hour to hour control of the intensity of
anticoagulation is desired
Harrison’s Principles of Internal Medicine, 20th Ed. (2018), p. 1914
7. Which of the following anticoagulants used in the management of acute Non-ST Segment Elevation Acute Coronary Syndrome:
coronary syndrome is an inhibitor of Factor Xa? C Anticoagulants
A. Unfractionated heparin - Part of the options available for anticoagulant therapy to be added to
B. Low molecular weight heparin antiplatelet agents include the indirect factor Xa inhibitor, fondaparinux
C. Fondaparinux (C), which is equivalent in efficacy to enoxaparin but has a lower risk
D. Bivalirudin of major bleeding.
o Excessive bleeding is the most important adverse effect of all
antithrombotic agents, including both antiplatelet agents and
anticoagulants.
o Relative efficacy and safety compared with unfractionated
heparin is less certain.
- Unfractionated heparin (UFH) is long the mainstay of therapy and is
given when PCI is planned to be performed; however, aPTT should be
monitored during maintenance therapy and target goal is about 1.5 to
twice the control value. IV administration of UFH, along with non-
specific fibrinolytics, has added mortality benefits. When UFH is given

BMED 2023 TC: LICLICAN, C


with fibrin-specific agents like tPA and rTPA, it can be used as an
adjunct to thrombolysis.
- Low-molecular weight heparin (LMWH), enoxaparin (B), is
superior to UFH in reducing recurrent cardiac events, especially in
patients managed by a conservative strategy. However, it is
accompanied by a slight increase in bleeding compared to UFH.
Can do away with monitoring aPTT and is given SC.
- Bivalirudin, a direct thrombin inhibitor (D), is similar in efficacy to either
UFH or LMWH but causes less bleeding and is used just prior to and/or
during PCI; however, this is not available in the Philippines.
Harrison’s Principles of Internal Medicine, 20th Ed. (2018), p. 1871
MED.CP.CCC2.CASE-ON-CHEST-PAIN (B2023), p.11
8. A 28 year-old male with chronic cough, minimal hemoptysis, weight loss and
night sweats. His sputum GeneXpert is positive and negative for RIF B
resistance. He was previously treated for PTB in 2018. What is the
recommended regimen?
A. 2HRZE/6HRE
B. 2HRZE/4HR
C. 2HRZES/1HRZE/5HR
D. 2HR/4HR

Regimen 1
• 2 months of HRZE (intensive phase) followed by 4 months of HR
(maintenance phase)
• Indicated for all forms of TB excluding TB of the CNS (ex. TB
meningitis) and TB of the bones and joints (ex. Pott’s disease)
Regimen 2 U
• Longer regimen, total of 12 months: 2 months of HRZE (intensive
phase) followed by 10 months of HR (maintenance phase)
• Applicable for all forms of EPTB
MED.PULMO.1.13.Tuberculosis
9. A 52-year-old chronic smoker consulted because of 4 weeks of cough and Clinical Manifestations
dyspnea. On chest x-ray, there was a homogenous rounded density B • The prototypical lung cancer patient is a current or former smoker of
measuring 8cm with irregular borders. What is the presumptive diagnosis? either sex, usually in the seventh decade of life
A. Benign Lung Tumor
B. Malignant Lung Tumor
C. Round Pneumonia
D. Tuberculoma

BMED 2023 TC: LICLICAN, C


Chest Radiographs
• Growth Rate
o doubling time <30days or >400 days indicate benignity
o 2 year stability = benignity
o Enlarging = malignant
• Size
o 80% of lung masses >3cm is malignant
• Edge Characteristics
o Benign = Well-differentiated borders
o Malignant = irregular, elongated borders
Harrison’s Principles of Internal Medicine, 20th Ed. (2018), p. 541
MED.PULMO.1.07.LUNG-CANCER (B2023), p.3
10. A 65 year old female with previous treatment for PTB consults because of “After resolution of an acute infection in patients with recurrences (≧ 3 episodes
blood streaked sputum and occasional cough with phlegm. A suspicion of per year), the use of suppressive antibiotics to minimize the microbial load and
bronchiectasis for the patient's condition prompted an initiation of macrolide D reduce frequency of exacerbations has been proposed...Possible suppressive
therapy. Which of the following describes the use of long-term macrolide treatments include:
therapy for bronchiectasis? (1) administration of an oral antibiotic (e.g. ciprofloxacin) daily for 1-2
A. Decreases mortality weeks per month
B. Improves FEV1 significantly
(2) use of rotating schedules of oral antibiotics to minimize risk of drug
C. Improves quality of life
D. Reduces Exacerbations resistance and
(3) administration of a macrolide antibiotic daily or three times/week
(4) inhalation of aerosolized antibiotics
(5) intermittent administration of IV antibiotics for patients with severe
bronchiectasis
In relation to macrolide therapy, a number of double-blind, placebo-controlled,
randomized trials have been published in non-CF bronchiectasis and support a
benefit of long-term macrolides (6-12 months of azithromycin or erythromycin)
in decreasing rates of bronchiectasis exacerbation, mucus production and
decline in lung function.”
Harrison’s Principles of Internal Medicine, 20th Ed. (2018), p. 1985
2

11. Which of the following factors indicate the likely need for a procedure more Factors indicating the need for a more invasive procedure than a thoracentesis:
invasive than a thoracentesis, in a patient with suspected parapneumonic COMPLICATED PARAPNEUMONIC EFFUSION
pleural effusion? A Thoracentesis findings / diagnosis
A. Loculated pleural fluid • Loculated pleural fluid
B. Pleural fluid pH > 7.20 • Turbid fluid
BMED 2023 TC: LICLICAN, C
C. Pleural fluid glucose > 3.3 mmol/L (>60 mg/dL) • pH < 7 (< 7.2)
D. Pleural fluid N-terminal pro-brain natriuretic peptide (NT-proBNP) > 1500 • Glucose <3.3 mmol/L (<60 mg/dL)
pg/mL • Positive Gram stain or culture of the pleural fluid

Do not do thoracentesis when there are multiple loculations seen since you
cannot drain all loculations
- Refer to TCVS for more permanent drainage

Video-Assisted Thoracoscopic Surgery (VATS)


- Used for loculated pleural effusion & empyema
- Break up all locules then chest tube is inserted to drain the fluid
MED.PULMO.1.02. Pleural Diseases, p. 2,4,5
12. If an intubated patient on mechanical ventilator has respiratory acidosis on Respiratory acidosis is a disturbance brought about by alveolar
ABG, what strategy can you use to increase alveolar ventilation? HYPOventilation. So, with the use of the Alveolar Ventilation Equation:
A. Decrease tidal volume RR (Va) = RR (Vt – Vd)
B. Decrease respiratory volume
C. Decrease dead space - Wherein (Va) is alveolar volume, (Vt) is tidal volume, and (Vd) is
D. Increase FiO2 dead space
- Increasing the alveolar ventilation would mean an increase in the
tidal volume (Vt) and a decrease in dead space
C From Harrison’s Principles of Internal Medicine (20th Ed.), p.1948
Treatment goals for Hypoventilation
- Increase RR
- Increase tidal volume (Vt)
- Decrease dead space (Vd)
Medicine II Trans on Arterial Blood Gas, p. 3
**According to Doc Visperas’ exam ratio: FiO2 has no direct correlation to
ventilation
13. Which finding would preclude the use of nitrates in a patient presenting with Therapy with nitrates should be avoided in patients who present with low
chest pain? systolic arterial pressure (<90 mmHg) or in whom there is clinical suspicion
A. PR interval of 0.24 sec of RV infarction (inferior infarction on ECG, elevated jugular venous pressure,
B. JVP of 6 cm at 30 degrees clear lungs, and hypotension). Nitrates should not be administered to patients
C. Audible S3 and S4 who have taken a phosphodiesterase-5 inhibitor for erectile dysfunction within
D. SBP < 90 mm Hg the preceding 24 h, because it may potentiate the hypotensive effects of nitrates.
From Harrison’s Principles of Internal Medicine (20th Ed.), p. 1877 / A2023
D Medicine II Trans on ACS and Prinzmetal Angina, pp. 4 & 7

The common side effects of short-acting and long-acting nitrates include


headache, flushing, hypotension, syncope, postural hypotension, reflex
tachycardia, and methemoglobinemia, so it is contraindicated in hypotensive
patients (Choice D).
From Harrison’s Principles of Internal Medicine (20th Ed.), p. 1862 / A2023
Medicine II Trans on Ischemic Heart Disease, p. 10
14. Which physical examination is indicative of impending Type 2 respiratory Signs of Respiratory Failure
failure? A à Abdominal Paradox
A. Abdominal Paradox à Central Cyanosis
B. Crackles over both lung fields à Altered Sensorium
C. Respiratory rate of 28 cycles per minute MED.PULMO.1.12.Acute Respiratory Failure 2023 trans, Page 1
D. Sternocleidomastoid muscle tensions

BMED 2023 TC: LICLICAN, C


15. Which pathological change will most likely show Type 2 respiratory failure? Type II Respiratory Failure
A. Increased respiratory rate This type of respiratory failure is a consequence of alveolar hypoventilation
B. Increased tidal volume and results from the inability to eliminate carbon dioxide effectively.
C. Increased dead space C Mechanisms are categorized by impaired central nervous system (CNS) drive
D. Increased minute ventilation to breathe, impaired strength with failure of neuromuscular function in the
respiratory system, and increased load(s) on the respiratory system.
From Harrison’s Principles of Internal Medicine (20th Ed.), p. 2026

Pathophysiology
As ventilation decreases below 4-6 L/min, PaCO2 rises precipitously. A
decrease in alveolar ventilation can result from a reduction in overall (minute)
ventilation or an increase in the proportion of dead space ventilation. A
reduction in minute ventilation is observed primarily in the setting of
neuromuscular disorders and CNS depression. In pure hypercapnic respiratory
failure, the hypoxemia is easily corrected with oxygen therapy.

Hypoventilation is an uncommon cause of respiratory failure and usually occurs


from depression of the CNS from drugs or neuromuscular diseases affecting
respiratory muscles. Hypoventilation is characterized by hypercapnia and
hypoxemia. Hypoventilation can be differentiated from other causes of
hypoxemia by the presence of a normal alveolar-arterial PO2 gradient.
Kaynar, Ata Murat. 07 April 2020. Respiratory Failure. Medscape. Retrieved
from https://emedicine.medscape.com/article/167981-overview#a4
16. A 55-year old male diabetic is a case of PTB, bacteriologically confirmed, new. For Bacteriologically confirmed PTB, first follow up examination should be on
The first repeat sputum test will be after the ____ of treatment? A the end of intensive phase which is on the 2nd month, second follow up on the
A. Second Month 5th month and 3rd follow up on the end of treatment which is on the 6th month.
B. – MED.PULMO.1.13.Tuberculosis 2023 trans, Table 6. Page 4
C. –
D. –
17. A 42-year-old male consulted because of cough and chronic dyspnea. His Spirometry
spirometry showed the following results: B → Gold standard in diagnosing COPD
FEV1/FVC - low → Determine the severity of airflow limitation
FEV1 - low → Low FEV1
FVC - normal → Low FEV1/FVC
Change from pre- to post-BD FEV1 - 205ml → Low FEF25-75% (forced expiratory flow between 25% and 75% of FVC): for
% change from pre- to post BD FEV1 - 11 % small airways
Change from pre- to post-BD FVC - 180ml Is there a significant response to bronchodilator?
% change from pre- to post BD FVC - 12 % ● Look at two criteria: FEV1
→ 11% change from pre to post-BD (12% was NOT achieved)
What is the likely cause of his symptoms? → 205 mL criteria (200 mL WAS achieved)
A. –
B. COPD Now look at FVC, 12% WAS achieved, but 200 mL was NOT achieved, only
C. – 180 mL in this case.
D. – Moderately severe obstructive ventilatory defect with probable restrictive
ventilatory defect or RV hyperinflation with no significant response to
bronchodilator. (Please correlate results with patient’s clinical data)
Diagnosis: COPD
MED.PULMO.1.03.Chronic Obstructive Pulmonary Disease, Page 4
MED.PULMO.1.05B.SPIROMETRY CASES, Page 3-4

BMED 2023 TC: LICLICAN, C


18. Which of the following is a risk factor for sudden cardiac death in hypertrophic Discussion from Dr. Garcia:
cardiomyopathy? C A. Left ventricular interventricular septal thickness of 2.0 cm = it
A. Left ventricular interventricular septal thickness of 2.0 cm should be if the thickness is >3.0 cm; not 2.0 cm
B. ECG showing premature ventricular complexes in couplet B. ECG showing premature ventricular complexes in couplet = >3
C. Blood pressure of 110/70 at peak exercise beats at >120/min; “PVC alone, even in couplet, is not a risk factor
D. A sibling that died of acute coronary syndrome at age 56 years old unless the patient has episodes of ventricular tachycardia. 3 PVC
is already ventricular tachycardia. Basta nag ventricular
tachycardia ayun best risk factor; but, couplet lang, bali wala iyun”
C. Blood pressure of 110/70 at peak exercise = Correct answer;
“Halos walang makalabas na dugo. When you talk about
hypertrophic cardiomyopathy, the outflow is narrow that the
cardiac output and stroke volume is very small. Kaya peak
exercise na hindi pa tumataas yung blood pressure”
D. A sibling that died of acute coronary syndrome at age 56 years
old = more due to IHD; “this talking more of an ischemic heart
disease so hindi siya risk factor but if there is a sibling who had
sudden death, it is not coronary, and the sibling who died has
hypertrophic cardiomyopathy then it is a risk factor for the patient
who had sudden cardiac death”
Harrisons:

Harrisons 20th edition p. 1796. & Discussion with Dr. Garcia

BMED 2023 TC: LICLICAN, C


19. A 58 year old male had 5 day history of cough productive of whitish sputum,
malaise and fever with highest temperature of 38.8 degrees Celsius. He self C
medicated with Paracetamol and was given unrecalled antibiotics. At the
emergency room, oxygen saturation was 88% at room air, BP 110/60 R 110,
regularly regular, Temp. 38.6 degrees Celsius, R 30/minute. He is lethargic,
arousable on painful stimulation GCS 11, in supine position and with
respiratory distress. Crackles were heard from T8 down on the right, no
wheezes. He is a hypertensive and has had previous admission for CAP 3
months ago with use of broad spectrum antibiotics then. Chest Xray showed
infiltrates on the right lower and middle lung zones. Sputum GS yielded Gram
+ cocci in pairs; SARS COV-2 RT PCR was negative. Which of the following
treatment regimen should be empirically given?
A. Azithromycin 500 mg/IV OD+ Ciprofloxacin 400mg/IV OD
B. –
C. Ceftriaxone 2g/IV OD + Azithromycin dihydrate 500 mg/IV OD
D. –

• Based on the Risk Stratification of CAP, patient falls under Moderate-


risk CAP
• Patients with moderate-risk CAP are given IV Non pseudomonal B-
lactam (BLIC, Cephalosporin + Extended Macrolides or Respiratory
Fluroquinolones
o Ceftriaxone + Azithromycin dihydrate
MED.1.06.Community-Acquired Pneumonia, p. 2, 4
20. What technical aspect is being assessed when we look for the level of the right • Full Picture- hemidiaphragm area should be seen
hemidiaphragm? • Right hemidiaphragm is usually 1 ICS higher than the left
A. – o Normal location: level of the 10th rib (patient with full
B. – D
inspiration at total lung capacity)
C. –
MED. PULMO. 1.01. CHEST IMAGING
D. Full Inspiration
21. A 65y male, retiree consults due to persistent BP elevation of 140-150/90-110. Normal Sinus Rhythm
He brought his 12L ECG tracing which shows regularly occurring P-QRST with - Regular P waves must be present
R-R interval occupying 5 big squares. P wave amplitude of 2mm and P wave - Must be followed by QRST complex
duration of 2.5mm. PR interval occupying 4 mm, QRS duration occupying P wave
2mm, and QT interval occupying 10mm. This ECG is consistent with: - Height/ Amplitude: 0.5-2.5 mm; should not exceed 2.5 small squares
A. Normal sinus rhythm. Normal ECG tracing o 1 small square= 0.1 mV
B. Sinus bradycardia, with left atrial enlargement o 2.5 small squares= <2.5 mV
C. Sinus tachycardia with 1st degree AV block - Width/ Time: < or = 2.5mm (0.10 sec)
D. Normal sinus rhythm with complete bundle branch block o 1 small square= 0.04 sec
o 0.04 x 2.5 squares= <0.10 sec
A PR interval
- Normal: 0.12 to 0.20 sec (3-5 small boxes); should not occupy >1 big
box
- AV block: >0.20 sec; prolonged PR interval, >1 big box
QRS duration
- Normal: < or = 2.5m; 0.06-0.10 sec
- Bundle branch block: Wide or prolonged QRS duration
QT interval
- 0.30-0.40 sec
MED.CARDIO.1.04.NORMAL ECG p.8

BMED 2023 TC: LICLICAN, C


22. A 55y, female, known diabetic, hypertensive, with recurrent angina presents Stage A: At risk for development heart failure, but heart is structurally normal
at the ER due to easy fatiguability even when walking 30 meters and when (eg. Hypertension, diabetes, obesity)
preparing a simple meal for her husband. PE showed JVP 4cm at 30°, apex Stage B: Patient presents structural problems, but without any symptoms
beat at the 6th ICS LAAL, grade 1 bipedal edema, and bibasal crackles. Stage C: Presents with symptoms (easy fatigability, shortness of breath,
Echocardiogram showed hypokinetic septum and anterior wall with EF of 40%. PND, orthopnea, pedal edema); Condition may improve with medication
Which is true regarding our patient? Stage D: End-stage heart disease/ Intractable; Medications do not improve the
A. Patient has Stage A heart failure, NYHA class II conditions
B. Patient has Stage B heart failure, NYHA class II Doc: If patient presents with symptoms, already assume Stage C or D
C. Patient has Stage C heart failure, NYHA class III Stage A and B do not present symptoms
D. Patient has Stage D heart failure, NYHA class III
New York Heart Association Functional Class
C
Class I – No limitation of physical activity
Class II – Slight limitation of physical activity. Comfortable at rest. Ordinary
physical activity results in fatigue, palpitation, dyspnea
Class III – Marked limitation of physical activity. Comfortable at rest. Less
than ordinary activity causes symptoms.
Class IV – Unable to carry on any physical activity without discomfort.
Symptoms of heart failure at rest. If any physical activity is undertaken,
discomfort increases.
Medicine II Cardio 1.02. Heart Failure & Cardio-Pulmo Shifting exam
discussion w/ Dr. Garcia
23. In the CHADSVASC risk stratification for stroke in nonvalvular atrial fibrillation, CHADSVASC Score – Risk Stratification for Stroke in AF
which of the following is true? C Congestive heart failure/LV dysfunction 1
A. Heart failure carries the highest risk for stroke Hypertension 1
B. Hypertension has a higher score than diabetes Age > 75 2
C. A previous stroke or TIA carries a higher risk than diabetes Diabetes mellitus 1
D. Patients younger than 75 years old carry no risk for cardioembolic stroke Stroke/TIA/Thrombo-embolism 2
Vascular disease 1
Age 65-74 1
Sex category (Female sex) 1

Options A & C: Previous stroke/TIA/thrombo-embolism and age more than 75


years carry the highest risk for stroke (2)
Option B: Hypertension and diabetes have the same score (1)
Option D: Patients younger than 75 (i.e., 65-74 y/o) have a risk for stroke (1)
MED.CARDIO.1.08.Disorders of Rhythm p. 15 (A2023)
24. Which of the following drugs acts by improving myocardial glucose utilization? Option B: Ranolazine is a drug that blocks the late sodium current that
A. Trimetazidine facilitates calcium entry via the Na-Ca exchanger. relatively newer agent that
B. Ranolazine affects myocardial metabolism, effective for IHD and controls angina.
C. Ivabradine Option C: Ivabradine is a bradycardic drug that reduces cardiac rate by
D. Diltiazem inhibiting the sodium channel in the SA node
Option D: Diltiazem is a calcium channel blocker that is able to slow down the
A heart rate. Used in atrial fibrillation for rate control

Option A: Trimetazidine works by selectively inhibiting the enzyme LC-3KAT


which is the final enzyme in the free fatty acid beta oxidation pathway. This
increases the metabolic rate of glucose. Trimetazidine also restores the
homeostasis between glucose oxidation and glycolysis, an imbalance
occurring during ischemia
Cardio-Pulmo Shifting Exam Discussion with Dr. Garcia

BMED 2023 TC: LICLICAN, C


25. Which of the following ECG findings is most suggestive of ischemic heart Option A: Leads III & aVR are not contiguous leads
disease? Option C: QS pattern in leads III & aVF greater than 35 mm signifies LVH
A. 2 mm T wave inversion in III & aVR Option D: Regularly occurring PQRST signifies a normal sinus rhythm
B. QS pattern in III & aVF > 35 mm B
C. S in V1 plus R in V5 Option C: QS pattern or significant Q wave in contiguous leads signifies an
D. Regular PQRST in I avL, V5, V6 old infarct or ischemic heart disease
Cardio-Pulmo Shifting Exam Discussion with Dr. Garcia
26. A 35-year-old female, office clerk had hemoptysis of 580 mL in 4 hours. BP Massive Hemoptysis
was 90/70, CR 128. She cannot sustain wakefulness. Which of the following A - potentially life-threatening bleeding
will you prioritize? - >200mL to 600mL of blood expelled in a 24-hour period
A. Intubation
B. Blood Transfusion If patient is unstable (hemodynamically compromised with altered
C. Antibiotics sensorium), we need to protect the airway via intubation prior to any
D. Chest CT Scan procedure.

From Medicine II Trans on ER 2 Hemoptysis, pp. 3-5


27. A 22 year old student presented with 3 week history of cough productive of Based on the symptoms such as the 3 week history of cough and fever, a likely
yellowish sputum consulted at the out patient department. There was diagnosis to consider is tuberculosis
accompanying anorexia, low grade fever, 2-3 bouts of hemoptysis amounting
to 1 teaspoon per episode; and 2 lb. weight loss since 1 month ago. She has A common finding when screening using a chest x-ray in patients with TB is
had treatment with amoxicillin for 1 week duration, however cough persisted. cavitary lesions on the upper lung field (apex)
Chest radiograph was done. Which chest radiograph findings is expected
taking into account the patient's presentation? Other choices:
B
A. Bilateral basal infiltrates A: basal infiltrates are more commonly seen in patients with pneumonia
B. Cavitary lesions on apex C: Hyperaeration is more common in atelectasis
C. Hyperaeration D: Westermark’s sign is commonly seen in Pulmonary embolism
D. Westermark’s sign
MED.PULMO.2.13.TUBERCULOSIS p2
MED.PULMO.2.01.CHEST IMAGING (B2023)

BMED 2023 TC: LICLICAN, C


28. A 40y male, PE teacher came in for his quarterly check-up. He is a diagnosed
case of RHD with moderate mitral stenosis and was admitted at the ICU last A
year due to congestive heart failure. At present, he teaches tennis and soccer
to his students without any discomfort. What is the functional classification of
our patient?
A. Class I
B. Class II
C. Class III
D. Class IV

From Harrison’s Principles of Internal Medicine (20th Ed.), p. 1650


29. In a patient presenting with acute cerebral infarction and very elevated BP but According to the Criteria for Thrombolytic Therapy, patients can be divided into
has no funds for thrombolysis, at what BP will you start parenteral two groups – Cerebral infarct patients who are candidates for thrombolytic
antihypertensive medication? therapy, and those who are not.
A. SBP >200 mmHg
B. SBP >220 mmHg A cerebral infarct patient who is not a candidate for thrombolysis, such as the
C. DBP >100 mmHg case given, can start an antihypertensive medication at SBP of >220 mmHg.
D. DBP >120 mmHg See table below.

Cerebral infarct not candidate to Cerebral Infarct candidate to


thrombolytic therapy thrombolytic therapy
• Allow permissive Goal blood pressure
hypertension • <185 mmHg SBP
• Institute anti-hypertensives • <110 mmHg DBP
B at SBP >220 mmHg, DBP
>130 mmHg
• Maintain MAP of 110-30
mmHg
• Reduce the blood
pressure by 15% during
the first 24 hours after
onset of stroke
Additional info: In Hypertensive Emergencies, at BP of 220/130 mmHg or higher,
this would already require an immediate (but controlled) reduction of BP with IV
medication within the first hour. This would also require admission for parenteral
therapy with intra-arterial monitoring in the ICU
MED.CARDIO.CCC3.CASE WITH RIGHT-SIDED WEAKNESS 2023 Trans p.10-11
30. A young basketball player is brought to the hospital for sudden sharp right Possible diagnosis: Pneumothorax
sided chest pain and severe dyspnea. What IS NOT expected on his chest x- → Symptoms: pleuritic chest pain and dyspnea
ray?
• Pneumothorax is a volume containing abnormality, thus it is seen as
A. Cardiac apex on 5th LICS parasternal line a contralateral deviation or pushing of the midline structures.
B. Visible visceral pleural line with absence of lung markings lateral to it A
• Since the patient has sudden sharp chest pain on the right side, the
C. Widened intercostal spaces on the right and narrowing on the left
cardiac apex should be displaced to the left side, therefore
D. Trachea shifted to the left
cardiac apex on the 5th LICS parasternal line is not expected on his
chest x-ray.
BMED 2023 TC: LICLICAN, C
• Options B, C, and D are expected findings because:
→ B: Visible visceral pleural line with absence of lung markings is
pathognomonic for pneumothorax.
→ C: In right pneumothorax, the right side is volume containing since
there is air in the pleural space, thus appearing as widened ICS. On
the left side, there is narrowing due to volume loss.
→ D: Trachea is shifted to the left due to the contralateral deviation of
structures from the affected side.
From Medicine II trans on Chest Imaging, pp. 6-8
31. What is the acid base interpretation of the following gas result: pH: 7.31. Step 1: Look for pH (acidosis or alkalosis)
PaCO2: 44. HCO3: 22 using our algorithm? • 7.4 = best pH
A. Uncompensated respiratory acidosis • >7.4 = alkalosis
B. Fully compensated metabolic acidosis • <7.4 = acidosis
C. Partially compensated metabolic acidosis • 7.31 = ACIDOSIS
D. Uncompensated metabolic acidosis Step 2: Look at PaCO2 (respiratory or metabolic)
• NV = 35-45
o Given PaCO2 = 44 = normal
o Since it’s normal, the problem may not be respiratory. We
then look at HCO3
§ NV = 22-28
§ Given HCO3 = 22 = low normal
o HCO3 is a computed value and 22 maybe just rounded off
from a value <22 = METABOLIC
• For LOW pH
o paCO2 low (<35) = same direction = METABOLIC
o paCO2 high (>35) = opposite direction = RESPIRATORY
D • For HIGH pH
o paCO2 low (<35) = opposite direction = RESPIRATORY
o paCO2 high (>35) = same direction = METABOLIC
Step 3: Know if uncompensated, partially, or fully compensated

• If problem is metabolic, PaCO2 will compensate


• If problem is respiratory, HCO3 will compensate
• Given: metabolic acidosis
o PaCO2 is 44 = normal = UNCOMPENSATED
MED.PULMO.2.10.ARTERIAL-BLOOD-GAS (B2023) pp3,4
32. A 59y male was referred by his endocrinologist for further management of Core Drug Treatment for Uncomplicated Hypertension
HTN. In your clinic, BP is 170/100, PR 95pm, RR 21cpm. He is a diagnosed
hypertensive & diabetic, maintained on Telmisartan/HCTZ 40/12.5 1 tab qD & • For grade 1 hypertension, only lifestyle modifications are advised,
Sitagliptin/Metformin 50/1g 1 tab qD. Diagnostic procedures done showed unless the patient already has complications
B
LVH on ECG & cardiomegaly on Chest X-Ray. Hgb 90g/L, Hct 0.32, creatinine • Once pharmacological agents are started, it is highly recommended
173.3 umol/L, FBS 5.3mmol/L, HBA1C 6.4%, Na 132mmol, K 5.5mmol/L. How that the initial therapy is dual combination given as a single pill fixed
are you going to address the HTN of the patient? combination

BMED 2023 TC: LICLICAN, C


A. = • If the patient’s condition is still not controlled, proceed to the next
B. Shift Telmisartan/HCTZ to Amlodipine 10mg 1 tab qD, and add Metoprolol 1 step: triple combination
tab qD • If the patient’s condition is not controlled with triple combination, it is
C. – labeled as resistant hypertension
D. Maintain Telmisartan/HCTZ to 40/12.5 1 tab qD, and add Metoprolol 100 mg • Beta blockers (Metoprolol): given at any point if there is any
1 tab once a day compelling indications (HF, angina, post-MI, atrial fibrillation)
• Core treatment algorithm is also appropriate for patients with
hypertension-mediated organ damage, cerebrovascular disease, DM,
or PAD

From Medicine II Trans on Systemic Arterial Hypertension, pp. 11


33. CJ, a 35 year old teacher presents in your clinic because of palpitations which Patient likely has ASD
was noted since a month ago. On PE, she has an irregularly irregular rhythm,
a 3/6 murmur at the left base followed by splitting of S2 that was fixed. She A. Spironolactone is for HTN and HF
was also tachycardic with a rate of 110/min. The best approach to give CJ is B. (For HFrEF) Valsartan is an ARB which decreases afterload and
A. Medical Therapy that must include spironolactone C preload by inactivating RAAS
B. Medical Therapy that must include valsartan-sacubitril a. Sacubitril is an endopeptidase inhibitor which increases
C. Surgical or transcatheter closure of the shunt survival benefit when given with ARBs (p. 1774, Harrison’s)
D. Medical Therapy that is shunt anomaly-specific C. Atrial septal defects have traditionally been repaired by surgical
closure. Recently, transcatheter device closure has increasingly been
used with excellent results.
(https://pubmed.ncbi.nlm.nih.gov/15876892/)
D. Surgically-treated patients had a significantly better estimated 10-
year survival than those treated medically
(https://www.jwatch.org/jw199509010000006/1995/09/01/surgery-vs-medical-
therapy-atrial-septal-
defect?fbclid=IwAR2Bf0k_j3cVpRMaClGP15oPYHPyiliRVij8_ryc4yg0y6RT__It
ZLMo9Bs)

BMED 2023 TC: LICLICAN, C


34. A 48y female, known case of ASD, consults because of palpitations and loss Patient has palpitations and syncope which indicate arrhythmia; hence, our
of consciousness. She was previously advised to undergo closure of the shunt workup should document what type of arrythmia the patient has. Choices B and
but she refused any intervention. PE revealed regular rhythm with a fixed split C can monitor the electrical activity of the heart; however, the 12 Lead ECG will
S2. What should you request to work up her syncope? not give us significant results because the patient has a regular rhythm. What
A. 2D Echo with Color Doppler Studies we need is the 24 Hour Holter Monitoring which is an ECG that continuously
C
B. 12 Lead Electrocardiogram records the heart’s rhythm for 24 hours.
C. 24 Hour Holter Monitoring From hopkinsmedicine.org and Dra. Garcia’s discussion of answers.
D. Treadmill Stress Test

35. A 58 year old male had 5 day history of cough productive of whitish sputum, Lung ultrasound has some clinical application which can be used to aid in
malaise and fever with highest temperature of 38.8 degrees Celsius. He self B diagnosing the findings in the chest x-ray. Lung ultrasound is needed to further
medicated with Paracetamol and was given unrecalled antibiotics. At the investigate other causes and etiologic agents (such as pleural effusion)
emergency room, oxygen saturation was 88% at room air, BP 110/60 CR 110, responsible for the clinical manifestations as the patient did not improve after 5
regularly regular, Temp. 38.6 degrees Celsius, R 30/minute. He is lethargic, days of empiric antibiotics.
arousable on painful stimulation GCS 11, in supine position and with
respiratory distress. Crackles were heard from T8 down on the right, no Request ABG – may also be performed as the patient presents with decreased
wheezes. He is a hypertensive and has had previous admission for CAP 3 sensorium, however, findings from the ultrasound may guide the physician with
months ago with use of broad spectrum antibiotics then. Patient also has regards to the necessity in requesting for ABG
maintenance for very severe COPD in the form of inhaled ICS LABA plus Start IV Steroids – no signs or symptoms of exacerbation, wheezing, or
LAMA. Chest Xray showed infiltrates on the right lower and middle lung zones. increased sputum production
He was admitted and has been on empiric antibiotics for 5 days but still had Continue with current antimicrobials – patient did not improve after
fever and decrease in sensorium and on examination of the chest, decreased administration of empiric antibiotics for 5 days, this warrants further investigation
breath sounds over the right T8 down. Which should be done? on other possible causes
A. Request ABG
B. Do Lung Ultrasound The patient was already given empiric antibiotic for 5 days. Recognize the new
C. Continue with current antimicrobials PE findings which will be tied up with pneumonia complication (patient not
D. Start IV Steroids improving over the next 5 days) and investigate for reasons. Consider the
possibility of pleural effusion using lung ultrasound or x-ray

From Dra. Visperas’ discussion of answers

BMED 2023 TC: LICLICAN, C


MED.PULMO.2.01.CHEST-IMAGING (B2023) REFORMATTED p. 18
36. HC, 75y male, hypertensive, diabetic and a heavy smoker was seen because “Cardiac catheterization, which can measure cardiac blood pressure and
of nocturnal cough. Two weeks prior, he complained of effort related chest A flow, may be used to confirm the haemodynamic severity of the stenosis in
pain which is noted to occur even at rest. His ECG showed sinus rhythm and patients with inconclusive or discordant echocardiography. However, this
diffuse ischemia while the chest ×-ray revealed pulmonary congestion. The invasive technique is associated with increased risk of bleeding and
patient's daughter mentioned that his father was diagnosed with moderate to cerebral embolism.”
severe aortic stenosis 2 years ago but was lost to follow up. He was scheduled https://www.ncbi.nlm.nih.gov/pmc/articles/PMC5127286/
for emergency coronary angiography with possible revascularization. What is
the main safety consideration for this procedure for HC who has a concomitant “Simply crossing a calcified aortic valve with a catheter increases the risk
AS? of cerebral embolism.”
A. Attempts to cross a heavily calcified aortic valve with a cardiac catheter
for measurement of LV pressures are associated with risk of cerebral https://www.ncbi.nlm.nih.gov/pmc/articles/PMC3898978/
embolization
B. –
C. –
D. –
37. A 35 year old female came in due to 5 day history of fever, body malaise, calf pH 7.27 = acidosis (7.35-7.45)
pain and decreased urine output. On physical examination, she was awake C PaCO2 25 = low (35-45) , low so under base
but agitated and had the following vital signs: BP 150/100, HR 119, RR 32, T HCO3 14 = low (22-28) , low so under acid
38, 02 sat 95% room air. She had clear breath sounds, no edema but (+) PaO2 = normal (80-100)
asterixis. She was diagnosed with leptospirosis. ABG taken at room showed
the following: pH 7.27 PaCO2 25, HCO3 14, PaO2 95, 02 sat 95%, a/A 02
0.80. What is the ABG interpretation of this patient?
A. –
B. –
C. Partially compensated metabolic acidosis with normal oxygenation
D. –
Not discussed from Dr. Visperas Ratio.

MED.PULMO.1.10.ARTERIAL-BLOOD-GAS (B2023) p5

BMED 2023 TC: LICLICAN, C


38. A 26 year old male was admitted due to intermittent difficulty of breathing for Appendix A. Suggested treatment algorithm for use of fibrinolytics to treat
the past few days and was admitted because of increase in severity of the acute pulmonary embolism. Probability of Pulmonary Embolism is dictated by
dyspnea with episodes of dizziness and near syncope. The pretest probability hemodynamic parameters. Patients with low-risk PE (Submassive without RV
for VTE is high and CT PA showed filling defect on the right and left pulmonary strain) are given only heparin anticoagulation. If the patient has an RV strain,
artery and was already on enoxaparin 0.6 mI/SC BID. While admitted, patient heparin anticoagulation is initiated and fibrinolytics are indicated if there are
developed sudden drop in BP at palpatory 60. Which should be the treatment? evidences of increased severity. Alteplase is the preferred fibrinolytic agent.
A. – Obtained from Dr. Visperas’ PPT.
B. Institute Alteplase
C. –
D. –

MED-PULMO.1.12C - ER 2: Massive VTE (23 SEPTEMBER 2021)


39. In the management of hypertensive emergency with acute cerebral infarction, In acute ischemic stroke, guidelines always say to maintain the mean arterial
why do we allow permissive hypertension to a certain extent? pressure (MAP) in a certain level because cerebral perfusion pressure (CPP) =
A. – MAP - ICP. This allows permissive hypertension to occur, especially during the
B. To avoid compromising cerebral perfusion and increasing cerebral first week to ensure adequate cerebral perfusion. The penumbra (which is still
infarction B viable cerebral tissue) is dependent on MAP. In stroke, there is cerebral edema
C. – and this would increase the ICP. Therefore, we need to set the MAP to a certain
D. – degree in order to offset the increased ICP and still maintain cerebral perfusion.
MED.CP.CCC3.CASE WITH RIGHT SIDED WEAKNESS
40. A 26-year-old female with SLE was referred for spirometry to evaluate the Restrictive Lung Disease
cause of her dyspnea. If she has interstitial lung disease, which of the following • PAINT: pleura, alveoli, interstitium, neuromuscular, throacic cage
results is expected in her spirometry? • Inability to expand alveoli
A. Narrow flow volume loop A • Lung volume measured as TLC (FVC in spirometry)
B. – • Affects volume parameter (FVC in spirometry)
C. –
D. –
BMED 2023 TC: LICLICAN, C
Obstructive Lung Disease
• Disease of the airways
• Affects flow parameter (FEV1/FVC)

MED-PULMO 1.05 - SPIROMETRY (7 SEPTEMBER 2021)


41. A 20y male student athlete presented with dyspnea at rest, easy fatigability, The clue to the etiology of the symptoms in the case is, “...preceded by three
chest pain on deep inspiration one day prior to consult. It was preceded by D days of fever with maximum temperature of 38C, sore throat and malaise”.
three days of fever with maximum temperature of 38°C, sore throat and Myopericarditis refers to the inflammation of the myocardium and pericardium,
malaise. On PE BP 100/60 HR 99bpm RR 28 cpr T 37.9°C. AB 5th LICS MCL, which has an infectious and non-infectious cause. In this case, the cause is
normal S1/2, (+)S3 (+) grade 4/6 murmur at the apex, with pericardial friction infectious as evidenced by the fever, sore throat, and malaise. Myopericarditis
rub. What is your next best step? starts with viremia and the entry of the virus into the myocyte, leading to
A. Request for coronary calcium CT scan to rule out early atherosclerosis infection. The body’s immune response is a lymphocytic reaction with antibodies
B. Request for atrial natriuretic peptides level to rule in cardiac cause of that will transform into antibodies that are directed against the pathogen, the
dyspnea surface antigen, and the myocyte proteins (cross-reaction). The end result is
C. Request for myocardial perfusion scan to determine cardiac viability dilated cardiomyopathy, which causes symptoms of heart failure in our patient
D. Request for 2D echocardiogram to rule out myopericarditis (dyspnea, easy fatigability) and chest pain on deep inspiration due to
pericarditis.

MED.CARDIO.1.03.CARDIOMYOPATHY-AND-RELATED-DISEASES

BMED 2023 TC: LICLICAN, C


42. A 79y hypertensive diabetic male with previous history of atrial fibrillation Choice A: The patient’s BP is high and one of the risk factors of atrial fibrillation
presents with palpitations and shortness of breath. He had history of stroke 2 is uncontrolled hypertension so you need to optimize the antihypertensive
years ago and heart failure 1 year prior. On examination, BP 150/100 mm Hg, medications which will address both the blood pressure and the heart rate of the
HR 97 bpm irregularly irregular, RR 15. There were no murmurs. Neurologic patient (ex. Beta blocker or non-dihydropyridine CCB to slow down the heart
exam is normal. Your advice would be: rate).
A. He needs optimal antihypertensive treatment for blood pressure control
in order to decrease the risk for stroke and heart failure Choice B: Elective coronary angiogram – only for symptomatic patients such as
B. He has risk factors for coronary artery disease and must undergo elective those with chest pain (absent in the given case)
coronary angiogram to evaluate any significant stenosis
C. He has significant risk for stroke and will need at least an antiplatelet as Choice C: Antiplatelet is not enough to prevent thromboembolic events from
thromboembolic prophylaxis happening.
D. He will need an antiarrhythmic in order to maintain sinus rhythm for
symptom relief A Choice D: In the study comparing antiarrhythmic and rhythm control vs. rate
control, the outcome is the same. Antiarrhythmics have more side effects
because these are also pro-arrhythmics so only give it to patients with intractable
symptoms or symptoms you need to control (ex. fast heart rate which may lead
to heart failure, so you need to control it. And in our patient, he is only
experiencing palpitations because of the atrial fibrillation).

Additional information:
• Moderate to severe mitral stenosis or prosthetic heart valves
à If absent: Give NOAC
à If present: Give Warfarin
From Dra. Garcia’s discussion of answers
43. A transudative pleural effusion has the following characteristics on pleural fluid Light’s Criteria: Transudate vs Exudate
analysis: → Pleural fluid protein/serum protein > 0.5 (Choice A is an exudate)
A. Pleural fluid protein/ serum protein >0.6 → Pleural fluid LDH/serum LDH > 0.6 (Choice B is an exudate)
B. Pleural fluid LDH/ serum LDH >0.5 → Pleural fluid LDH > 2/3 ULN serum LDH (Choice C is a
C. Pleural fluid LDH less than two-thirds the normal upper limit for serum transudate)
D. Difference between the protein levels in the serum and the pleural fluid is
more than 31g/L (3.1 g/dL) C Choice D: If one or more of the exudative criteria are met and the patient is
clinically thought to have a condition producing a transudative effusion,
difference between the protein levels in the serum and the pleural fluid should
be measured. If the difference is >31 g/L (3.1 g/dL), the exudative categorization
by these criteria can be ignored because almost all such patients have a
transudative pleural effusion.
MED.PULMO 2.02 – Pleural Diseases pg. 5
44. A patient consults and tells you that she was earlier told to have congenital The physiology of an ASD is predominantly that of a “left-to-right” shunt (flow of
heart disease. The echo reports a dilated right ventricle and main pulmonary A pulmonary venous, or oxygenated, blood toward systemic venous, or
artery. Which of the following will produce these findings? deoxygenated, chambers or vessels). The degree of left-to-right shunting
A. Atrial Septal Defect determines the amount of right heart volume loading and is dictated by the size
B. Ventricular Septal Defect of the defect as well as the diastolic properties of the heart.
C. Patent Ductus Arteriosus
D. Mitral Valve Prolapse Right heart dilation, without additional etiology for such, in the setting of
unrepaired ASD is considered a risk for progression toward symptomatic right
heart failure, atrial arrhythmias, and potential development of pulmonary
arterial hypertension (if such is not already present)
From Harrison’s Principles of Internal Medicine (20th Ed.)
45. A 68-year-old female with repeated pulmonary infections in the past consults A BSI score of 3 is considered mild.
at the clinic due to chronic cough and sputum production. Crackles were heard B BSI:
0-4: Mild bronchiectasis

BMED 2023 TC: LICLICAN, C


over the right base. Which of the following management is recommended if 0-8: Moderate
her Bronchiectasis Severity Index (BSI) score is 3? 9+: Severe
A. Start Azithromycin (https://bronchiectasis.com.au/assessment/medical/bronchiectasis-severity-
B. Chest physiotherapy index)
C. N Acetylcysteine
D. Nebulization with budesonide Based on the stepwise management of non-CF bronchiectasis, the
management for mild bronchiectasis is Daily physiotherapy

FIG.14.MED2.PULMO.2.14.BRONCHIECTASIS
46. 38 year-old male, came in at the ER because of repeated episodes of Among choices A,B, and C, chest CT provides both a high diagnostic yield for
hemoptysis approximately 20 mL per episode, 4-5x a day. He is otherwise B location and etiology of hemoptysis. Since the patient’s condition is not
asymptomatic. For more information about its location and etiology, what will mentioned to be unstable, transferring them in order to do a CT scan is
you prioritize? possible. A CT scan of the chest is therefore the best diagnostic procedure for
A. CXR our patient.
B. Chest CT MED.PULMO.1.12.B.ER2.HEMOPTYSIS p. 3
C. Bronchoscopy
D. Sputum studies

47. A 30y female previously told to have RHD came for second opinion regarding “It shows regularly occurring P followed by regularly occurring QRS complexes
her ECG.: It shows regularly occurring P followed by regularly occurring QRS at 110 per minute”
complexes at 110 per minute. Lead Il shows P wave amplitude of 0.2mV with • Sinus tachycardia
P wave duration of 0.14 sec. In V1, P wave shows prominent negative P wave o Rate is >100/ min, Regular
with R wave of 10mm and S wave of 5mm. V5 shows R wave of 15mm and S o Always have a P followed by QRS 110 per minute
wave of 10mm. You will tell her that most likely she has • Not Ventricular tachycardia because although the rate us >100/min,
A. Normal sinus rhythm, left and right atrial enlargement and LVH there is a P wave. Thus, the impulse came from the SA node/
B. Sinus bradycardia with right atrial enlargement and RVH anywhere in the atria.
C. Sinus tachycardia with left atrial enlargement and RVH C P wave amplitude is normal at 0.2 mV (NV: £ 0.25 mV), but the duration is
D. Ventricular tachycardia with left and right ventricular hypertrophy prolonged at 0.14 sec (NV: £10 sec).

“In V1, P wave shows prominent negative P wave with R wave of 10mm and S
wave of 5mm. V5 shows R wave of 15mm and S wave of 10mm.”
RVH
- R wave: Very tall in V1, V2
- S wave: Very deep in V5, V6

BMED 2023 TC: LICLICAN, C


48. A 29 year old female, newly diagnosed with asthma, presented with shortness
of breath once a week and no nighttime awakenings. Based on the 2021 GINA
Guidelines, what is recommended as an initial controller for this patient?
A. Daily low-dose inhaled corticosteroid alone plus as needed short-acting B2-
agonist
B. –
C. –
D. –

49. What is the best treatment approach for a patient who is hypotensive with no B Lines
a lines and with numerous B lines on both hemithoraces on lung ultrasound? • comet-tail artifacts perpendicular to parietal pleura
A. – • Implication: fluid found in the interstitium of the lungs (normal)
B. Vasopressors • Normal: <3 B lines
C. – • Too many B lines = too much fluid in interstitium
D. – • ≥2: pulmonary edema
B
Clinical Implication:
- ≥2 B line = Pleural edema
- Give diuretics to the patient if with no contraindications
- IV fluid cannot be done safely in patients with hypotension;
Pressors will be an option
MED.PULMO.1.01.Chest Imaging, p. 15
50. A 22 year old student presented with 3 week history of cough productive of If TB is suspected, start first with the cardinal signs and symptoms
yellowish sputum consulted at the out patient department. There was B • If the patient presents with cough, unexplained fever, unexplained
accompanying anorexia, low grade fever, 2-3 bouts of hemoptysis amounting weight loss, or night sweats lasting for > 2 weeks, then the
to 1 teaspoon per episode; and 2 lb. weight loss since 1 month ago. She has patient is tagged as a case of presumptive TB.
had treatment with amoxicillin for 1 week duration, however cough persisted. • Plan of action is to request for microbiological confirmation via GENE
If this patient has had previous treatment for PTB in 2015, which diagnostic X-PERT (if readily available).
test is warranted to determine the disease activity? o Primary microbiological confirmatory testing tool especially
A. – in NCR wherein the incidence of drug-resistant TB cases is
B. Gene Xpert/Rif quite high.
C. – MED.PULMO.1.13.Tuberculosis, p. 2
D. –
51. What is the best treatment approach for a patient who is hypotensive with A Lines
visible A lines and few B lines on both hemithoraces on lung ultrasound? B • Implication: air in the lungs
A. Needle decompression • A lines=Air
B. Intravenous fluid infusion • Parallel to parietal pleura
C. Vasopressors • Equidistant from each other
D. Thoracentesis • How do I use this?
BMED 2023 TC: LICLICAN, C
o Rules out pulmonary edema
o Correlates to an LA pressure of </= 13 mmHg
§ No evidence yet of CHF
• Rule out atelectasis
o IV fluid infusion can be safely done in patient with
hypotension
MED.PULMO.2.01.CHEST-IMAGING (B2023) REFORMATTED p.15 of 19

Tension pneumothorax is a medical emergency caused by trapped


intrathoracic air that precipitates hemodynamic collapse.
Harrison’s Principles of Internal Medicine (20th Ed.), p. 76
52. A 50y patient with documented severe bicuspid aortic stenosis sought a In patients with severe aortic stenosis (AS) (valve area <1cm2), strenuous
second opinion regarding medical clearance prior to joining a bicycle club. The C physical activity and competitive sports should be avoided even in the
patient is asymptomatic with regular biking which he engaged in without asymptomatic stage. (Note that this is taken from the book verbatim)
consultation from his previous cardiologist. The club is planning a trek to
Tagaytay in the coming weekend. He insists he wants to improve his A is wrong because of the statement above; however, it is correct that care
cardiovascular fitness because his father died of a heart attack at age. Will must be taken to avoid dehydration and hypovolemia to protect against a
you clear him and why? significant reduction in CO in patients with severe AS
A. Yes. Our patient may engage in cycling but care must be taken to
avoid dehydration and hypovolemia to protect against a significant reduction B and D are wrong because of the statement above; strenuous activities and
in CO. competitive sports should be avoided even in the asymptomatic stage (no
B. Yes. Provided an exercise stress test will show that he will not symptoms at 10 mets in an exercise stress test).
develop symptoms at 10 mets.
C. No. Strenuous physical activity and competitive sports should be Harrison’s Principles of Internal Medicine (20th Ed.), p. 1806
avoided, even in our patient who is asymptomatic.
D. No. Our patient can only engage in sports such as basketball and
tennis provided they do not last longer than 1 hour.
53. A 23-year-old male student consults with episodic dyspnea, chest tightness Assessment of Asthma Control
and cough. He has (+) atopy. He has consumed 12 canisters of Salbutamol C • Asthma is a chronic disorder thus regular and long-term follow up and
the past 12 months. For the past 4 weeks he has nocturnal cough (3x a week) assessment is necessary
and has missed school for the past week. But he claims he only has mild • Consists of 2 domains
shortness of breath and is satisfied with his short-acting B2-agonist. How o Assess symptom control over the last 4 weeks
would you go about in further assessment of his asthma? o Assess risk factors for poor outcomes, including low lung function
A. He has good control
B. Would trust the patient and ask no more questions According to the GINA guidelines, the patient should be asked if in the past 4
C. Would perform asthma control scoring weeks, has he/she experienced:
D. Would refer to a specialist • Daytime asthma symptoms more than twice/week?
• Any night waking due to asthma?
• Need for reliever for symptoms more than twice/week?
• Any activity limitation due to asthma?
Scoring: 0 = well controlled, 1-2 = partly controlled, 3-4 = uncontrolled
MED.PULMO.1.04.Asthma, p. 4
54. A normal to decreased left ventricular wall thickness is a feature of which of According to this table entitled "Phenotypic Classification of Cardiomyopathy",
the following cardiomyopathy phenotype? C left ventricular wall thickness varies from normal to decreased levels.
A. –
B. –
C. Dilated Cardiomyopathy
D. –

MED.CARDIO.1.03.CARDIOMYOPATHY-AND-RELATED- DISEASES
BMED 2023 TC: LICLICAN, C
55. An 85-year-old diabetic hypertensive female, smoker was seen because of left Peripheral Artery Disease is defined as a clinical disorder in which there is
calf pain while walking, this was accompanied by pain in the right buttocks C stenosis or occlusion in the aorta or the arteries of the limbs... As in patients with
which occur during exertion and which she attributes to a minor fall 10 years atherosclerosis of the coronary and cerebral vasculature, there is an increased
ago. She is sporadically compliant to medications and continues to smoke risk of developing PAD in cigarette smokers and in persons with diabetes
although she insists, she has cut down significantly. A lumbosacral series and mellitus, hypercholesterolemia, hypertension, or renal insufficiency.
hip ×-ray were unremarkable except for degenerative changes. An ultrasound
of the abdominal aorta showed widespread atherosclerosis but no aneurysm. The primary sites of involvement are the abdominal aorta and iliac arteries
What is your working diagnosis? (30% of symptomatic patients), the femoral and popliteal arteries (80-90%
A. She has PAD involving the aortoiliac vessels of patients), and the more distal vessels, including the tibial and peroneal
B. She has PAD involving the femoral-popliteal vessels arteries (40-50% of patients)
C. Both are correct
D. Neither are correct The most common symptom is intermittent claudication, which is defined as
a pain, ache, cramp, numbness, or a sense of fatigue in the muscles; it occurs
during exercise and is relieved by rest. The site of claudication is distal to the
location of the occlusive lesion. For example, buttock, hip, thigh, and calf
discomfort occurs in patients with aortoiliac disease, whereas calf
claudication develops in patients with femoral-popliteal disease.
Harrison’s Principles of Internal Medicine (20th Ed.), p. 1923
56. An 85-year-old female was admitted for syncope. She also reports tiring easily Exertional dyspnea, angina pectoris, and syncope are the three cardinal
when walking about her garden, an activity she used to perform with ease until symptoms of Aortic Stenosis.
one month ago. On auscultation, a 3/6 systolic rasping murmur is audible at
the base of the heart. The working diagnosis is valvular aortic stenosis. To Often, there is a history of insidious progression of fatigue and dyspnea
what will you attribute the patient's exertional dyspnea? associated with gradual curtailment of activities and reduced effort tolerance.
A. Elevation of pulmonary capillary pressure A
B. Concomitant severe aortic regurgitation Dyspnea results primarily from elevation of the pulmonary capillary pressure
C. A possible occult acute lung infection caused by elevations of LV diastolic pressures secondary to impaired relaxation
D. Elevation of systemic pressure and reduced LV compliance.
Harrison’s Principles of Internal Medicine (20th Ed.), p. 1804
57. In the management of atrial fibrillation, which of the following is the most Anticoagulants has been shown to reduce the risk of stroke and death in
appropriate? patients with atrial fibrillation. Majority of patients warrant chronic
A. In patients with atrial fibrillation, cardioversion to sinus rhythm equates to anticoagulation.
higher survival
B. Rhythm control with antiarrhythmics is associated with better outcomes Choice A is wrong because when Atrial fibrillation is present for >48h in patients
and lesser stroke compared to rate control with high-risk thromboembolism, such as conversion to sinus rhythm it is
C. Anticoagulation for stroke prevention is associated with lower mortality associated a delayed atrial mechanical function and thrombi formation thus
among patients lower survival
D. Anti-platelets are associated with lower risk for stroke and lesser bleeding C
compared to anticoagulants Choice B, in a randomized control trial, administration of antiarrhythmic
medications to maintain sinus rhythm did not improve survival or symptoms
compared to a rate control strategy

Choice D, Antiplatelets agents are inferior to warfarin(anti-coagulants) for


stroke prevention in Atrial fibrillation and do not have less risk for bleeding
Harrison’s Principles of Internal Medicine 20th Ed p 1746-1748
58. Which of the following class of drugs improves perfusion and blood pressure In patients with acutely decompensated heart failure with low blood pressure the
in acute decompensated heart failure? only thing that will improve the blood pressure is inotropic support which is your
A. Oral Beta Blockers dobutamine infusion.
B. Intravenous Loop Diuretic
C
C. Dobutamine Infusion Oral Beta Blockers – we do not give in acute decompensated Heart failure
D. Oral Digoxin patients because the more the patient would go into heart failure Due to its
negative inotropic properties

BMED 2023 TC: LICLICAN, C


Loop Diuretic – will unload the patient but it will not improve perfusion and blood
pressure

Oral digoxin – it increases contractility but has no effect in blood pressure

INOTROPIC THERAPY
“Impairment of myocardial contractility often accompanies ADHF (Acute
Decompensated Heart Failure), and pharmacologic agents that increase
intracellular concentration of cyclic adenosine monophosphate via direct or
indirect pathways, such as sympathomimetic amines (dobutamine) and
phosphodiesterase-3 inhibitors (milrinone), respectively, serve as positive
inotropic agents. Their activity leads to an increase in cytoplasmic calcium.
Inotropic therapy in those with a low-output state augments cardiac
output, improves perfusion, and relieves congestion acutely.”
From Dra. Garcia’s discussion of answers
Harrison’s Principles of Internal Medicine (20th Ed.), p. 1772
59. What the most appropriate management for an elderly patient with moderate CPAP
sleep apnea, a BMI of 30 and somnolence? C ● CPAP Use
A. Lifestyle changes → Gold standard treatment for sleep apnea: Continuous Positive Airway
B. Mandibular appliance Pressure (CPAP) |U
C. Continuous positive airway pressure → Breathing of patients with CPAP will be better after use. |&
D. Bariatric surgery → Used to assess extubation potential in patients who have been effectively
weaned and who require little ventilatory support and in patients with intact
respiratory system function who require an endotracheal tube for airway
protection|&
→ Improves daytime hypercapnia and hypoxemia in more than half of
patients with OHS and concomitant OSA|&
● CPAP Mechanics
→ Pushes positive pressure that will kick the airway open and splint it,
preventing airways from collapsing. |U
→ Increase in pressure will result to increase in airway caliber. |U
→ Titration sleep studies will determine the best pressure suited for the
patient
→ The patient must use it for at least 4 hours at night time. Must likewise
check the adherence for it to work.
● OTHER CHOICES:
→ Not A: Lifestyle changes, such as weight loss, may also be advised since
the BMI of our patient is 30. However, this is not the BEST answer for this
question.
→ Not B: Mandibular appliances are suitable for younger patients who are
not obese and have only mild sleep apnea. Since our patient is already an
elderly with moderate sleep apnea, this is not the treatment of choice.
→ Not D: Bariatric surgery is usually recommended for patients with BMI
above 40.
MED 2- PULMO 1.09 – Sleep disordered breathing (pages 7 & 10)
60. What does an elevated PaCO2 indicate? Respiratory hypoxemia may also be caused by hypoventilation, in which case
A. Diffusion Anomaly B it is associated with an elevation of Paco2
B. Hypoventilation
C. Shunt Harrison’s Principles of Internal Medicine (20th Ed.), p. 235
D. Ventilation perfusion mismatch

BMED 2023 TC: LICLICAN, C


Mechanism of hypoxemia
→ Check if PaCO2 is increased
■ Increased PaCO2–possible hypoventilation
Med.Pulmo.WS1.CXR and ABG page 6
61. A 30-year-old female pregnant patient with Marfan's syndrome is seen at the 12L Electrocardiogram is a valuable record of the heart’s electrical activity. It is
ER because of chest pain. Her BP was noted to be unstable and there are bi- C an important diagnostic tool in the evaluation of cardiovascular diseases
basal crackles present. Your working diagnosis is aortic dissection. What is • Abnormal heart rhythm (arrhythmias)
the best diagnostic procedure to request in the ER to establish the diagnosis? • Myocardial ischemia/infarction
A. 12L Echocardiogram • Chamber enlargement
B. Chest radiograph • Electrolyte imbalance
C. Transthoracic echocardiography MED.CARDIO.1.04.NORMAL-ECG
D. MR aortography
Chest Radiography
• Plain radiography of the chest is performed routinely when patients
present with acute chest discomfort and selectively when individuals
who are being evaluated as outpatients have subacute or chronic pain
• It is most useful for identifying pulmonary processes such as
pneumonia or pneumothorax

Transthoracic Echocardiography (TTE)


• It is diagnostic in patients with mechanical complications of MI or in
patients with pericardial tamponade.
Harrison’s Principles of Internal Medicine (20th Ed.), pp. 80

According to Dr. Garcia, “Through transthoracic echocardiography, you can see


that the aortic root is dilated, the flap where it was dissected, and also severe
aortic regurgitation.”

TTE permits adequate assessment of several aortic segments, particularly the


aortic root and proximal ascending aorta and in most cases, the aortic arch,
proximal descending aorta and abdominal aorta. TTE is most useful when the
flap is located in the aortic root.
https://www.ncbi.nlm.nih.gov/pmc/articles/PMC6454227/

BMED 2023 TC: LICLICAN, C


Aortography
• Aortography in the cardiac catheterization laboratory visualizes
abnormalities of the ascending aorta, including aneurysmal dilatation,
and involvement of the great vessels, as well as dissection with
compression of the true lumen by an intimal flap that separates the
true and false lumina
Harrison’s Principles of Internal Medicine (20th Ed.), pp.1713
62. A 58 year old male had 5 day history of cough productive of whitish sputum, The patient has an obstructive disorder. You will have to cover more bacterial
malaise and fever with highest temperature of 38.8 degrees Celsius. He self D infection which are the etiology for COPD patients.
medicated with Paracetamol and was given unrecalled antibiotics. At the A. Piperacillin tazobactam – not considered because there is no
emergency room, oxygen saturation was 88% at room air, BP 110/60 CR 110, macrolide included. It can be considered if there is a macrolide or
regularly regular, Temp. 38.6 degrees Celsius, R 30/minute. He is lethargic, quinolone included.
arousable on painful stimulation GCS 11, in supine position and with B. Vancomycin – not considered because we have to cover gram (-)
respiratory distress. Crackles were heard from T8 down on the right, no organisms and streptococcus. Not usually staphylococcus not unless
wheezes. He is a hypertensive and has had previous admission for CAP 3 you are thinking of abscesses or Hospital-acquired pneumonia
months ago with use of broad spectrum antibiotics then. Patient also has C. Ceftriaxone + Levofloxacin - although there is Levofloxacin but
maintenance for very severe COPD in the form of inhaled ICS LABA plus azithromycin has a better coverage for atypical organisms and at the
LAMA. Chest Xray showed infiltrates on the right lower and middle lung zones. same time although ceftriaxone has a good coverage for
Which of the following should be given for treatment? streptococcus, you are dealing with infiltrates bilaterally or in two zones
A. Piperacillin tazobactam 4.5 g/IV OD or lobes, right lower and middle, so you want to be more aggressive
B. Vancomycin 15mg/kg q 12h antimicrobial coverage in the form of carbapenems with azithromycin.
C. Ceftriaxone 2 g/IV OD + Levofloxacin 750mg/IV OD Ceftriaxone and Levofloxacin is lacking if there is more than one lobe
D. Meropenem 1 g/IV OD + Azithromycin dihydrate 500 mg/IV OD affected. If the infection is new or only affects one lobe then
Ceftriaxone + Levofloxacin can be considered.
From Dr. Visperas discussion of answers
63. Which of the following patient characteristics and symptomatology would In angina pectoris, when the patient is asked to localize the sensation, he or she
make you suspect angina? D typically places a hand over the sternum, sometimes with a clenched fist, to
A. 51 year-old woman indicate a squeezing, central, substernal discomfort (Levine’s sign).
B. Decrescendo pain pattern
C. Pain radiating above the mandible MED.CARDIO.1.06.1.Ischemic Heart Disease, p. 3
D. Clenched fist over the sternum
64. In the work-up of a patient with HTN, which of the following are considered Request these to be able to identify comorbidities and target organ damage
basic diagnostic procedures? C
A. Chest X-ray PA, Lateral & 12L Electrocardiogram
B. BUN, Serum Creatinine, Renal ultrasound
C. Thyroid stimulating hormone, FBS, lipid profile
D. Serum potassium, calcium, magnesium, inorganic phosporus

• Electrolytes (Serum Na, K, Ca, TSH)


à Determines if patient has primary aldosteronism,
hyperparathyroidism, hypo/hyperthyroidism
• Hgb and Hematocrit
à Polycythemia can elevate blood pressure
• ECG
à Earliest sign of target organ damage in ECG: LVH
MED.CARDIO.1.10 Systemic Arterial Hypertension, p. 7
BMED 2023 TC: LICLICAN, C
65. A 38y female lawyer consults due to frequent palpitations and occasional Elevated BP, PR, absent a wave, and an irregularly irregular rhythm all points
breathlessness when climbing to the 2nd floor. PE revealed BP 150/90, PR out to an atrial fibrillation. On the other hand, mitral stenosis is also presented
112bpm irregularly irregular, RR 21cpm. JVP 3cm at 45 degree with absent a because of her RV heave, LA lift, and diastolic thrill at apex. Intervention is not
wave. Hyperdynamic precordium with RV heave and LA lift, & diastolic thrill at an option since on 2D Echo, the patient’s valve is only mildly diseased. Valve
the apex. (+) occasional crackles at the lung bases : 2D Echo-Doppler replacement is also not ideal since the risk of surgery is high and the
revealed mild mitral stenosis, Wilkin score of 6 with mild mitral regurgitation complications might be difficult for a female patient especially if she gets
and mild pulmonary HTN. How are you going to manage this patient? pregnant. Therefore, choice A,B, and D are not options for the management of
A. Start ASA 100mg qD, Metoprolol 50mg 1 tab qD, Captopril 25mg ½ tab this patient.
BID, and prepare for mitral valve replacement with mechanical valve
B. Start ASA 100mg qD + Clopidogrel 75mg qD, Digoxin 0.25mg 1 tab qD, C A NOAC or warfarin would be the best management among the choices because
Perindopril 5mg ½ tab qD, and prepare patient for mitral valve ASA and Clopidogrel will not prevent the patient from having a stroke. However,
replacement warfarin is very difficult to administer because it requires to be titrated first, has
C. Start Apixaban 5mg 1 tab BID, Bisoprolol 2.5mg 1 tab qD, Candesartan a lot of drug interactions, and INR is required to be checked frequently.
8mg 1 tab qD and regularly follow up and 2D Echo-Doppler to note for From Dr. Visperas’ discussion of answers
progression
D. Start Warfarin 2.5mg 1 tab qD to achieve INR of 2-3, Diltiazem 30mg 1
tab qD, Losartan 50mg 1/2 tab qD, and prepare for percutaneous
transvenous mitral balloon commissurotomy
66. A 58 year old male had 5 day history of cough productive of whitish sputum, Intubation should be done ASAP since the patient is in respiratory distress,
malaise and fever with highest temperature of 38.8 degrees Celsius. He self lethargic, and has an oxygen saturation of 88%.
medicated with Paracetamol and was given unrecalled antibiotics. At the
emergency room, oxygen saturation was 88% at room air, BP 110/60 CR 110,
regularly regular, Temp. 38.6 degrees Celsius, R 30/minute. He is lethargic, *Not discussed in Doc Visperas’ ratio
arousable on painful stimulation GCS 11, in supine position with respiratory
distress. Crackles were heard from T8 down on the right, no wheezes. SARS
D
COV2 RT PCR was negative, which of the following should be given/ done
ASAP?
A. Furosemide
B. D50 Water
C. Oxygen supplementation
D. Intubation

67. A 70y hypertensive diabetic male presents in the R with sudden onset left Recommendation:
sided weakness of one hour duration. On examination, BP 160/100 mm Hg, • 12L-ECG
HR 119 bpm irregularly irregular, HR 99 bpm irregularly irregular, R 15. • Neuroimaging
Auscultation revealed varying intensity of S1. There were no murmurs. MMT • Lab tests and other ancillary procedures
3/5 on left upper and lower extremities. Which of the following is the most
appropriate statement? Magnetic Resonance Imaging:
A. Transthoracic 2D echocardiogram will help rule out left atrial thrombus • T12, T2W – age of the infarct, slightly bright = late hyperacute
and negate the need for anticoagulation B phase infarct
B. Cranial MRI with MRA can assess the presence of hyperacute cerebral • Final impression: LATE HYPERACUTE ISCHEMIC INFARCT,
infarct and cerebral atherosclerosis LEFT CAPSULOGANGLIONIC REGION
C. Plain cranial CT scan must be done immediately to rule out intracranial
tumors as a cause of the patient’s problem MED.CARDIO.CCC3.CASE WITH RIGHT-SIDED WEAKNESS 2023 Trans p.5-6
D. 24 hour Holter monitor must be done to assess cardiac rhythm and
assess need for permanent pacemaker

68. Which of the following diagnostic tests is capable of evaluating left ventricular 2D-Echocardigraphy
ejection fraction? C • Ejection fraction
A. 12 lead electrocardiogram o Used in assessing left ventricular function
B. Treadmill stress test o EF (%) = SV / LEVD x 100
C. 2d echo-Doppler • Visualize wall motion abnormalities
D. Coronary CT angiogram • Normal: 55% and above

BMED 2023 TC: LICLICAN, C


12 lead electrocardiogram
• To record heart’s electrical activity
• Clues suggestive of IHD
Treadmill stress testing
• To confirm and have objective evidence that patient has IHD
• Most widely used test for diagnosis of IHD
Coronary CT angiogram
• 3D reconstructed images of epicardial vessels

MED.CARDIO.1.06.1.Ischemic Heart Disease, p. 5


69. A 42-year-old Filipino female, non-smoker, with no known significant Adenocarcinoma
environmental exposure consulted because of an incidental finding of a A • Most common in non-smokers
peripheral lung mass on chest x-ray. If malignancy is considered, what is the • Most common histologic type in women and young adults
presumptive histopathologic cell type based on her features? • Peripheral lesion – because non-smoking
A. Adenocarcinoma • Most common non-small cell lung cancer (50%)
B. Neuroendocrine tumor o NSCLC arises from epithelial cells
C. Small cell lung cancer
D. Squamous cell carcinoma *Squamous & small-cell carcinomas are most associated with heavy tobacco
use
MED.PULMO.1.07.Lung Cancer, p. 6
70. A 74y male with symptoms of chest heaviness while seated on the couch and New York Heart Association Functional Class
talking to a friend has a NYHA functional class of ____. D Class I – No limitation of physical activity
A. I Class II – Slight limitation of physical activity. Comfortable at rest. Ordinary
B. II physical activity results in fatigue, palpitation, dyspnea (shortness of breath)
C. III Class III – Marked limitation of physical activity. Comfortable only at rest. Less
D. IV than ordinary activity causes symptoms.
Class IV* – unable to carry on any physical activity without discomfort.
Symptoms of heart failure at rest. If any physical activity is undertaken,
discomfort increases.

*Patient is class IV since there are symptoms even while seated and talking.
From Dra. Garcia’s discussion of answers
71. A 65 year-old male with 35 pack-year smoking history, presented with Classification of Severity of Airflow Limitation
progressive shortness of breath. You requested spirometry which revealed the C In patinets with FEV1/FVC <0.70:
FEV1 was 35% predicted, what is the airflow limitation severity? GOLD 1 (Mild) – FEV1 > or equal to 80% predicted
A. GOLD 1 GOLD 2 (Moderate) – 50% < or equal to FEV 1 <80% predicted
B. GOLD 2 GOLD 3 (Severe) – 30% < or equal to FEV1 <50% predicted
C. GOLD 3 GOLD 4 (Very severe) – FEV1 <30% predicted
D. GOLD 4 MED.PULMO.1.03.Chronic Pulmonary Disease, p. 4
72. An 80-year-old hypertensive, diabetic male with ischemic heart disease was Transthoracic Needle Aspiration (taken from lung cancer diagnosis
referred because of a solid pulmonary nodule with high-risk features for A algorithm)
malignancy. The mass is located in the right lower lobe adjacent to the • For diagnosis of solitary pulmonary nodule
posterior chest wall. What is the best way to confirm the diagnosis in this case? • Also amenable for peripheral lesions such as adenocarcinoma or
A. Transthoracic biopsy (TTNA) large cell carcinoma
B. Sputum cytology • Alternatives: EBUS-GS or EMN
C. Transbronchial biopsy (TBNA)
MED.PULMO.2.07.Lung Cancer, p. 6
D. Tumor resection
73. A 58 year old male had 5 day history of cough productive of whitish sputum, This is a case of Community acquired pneumonia.
malaise and fever with highest temperature of 38.8 degrees Celsius. He self B CRB 65
medicated with Paracetamol and was given unrecalled antibiotics. At the • British Thoracic Society Classification

BMED 2023 TC: LICLICAN, C


emergency room, oxygen saturation was 88% at room air, BP 110/60 CR 110, • Scores 1 point for each:
regularly regular, Temp. 38.6 degrees Celsius, R 30/minute. He is lethargic, o Confusion
arousable on painful stimulation GCS 11, in supine position with respiratory o Respiratory rate >30/min
distress. Crackles were heard from T8 down on the right, no wheezes. If CRB o Blood pressure (Systolic BP <90mmHg; Diastolic BP
65 score will be done, what is the patient's stratification for risk of death? <60mmHg
A. Low risk o Age >65
B. Intermediate risk • Higher score = higher risk of mortality
C. Moderately high risk Score:
D. High risk 0 – very low risk (usually does not require hospitalization
1-2 – increased risk (consider hospitalization)
• Patient is in respiratory distress
3-4 – high risk of death (urgent hospitalization, may also consider ICU
admission)
MED.1.06.Community-Acquired Pneumonia, p. 3
74. HR a 70y male was seen in the ER for chest pain radiating to the back. His Treatment of Aortic Dissection
BP was 220/100 and the pulse rate was 140/ minute, irregularly irregular. His Medical therapy should be initiated as soon as the diagnosis is considered
ECG showed sinus rhythm with episodes of atrial fibrillation with rapid • Admit to ICU for hemodynamic monitoring
ventricular response and LVH with non-specific ST T wave changes. His chest • Unless hypotension is present, therapy should be aimed at reducing
x ray showed a widened mediastinum. A transesophageal echocardiogram cardiac contractility and systemic arterial pressure, and thus
showed dissection of a thoracic aneurysm. The antihypertensive of choice for shear stress
HR is • For acute aortic dissection, unless contraindicated
A. IV Diazoxide o Beta-adrenergic blockers should be administered
B. IV Sodium nitroprusside parenterally, using IV propranolol, metoprolol, or short-
B
C. IV Minoxidil acting esmolol to achieve ~60 beats/min
D. IV Digitalis o Accompany with sodium nitroprusside infusion to lower
systolic BP to < or equal to 120mmHg
Sodium nitroprusside
• Arterial and venodilator
• Used for CV surgery
MED.CARDIO.1.10.Systemic Arterial Hypertension, p. 11
Harrison’s Principles of Internal Medicine, 20th edition (p. 1921)
75. An 88y patient consults because of syncope. ECG showed the following “The ECG tracing shows a sinus rhythm, followed by another sinus rhythm,
tracing. followed by another sinus rhythm, then a flat line (asystole – period of no
electrical activity) – this shows sick sinus syndrome. In a patient who had
syncope, they need a pacemaker.

It is NOT a 2nd degree AV block because there’s no P wave (2nd degree AV block
has P but no QRS). Since it’s a flat line, there’s total pause of the heart, so its
sinus arrest or sinus pause.

D It is NOT a complete heart block because you see regular P to P and regular R
Which of the following is the most appropriate statement? to R but they are not associated with each other (also called AV disassociation).
A. The tracing shows 2nd degree AV block Mobitz type 1 and does not Thus, the answer is sinus rhythm with long sinus arrest and patient needs a
warrant treatment pacemaker.”
B. The tracing shows 2nd degree AV type 2 and the patient needs further
From Dra. Garcia’s discussion of answers
workup
C. The tracing shows complete heart block and the patient needs a
permanent pacemaker
D. The tracing shows sinus rhythm with long sinus arrest and the patient
needs a permanent pacemaker

BMED 2023 TC: LICLICAN, C


76. Based on the GOLD strategy, if your COPD patient is still symptomatic after In the COPD GOLD Guidelines 2019, after the initiation of the initial
your initial pharmacologic therapy, which of the following features/traits should pharmacologic therapy, it is important to check the patient is symptomatic. If the
be considered? patient is still symptomatic, dyspnea or exacerbations should be considered
A. – as different modifications to the therapy are necessary.
B. Dyspnea or exacerbation
C. –
D. –

MED.PULMO.1.03. Chronic Obstructive Pulmonary Disease, p. 6


77. A patient presented to the ER in severe distress. A stat CTPA (PE protocol) Pulmonary Embolism
reveals obstruction affecting > 70% of the pulmonary vasculature. Which of C Massive PE accounts for 5–10% of cases, and is characterized by extensive
the clinical presentations would be most likely in our patient? thrombosis affecting at least half of the pulmonary vasculature. Dyspnea,
A. – syncope, hypotension, and cyanosis are hallmarks of massive PE. Patients
B. – with massive PE may present in cardiogenic shock and can die from multisystem
C. Dyspnea, syncope, hypotension, and cyanosis organ failure.
D. –
Submassive PE accounts for 20–25% of patients, and is characterized by RV
dysfunction despite normal systemic arterial pressure. The combination of right
heart failure and release of cardiac biomarkers indicates a high risk of clinical
deterioration.

Low-risk PE constitutes about 65–75% of cases. These patients have an


excellent prognosis.
Page 1911, Harrison’s Principle of Internal Medicine 20th Ed.
78. A 65 year-old male took a walk to the park just 2 blocks away from his house. Grade II
After reaching the park, he experienced chest pain described as "heaviness" B • The patient experienced chest pain after walking 2 blocks away from
which lasted 4 to 5 minutes. His Canadian Cardiovascular Society functional his house
classification of angina is:
A. I
B. II
C. III
D. IV

BMED 2023 TC: LICLICAN, C


79. What is the most appropriate intervention for a patient with a hemorrhagic Invasive mechanical ventilation
stroke with a Glasgow coma scale of 4 and a respiratory rate of 6 cycles per • Use of endotracheal tube
minute? • Required in the following conditions:
A. Low-flow face mask D o Altered sensorium
B. High-flow face mask o Failure to protect the airway
C. Non-invasive ventilation
D. Invasive ventilation Doc Visperas (2021): Intubation is required for patients with signs of respiratory
failure:
• Decrease in sensorium
• Paradoxical breathing
• Central cyanosis
Batch 2022 Pulmo Long Quiz 3 SBR (May 11, 2021)
80. One of the following drugs may be used for rate control of AF Acute Treatment: Rate Control
A. Digoxin • Control ventricular rate
B. – o Beta blockers (e.g. Metoprolol) – first-line
C. – o Calcium channel blocking agents: verapamil or diltiazem
D. Sotalol o Digoxin – uncommonly used as a stand-alone agent,
usually given in combination with BBs or CCBs
• Anticoagulation (if with indications)
o IV heparin – if >48 h
A
Acute Treatment: Rhythm Control
• Pharmacologic Cardioversion – antiarrhythmics
o Amiodarone – IV or oral (available in the PH)
o Flecainide – IV or oral (available in the PH)
o Propafenone – IV or ora
o Dofetilide – oral
o Ibutilide – IV

BMED 2023 TC: LICLICAN, C


• Electrical Direct Current Cardioversion
o Treatment of choice in Acute AF with severe symptoms and
hemodynamic instability
§ Unstable hypotension and/or heart failure; altered
sensorium
§ DC Cardioversion (200 J biphasic synchronized
shock)
• Establishes normal sinus rhythm in <
90% of patients
• Must sedate patient first (!!) - very
painful intervention

Long-term Management: Rate Control


• Beta blocker
• Calcium channel blocker: verapamil, diltiazem
• Digoxin
• Combination

Long-term Management: Rhythm Control – antiarrhythmics


• Amiodarone (Available in the PH)
• Flecainide (Available in the PH)
• Disopyramide
• Dofetilide
• Propanefone
• Sotalol
MED.CARDIO.1.08. Disorders of Rhythm, p.16
81. A 45 year old male patient is referred because of loss of consciousness. Vital If rhythm is either VF or Asystole:
signs of the patient showed BP 0, HR 0, RR 0. Chest compression is B → Don’t check for pulse anymore
immediately initiated. Upon arrival of the ECG, tracing showed: → DO CPR
→ Defibrillate VF
→ Ventricular Fibrillation or Pulseless VT
● Remember “SCREAM”|
- Shock → CPR → Rhythm check → Epinephrine → Antiarrhythmic
What is the next appropriate thing to do: Medication (AMiodarone)
A. – ● Reminders
B. Immediately get a defibrillator and defibrillate at 200j monophasic → Defibrillation
defibrillator ■ 200 J for biphasic defibrillator
C. – ■ 360 J for monophasic defibrillator
D. – → Always do chest compressions immediately after defibrillation
→ Lidocaine IV – alternative to amiodarone

*Despite 200j being used for biphasic, the answer key indicated it to be used
with monophasic
MED.CARDIO.1.08. Disorders of Rhythm, p.19
82. In the management of atrial fibrillation, which of the following is true? Atrial fibrillation increases risk of having stroke by 4- to 5-fold than patients
A. Anticoagulation for stroke prevention is associated with lower mortality A without AF. Strokes secondary to AF are more severe resulting to higher
among patients mortality. Thus, stroke prophylaxis with the use of anticoagulants decreases
B. – thrombus formation à decreased cardioembolic stroke à decrease mortality.
C. – MED.CARDIO.CCC3. Case with Right-Sided Weakness, p.10
D. –

BMED 2023 TC: LICLICAN, C


83. Which of the following is a typical characteristic of allergic asthma? Asthma Phenotypes
A. Responsive to antihistamines • Allergic asthma
B. Usually commences in adulthood o Most easily recognized
C. Associated with family history of atopy o Usually commences in childhood
D. Presents with predominantly monocytic airway inflammation o Associated with a past or family history of allergic
diseases such as eczema, allergic rhinitis, or food and
C drug allergy
o Examination of the induced sputum of these patients before
treatment usually reveal eosinophilic airway inflammation
o Patients with this phenotype usually responds well to
inhaled corticosteroid therapy
MED.PULMO.1.04. ASTHMA, p. 1
84. EG, A 75-year-old male patient with PAD who was lost to follow up now sees There is no definitive medical therapy for ciritical limb ischemia.
you for a second opinion. He was advised by another cardiologist to undergo Revascularization procedures, including catheter-based and surgical
arterial bypass because of a nonhealing dry ulcer in the left foot. He was told interventions, are usually indicated for patients with disabling, progressive, or
that if not managed properly that his condition may eventually lead to severe symptoms of intermittent claudication despite medical therapy in order to
amputation. The patient has an active lifestyle but is mostly non-compliant. He improve walking distance and functional capacity. These are also indicated in
wants to give maximal medical therapy another try. What principle of patients with critical limb ischemia to relieve pain and prevent limb loss.
management of PAD will you base your opinion on regarding the contemplated C
revascularization procedures for EG? Page 1925, Harrison’s Principle of Internal Medicine 20th Ed.
A. –
B. –
C. Disabling, progressive, or severe symptoms of intermittent claudication
despite maximal medical therapy is an indication for revascularization
D. –
85. In the GINA guidelines, which of the following is NOT included as an option STEP 1:
for asthma treatment? • Preferred controller and reliever: as-needed low dose ICS-Formoterol
A. ICS/LABA combination both as maintenance and reliever (D)
B. Low dose ICS maintenance and SABA as reliever • In order to prevent SABA overuse, which can lead to increased
C. Rescue SABA in the absence of ICS maintenance treatment mortality and morbidity without an adequate controller (ICS)
D. Rescue ICS/LABA without maintenance treatment STEP 2:
• Option 1: daily low-dose inhaled ICS (controller) + as-needed low
dose ICS-formoterol (reliever)
• Option 2: as-needed low dose ICS-formoterol (both controller and
reliever)
STEP 3:
C • Option 1: low-dose ICS-LABA (controller) + as-needed SABA
(reliever) (B)
• Option 2: low-dose ICS-formoterol for patients prescribed
maintenance and reliever therapy
STEP 4:
• Option 1: medium dose ICS-LABA (controller) + SABA (reliever)
• Option 2: low dose ICS-formoterol (controller and reliever) (A)
STEP 5:
• High dose ICS-LABA + SABA
• ICS-Formoterol
MED.PULMO.1.04. ASTHMA, p. 5-6.

BMED 2023 TC: LICLICAN, C


86. Which information leads one to suspect obstructive sleep apnea? Snoring
A. – D Tiredness
B. – - when waking up
C. – Obstruction
D. Neck circumference of 18 inches - noticed to stop breathing or choking episodes at night
Pressure
- high BP
BMI (>30)
Age (>50 y.o.)
Neck Circumference
- 17in for men
- 16in for women
Gender (Male>Female)
MED.PULMO.1.09.b.Sleep Disordered Breathing, p. 6
87. A patient has homogenous pacification of the right hemithorax. What physical (To be clarified by Dra Visperas, question seems wrong and might be which
examination finding should make us suspect that a patient has both atelectasis B PE finding should make us suspect patient has atelectasis and NOT pleural
and pleural effusion on the right? effusion)
A. Lagging on the right
B. Location of the apex beat
C. Egophony right above the level of dullness
D. Absence of breath sounds
88. For patients allergic to Benzathine PCN G, what is the alternative drug we Benzathine Penicillin G is the drug of choice for the secondary prevention of
could give to prevent recurrent acute rheumatic fever? C rheumatic fever.
A. Amoxicillin If patient doesn’t like Benzathine Penicillin G because it is injectable, you may
B. Penicillin V opt to give oral Penicillin V but it is less effective.
C. Erythromycin Erythromycin is given for patients who are allergic to penicillin.
D. Co-Amoxiclav
From Dra. Garcia’s discussion of answers
MED.CARDIO. CCC1.CASE-ON-DYSPNEA. p.12
89. A 68 year old male was admitted due to intermittent difficulty of breathing. A. CT pulmonary angiogram (or CTPA) – can be used but the patient has high
Patient had previous history of DVT. 12 L ECG showed sinus tachycardia and C creatinine, uses contrast medium; VQ scan is the better alternative
creatinine was elevated. After 45 minutes at the emergency room, patient B. MRI – no role in the diagnosis of PE
developed sudden drop in BP at palpatory 60. Which should be done to pursue C. VQ Scan – used if the patient has radiocontrast allergy or an elevated
the diagnosis before starting treatment? creatinine
A. CTPA D. Pulmonary angiography - uses contrast medium
B. MRI
C. VQ Scan From Dra. Visperas’ discussion of answers
D. Pulmonary angiography MED.PULMO.Supplement Q&A on CAP, VTE, Lung CA
90. Which of the following ECG findings may be suggestive of the presence of II and aVF are contiguous leads; a 2mm ST depression in those leads indicates
IHD? C inferior wall ischemia
A. QRS duration 100 milliseconds in V1-V6
B. Isoelectric ST segment in V5-V6, I, aVL MED.CARDIO.1.05.1.ABNORMAL ECG pp. 12-13
C. 2mm ST depression in II, aVF
D. T wave inversion in V1 and aVR
91. A 48-year-old male was referred for spirometry to evaluate his dyspnea. He is For obstructive ventilatory defect, flow parameters such as FEV/FVC are
a chronic smoker and claims to have asthma during his childhood. Which of D affected. For restrictive ventilatory defect, volume parameters such as TLC
the following spirometric parameters will be able to distinguish between (seen as FVC in spirometry) are affected.
asthma and COPD?
A. FEV1/FVC Among the choices, significant changes in pre- and post-bronchodilator FEV1
B. Post bronchodilator FEV1 or FVC are more evident in asthma as compared to COPD and hence, can
C. FVC differentiate between the two conditions.
D. Pre- and post-bronchodilator FEV1 or FVC
BMED 2023 TC: LICLICAN, C
To know if there is a significant response to bronchodilator, 2 criteria should be
met:
1. >=12% change from pre- to post-BD FEV1 or FVC
2. >= 200 ml increase from pre- to post-BD FEV1 or FVC
MED.PULMO.2.05a. SPIROMETRY (B2023), p. 1
MED.PULMO.2.05b. SPIROMETRY APPLICATION (B2023), p. 2
92. A 65y male patient, diagnosed case of ischemic heart disease consulted due Carvedilol - Beta blocker
to persistent easy fatiguability and palpitations. On PE: BP 150/90, HR Overall, beta blockers may be less protective against cardiovascular and
102bpm, RR 22cpm. JVP 4cm at 45°. AB at 6th LICS AAL, (+) LV heave, (+) cerebrovascular endpoints, and some beta blockers may have less effect on
grade 3/6 MR murmur, occasional bibasal crackles, grade 1 bipedal edema. central aortic pressure than other classes of antihypertensive agents. However,
Current medications Valsartan 160 mg BID, ASA 100 mg OD, and beta blockers remain appropriate therapy for hypertensive patients with
Rosuvastatin 20 mg/tab OD. What is the best medication to add to his present concomitant heart disease and related comorbidities.
medications to improve hard outcomes?
A. Ivabradine 5 mg BID Ivabradine – HCN channel blocker
B. Furosemide 40 mg TID Ivabradine should be considered in patients who remain symptomatic after
C. C.Carvedilol 25 mg BID guideline-based ACEIs, beta blockers, and mineralocorticoid receptor
D. D.Sacubitril Valsartan 100 mg BID antagonists and with residual heart rate >70 beats/min. Another group in whom
potential benefit may be expected includes those unable to tolerate beta
blockers.
C
Furosemide – Loop diuretic
Loop diuretics generally are reserved for hypertensive patients with reduced
glomerular filtration rates (reflected in serum creatinine >220 μmol/L [>2.5
mg/dL]), CHF, or sodium retention and edema for some other reason, such as
treatment with a potent vasodilator, e.g., minoxidil.

Valsartan – ARB
ACEI/ARB therapy has been associated with more adverse events (e.g.,
cardiovascular death, myocardial infarction, stroke, and hospitalization for heart
failure) without increases in benefit.

Harrison’s Principles of Internal Medicine, 20th edition (pg. 1774, 1901-1903)


93. A 66-year-old male presented with a lung mass on the right upper lobe which
was eventually diagnosed as lung adenocarcinoma. On physical examination,
there were enlarged supraclavicular nodes on both sides. There was lagging
on the right, decreased vocal and tactile fremiti, dullness on percussion and
decreased breath sounds on the right lower lung fields from mid to base. The
apex beat was at the 5th LICS, AAL. What is the presumptive stage of the lung
cancer? D
A. –
B. –
C. –
D. Stage 4

BMED 2023 TC: LICLICAN, C


94. Based on JNC 7, a BP of 140/90 taken last week, and a BP of 150/90 taken JNC 7 Guidelines for Classification of Hypertension in Adults Classification
today would label the patient as having
A. Prehypertension
B. Stage 1 HTN
C. Stage 2 HTN B
D. Stage 3 HTN

MED.CP.1.10.Systemic Arterial Hypertension (B2023)


95. What apnea hypopnea index result indicates moderate obstructive sleep • AHI (Apnea Hypopnea Index): Averaged frequency of apnea and
apnea? C hypopnea events per hour of sleep
A. – o AHI = (Apneas + Hypopneas)/ hours of sleep
B. – • Normal Cessation of Breathing during sleep: 5 times per hour
C. 20 per hour of sleep.
D. – o 5-15=Mild
o 16-30 = Moderate
o >30 = Severe
MED 2- PULMO 1.09 – Sleep disordered breathing p. 6
96. Which of the following is an indication for permanent pacing? There is no medical treatment for symptomatic complete heart block; sinus sick
A. Atril Fibrillation C syndrome and a heart block definitive treatment is permanent pacemaker.
B. Premature Ventricular Complex
C. Complete Heart Block MED 2- PULMO 2.09 – Disorders of Rhythm p. 6
D. Ventricular Tachycardia

BMED 2023 TC: LICLICAN, C


97. Which diagnostic finding fulfills the criteria for an acute myocardial infarction Choice A: Angina is rarely localized below the umbilicus or above the
in addition to an elevated troponin level? C mandible.
A. Pain localized over epigastrium and radiating to the umbilicus Choice B: Left bundle branch block = QRS duration > 120 ms
B. QRS duration 100 ms in limb and precordial leads Choice D: Presence oh hypokinetic left ventricular walls
C. New Q waves more than half the height of succeeding R in I & aVL
D. Presence of hyperkinetic left ventricular wall Significant/Pathologic/Abnormal Q Wave
• > 0.04 sec
• > 2 mm
• > 25% of R in 2 or more contiguous leads
MED 2-CARDIO 2.05 – Abnormal ECG p. 12 of 17
98. Which of the following antithrombotic drugs is optimal for stroke prophylaxis in NOACs have thus emerged as the standard of care treatment for the prevention
a nonvalvular AF patient with previous history of stroke? C of thromboembolism in patients with nonvalvular AF and vascular risk factors.
A. Aspirin
B. Clopidogrel
C. Rivaroxaban
D. Ticlopidine

MED 2-CARDIO 1.08 – DISORDERS OF RHYTHM p. 15 of 20


99. Which of the following angiographic findings would favor PCI over CABG as Percutaneous Coronary Prevention (PCI)
the primary mode of revascularization? B • PCI involves balloon dilatation usually accompanied by coronary
A. Highly calcified and tortuous arterial segments stenting is widely used to achieve revascularization of the myocardium
B. 90% stenosis of the proximal left anterior descending artery in patients with symptomatic IHD and suitable stenoses of
C. 80% of (L) anterior descending, (L) circumflex & ( R) coronary arteries epicardial coronary arteries.
D. 70% & 80% stenosis of proximal (L) circumflex artery & (L) main coronary • PCI is widely employed in patients with symptoms and evidence of
artery respectively ischemia due to stenoses of one or two vessels and even in selected
patients with three-vessel disease (and, perhaps, in some patients with
left main disease) and may offer many advantages over surgery.
• Patients with single- or two-vessel disease with normal LV
function and anatomically suitable lesions ordinarily are advised to
undergo PCI.

Coronary Artery Bypass Grafting (CABG)


• Patients with stenosis of the left main coronary artery and those with
three-vessel IHD (especially with diabetes and/or impaired LV
function) who require revascularization are best treated with CABG.
• Patients with three-vessel disease (or two-vessel disease that
includes the proximal left descending coronary artery) and
impaired global LV function (LV ejection fraction <50%) or
diabetes mellitus and those with left main CAD or other lesions
unsuitable for catheter-based procedures should be considered for
CABG as the initial method of revascularization.
Harrison’s Principles of Internal Medicine (20th Ed.), p. 1863-1864

BMED 2023 TC: LICLICAN, C


100. Among COPD phenotype B, C and D, which of the following intervention is Pulmonary Rehabilitation
proven to improve exercise endurance/tolerance among COPD patients? B - This refers to a comprehensive treatment program that
A. – incorporates exercise, education, and psychosocial and
B. Pulmonary Rehabilitation nutritional counseling. In COPD, pulmonary rehabilitation has
C. – been demonstrated to improve health-related quality of life,
D. – dyspnea, and exercise capacity. It has also been shown to
reduce rates of hospitalization over a 6- to 12-month period.
Harrison’s Principles of Internal Medicine (21st Ed.), p. 1916

BMED 2023 TC: LICLICAN, C


MED.CP.SBR1 Samplex Based Ratio – Quiz 1 40 of 40
MEDICINE 2 (CARDIO-PULMO MODULE)

First Shifting Examination


OCTOBER 15, 2020 | First Shift A.Y 2020-2021

QUESTION RATIONALE
A Notice the ST elevation on leads II, III, and aVF

Anteroseptal- V1 and V2
Anterior Wall- V1- V4
Inferior wall- II, III, aVF
Lateral wall- I, aVL, V5, V6

1. Based on this ECG, there is e

a. Inferior
b. Lateral
c. Anterior
d. Septum
2. One of the following drugs may be used for D Pharmacologic cardioversion uses anti-arrythmic
pharmacologic cardioversion of AF: medications to restore normal heart rhythm in Atrial
fibrillation
a. Verapamil
b. Digoxin Use Class 3 or Class IC
c. Metoprolol
d. Amiodarone

For rate Control and Rhythm Control


- Beta-blocker
- Verapamil
- Diltiazem
- Digoxin
- Also consider chronic rate control due to
increased chance of recurrence
- Digoxin (rhythm control)

3-4 A 23y COVID patient presented at the ER due to chest pain, dyspnea on exertion, orthopnea, paroxysmal nocturnal
dyspnea, bipedal edema. Physical examination showed BP 130/80 HR 110bpm, RR 23cpm, AB 5t LICS MCL, (+)S3, no
murmurs, (+) friction rub, (+) Bibasal crackles from T7down, grade 2 bipedal edema.

Alea, EP; De Guzman, EJ Page 1 of 26


MED 2- First Shifting Examination (A2022) Page 2 of 26

D Patient appears to be in acute CHF but may also overlap


3. Which of the following is the appropriate management with symptoms of pneumonia due to COVID-19
for the underlying cause of decompensation?
A- management for ACS-STEMI
a. Percutaneous coronary intervention for coronary B- no signs of arrythmia
atherosclerosis C- patient will not benefit from inotropes since his BP and
HR are elevated
b. Anti-arrhythmics to decrease heart rate

c. Inotropes to support cardiac pump function

d. Guideline directed antiviral agents

4. Which of the following is an appropriate initial D Choices A to C are the initial treatment modality for
management at the ER? patients with acute MI. (MONA)

a. Give morphine 2mg IV q 3-4 hrs Patient is in congestive state, therefore furosemide should
b. Give Aspirin 325 mg/tab and Clopidogrel 75 mg/tab, be administered.
4 tabs orally - Used in any phase of HF to control fluid overload
c. Start nitrogylcerine 10 mg/hr infusion and improve symptoms and signs of congestion.
d. Give furosemide 40 mg IV push - “By contrast, diuretic agents are extremely
effective, as they diminish pulmonary congestion
in the presence of systolic and/or diastolic heart
failure. LV filling pressure falls and orthopnea and
dyspnea improve after the intravenous
administration of furosemide or other loop
diuretics.” Harrison’s

5. A 24y female consults because of palpitations. On B sinus rhythm with premature ventricular complexes is
examination, BP 120/80 mm Hg, HR 80 bpm with usually benign
skipped beats, RR 16. ECG showed sinus rhythm with
premature ventricular complexes. What will be your Premature Ventricular Contraction (PVCs)
advice to the patient? • Prematurely occurring complex
• Wide, bizarre looking QRS complex
a. She must immediately stop her studies and undergo • Usually no preceding P wave
comprehensive workup and cardiovascular clearance
• T wave opposite in deflection to the QRS complex
prior to resuming her studies
• Complete compensatory pause following every
b. Her condition is benign but further tests are premature beat
recommended for a more complete cardiac evaluation • Usually benign

c. She must be treated immediately with antiarrhythmic


medications and an implantable defibrillator to relieve
her symptoms and prolong her survival

d. Her condition is associated with increased risk for


sudden death and she must undergo complete cardiac
workup

6. A 26y male student consults because of sudden D Verapamil- used for long term management to prevent
onset palpitations. Vital signs showed BP 120/80 mm recurrences
Hg, HR 180 bpm, RR 22. ECG showed the following:
Lidocaine is used for the acute mgt of sustained acute
Vtach or VFib

Transcutaneous pacing is used for the mgt of Bradycardia


with unstable BP
What is the next step in the management of this patient?
Supraventricular tachycardia
etation? B Supraventricular
A- AVRtachycardia
class IIa
● Re-entry around
B- Will only be classthe1 forAV node tissue
Asymptomatic, severeor
AS if
chycardia involvingwith the normal
LV failure and hadAVother
nodal conducting
cardiac surgery
rdia MED 2- First Shifting Examination (A2022) C- For
system andACRan class IIa
accessory pathway Page 3 of 26
coring system
on a. Givefor stroke 2.5
Verapamil riskmg in IV
non-valvular atrial A ● CHA2DS
• Re-entry
Tachycardia 2-VASc
around theScore
with narrow AV node tissue
QRS or involving
complex
n, one of b.
theDofollowing is true?pacing
transcutaneous the normal AV nodal conducting system and an
pertension is equivalent to infusion
c. Start Lidocaine 1 point
● Sudden onset and termination
extra accessory pathway
controlledd.diabetes
Give Adenosine
equates6tomg IV
3 points ● 150-250 beats/min
• Tachycardia with a narrow QRS complex
revious stroke or transient ischemic attack scores 4 ● Regular
• Sudden rhythm
onset and termination
nts ● QRS complex
• 150-250 is normal in contour and duration
beats/min
art failure is equivalent to 2 points ● No • discernable
Regular rhythmP waves (buried within QRS)
34 | PAGE
● FAST, NARROW, REGULAR, NO P WAVE
• QRS complex is normal in contour and duration
• No discernable P waves
• Fast, Narrow, Regular, No P wave.

Patient has pulse


Patient has stable BP à medication

fis
thethe first line
following is thepharmacologic
first line pharmacologic agent C for D
ntricular
mergency tachycardia
management
MED 2-Cardio-
if the BP is 120/80,
of Heart
QUIZ 1: Cardiac Diagnosis,
PR 180bpm,
thisFailure, Cardiomyopathy, ECG, RHD Page 4 of 5
pm? (13 AUGUST 2020)
nolol
iodarone severe MS with moderate MR
opine - Assumed that it is symptomatic
11. The drug of choice for patients with RHD allergic to Penicillin B Secondary Prevention of Rheumatic Fever (AHA):
enosine requiring secondary prophylaxis for rheumatic fever is
a. Azithromycin Mainstay of secondary prevention of ARF and RHD: long-term
b. Erythromycin Penicillin prophylaxis
c. Ciprofloxacin • Best: Benzathine Penicillin G (1.2million units, or 600,000
d. Ofloxacin units if </=27kg) q4weeks.
• Oral Penicillin V (250mg) BID as alternative but less
effective that Benzathine Pen G
• Erythromycin (250mg) BID for Peniciliin allergic patients
12. A 38, year old male consulted because of easy fatigue that C Patient was diagnosed with heart disease (3 y.o.) – points to a
f the following
started 8is months
true regarding
ago. In the thepastimaging
6 months,modality
he noted C Case based
Congenital anomaly ~ ASD
s used in the patient
gradually increasing discussed
size of his feetin that
the inclinical case
the last few weeks,
nce? his7. shoes
A 2D no Echocardiogram
longer fit. Laterwas on requested for a patient D
he noted increasing In ASD, there is a defect in the interatrial septum. Since pressure in
suspected
enlargement ofof both
having
nial X-ray showed temporal bone fracture legs.a congenital
This was heart disease.
accompanied by The the left is greater, blood tends to flow towards the right atria during
apical 4 chamber
enlargement of the abdomenview unequivocally
and loss of appetite.showed a dilated
A week contraction of the heart.
nial Doppler
prior showed intracranial artery stenosis
rightto atrium
consult he andalso noted right
dilated yellowish color of Which
ventricle. his eyes.of the
nial MRIPersistent
showed
following acute
symptoms
most cerebral
prompted infarction
common
him to seek consultation. She
adult congenital heart disease
Overtime, there will be volume overload to the right heart chambers
nial CT was
scan
is
diagnosed
therevealed
likely
to have
cause
“heart disease”
intracerebral
of these
when she was 3 years
hemorrhage
chamber enlargements?
(RVH), which can also manifest as tricuspid regurgitation (4/6
old holosystolic murmur heard louder during inspiration at the12lower
ofleft
28
ar old male consulted
Physical examination: at theNotemergency room distress,
in cardio-respiratory due to BP B Most appropriate
parasternal area) andinitial anti-hypertensive
a pulmonic flow murmur (4/6 medication
mid-systolic in HPN
onset (R) sided
PR 100weakness.
A. Ventricular
110/70 septal
/min, defect
regular He isRRa16/min
rhythm, known T 37˚C emergency should have
crescendo-decrescendo murmuraatrapid onset of action. So it
the base). must
nsive with BMI poor
b. 20. blood
Icteric
Patent sclerae,
pressure
Ductus JVPcontrol.
Arteriosus at the angle
At theof the
ER,mandible..
his be in IV form
Hyperdynamic precordium. Apex beat at 5th LICS anterior Eventually patient’s condition will lead to congestive heart failure
essure was 240/120 mmHg,
c. Coarctation of the withAortaMMT of 3/5 on both
axillary line. + strong substernal impulse and a PA lift. A (elevated JVP, bipedal edema, dyspnea, passive congestion of the
er and lower d. Atrial
systolic thrillseptal
is felt atdefect
extremities with
the left5/5 on the and
parasternal (L) pulmonary
upper and areas. BP- 240/120 mmHg
liver=hepatomegaly). Pulmonary hypertension can also develop
tremities. Which
At the apex, of theS2
S1 and following
are both loud.wouldS1 isbe the most
followed by a grade Patient hadasadilated
and manifest sudden onset
PA (PA of R
lift), RV sided
heave, andweakness
accentuated P2.
ate initial4/6antihypertensive
holosystolic murmur you heardcanlouder
giveduring
thisinspiration
patient?at the Due to elevated pressures, there will be dilatation of the
lower left parasternal area. At left the base, S1 is soft followed
nidine sublingual
by a grade 4/6 mid-systolic crescendo-decrescendo murmur. right heart chambers.
S2 is widely split and relatively fixed in relation to respiration.
8. A sounds
Breath 38 yearareoldnormal.
male was diagnosed
The liver to cm
span is 20 have an ASD
at the C Indications for surgical intervention:
LMCL. There is of
with QP:QS bilateral
1.6:1.pitting
The edema up to the
most ideal level of the is
management
knees.
The main problem of this patient is:
A.a. Close
Mitral monitoring of complications
Stenosis
B.b. WatchfulSeptal
Ventricular waiting for it to close
Defect
C.c.Atrial
Percutaneous
Septal Defector surgical closure of the defect
D.d.Mitral Valve Prolapse
Diuretics with Mitral
if he develops Regurgitation
pedal edema
13. As to etiologic diagnosis, this is classified as B Please refer to no. 12
a. Degenerative
b. Congenital
c. Atherosclerotic
d. Rheumatic
14. The increased flow across the pulmonary valve was heard as D Please refer to no. 12
a:
a. Systolic blowing murmur after S1 at the apex
b. Systolic crescendo decrescendo murmur after S1 at the
apex
c. Systolic blowing murmur after S1 at the base
d. Systolic crescendo decrescendo murmur after S1 at the
carries a high risk of recurrent thromboembolic stroke
needs anticoagulation
c. Based on the CHADSVASC score of the patient, he
carries a high risk of recurrent thromboembolic stroke
MED 2- First Shifting Examination (A2022) Page 4 of 26
and does not need anticoagulation
d. Based on the CHADSVASC score of the patient, he
carries a low risk of recurrent thromboembolic stroke and
9. does
48y notfemale presents to the emergency room with D
need anticoagulation
2. palpitations.
What Vitalsprocedure
is the diagnostic signs showed BP 120/80,
that will distinguish HR 170 C
between A- GOLD STANDARD in identifying presence or absence of
bpm,MIRR
acute and24. ECG showed
dissecting aneurysmthe following:
at the emergency room? arterial narrowing related to atherosclerotic CAD
a. Coronary angiography
b. Transesophageal echocardiography B- Choice For:
c. 12 lead ECG • valvular vegetations (eg infective endocarditis)
d. MRI prosthetic valve disease
• intracardiac thrombi
What is the ECG interpretation? • assessment of congenital abnormalities (eg RHD)

D- Viability assessment post MI; also:


a. Atrial fibrillation
b. Ventricular fibrillation Clue:• atrial
evaluation of patients with
tachyarrhythmias are known or ventricular
narrow, suspected are
CAD
c. Supraventricular tachycardia wider
• assess regional myocardial perfusion
d. Ventricular tachycardia • assess wall motion at rest & during stress
10. A 50y male was brought unconscious into the B • myocardial & pericardial diseases
3. Aemergency
45y male department.
patient is referred because of loss of D Ventricular Fibrillation
consciousness. Vital signs of the patient showed BP 0, HR 0, • Can come from V-Tach
RR 0. Chest compression is immediately initiated. Upon • Associated with coarse or fine chaotic undulations
On hooking to a cardiac monitor, the following tracing
arrival of the ECG, tracing showed: of the ECG baseline
was taken: • No P wave
• No true QRS complexes
• Indeterminate rate
• Patient is in cardiac arrest
• EARLY DEFIBRILLATION IS KEY!
• If in arrest. Do CPR immediately
Above
What is the ECG tracing is compatible with:
interpretation?
a. Ventricular tachycardia Based on ACLS guidelines:
b.a. Atrial
Atrial fibrillation
fibrillation • CPR (Airway, oxygen, connect monitors)
c. Atrial flutter •
b. ventricular fibrillation 120-200 joules if biphasic defibrillator
d. Ventricular fibrillation
c. Atrial tachycardia • 360 joules if monophasic defibrillator
d. ventricular tachycardia • Continue CPR for 2 min.
• Epinephrine 1mg every 3-5 min.
• Amiodarone OR Lidocaine
• If spontaneous circulation is still absent, go back to
defibrillation
• If spontaneous circulation present, go to post
cardiac arrest case

4.
11 A 55y female complains of angina when she climbs
True about the management of atrial fibrillation A
B Class I: No symptoms of heart failure.
B- Treat the etiology of the arrythmia before giving anti-
a.4 flights
Chronicofanticoagulation
stairs rapidly.is What is her New
recommended York Heart
for patients arrhythmic drugs
Association
with risk forFunctional
stroke Classification? Class II: Symptoms of heart failure with moderate
exertion, such as ambulating two blocks or four flights of
a. DE
ALEA, EP; I GUZMAN, EJ stairs. Page 1 of 7
b. II
c. III Class III: Symptoms of heart failure with minimal exertion,
d. IV such as ambulating one block or one flight of stairs, but
no symptoms at rest.

Class IV: Symptoms of heart failure at rest.

12-17 A 60y known hypertensive for 15 years but poorly compliant with medications presents with sudden onset of severe
constricting chest pain grade 10/10 accompanied by diaphoresis. Vital signs showed BP of 180/120, CR 110bpm regular,
RR 22cpm, O2 sat 94% at room air. Except for numerous tophi noted on his hands, the rest of your PE is unremarkable.

12 How would you classify the hypertension of the A Patient appears toxic and is at high risk of having end-
patient? organ damage (very high BP, diaphoresis, pain of 10/10,
and is in distress)
a. Hypertensive emergency
b. Hypertensive urgency He is 15 years hypertensive with poor complianceà
c. Stage 2 HTN suspect cardiac changes (such as LVH)
d. Stage 1 HTN
c. Coagulopathy or a build-up of plaque on the inside of the arteries
d. Atherothrombosis
Thrombosis- formation of clot (thrombus) within the blood
vessels
MED 2- First Shifting Examination (A2022) Page 5 of 26
7.
Auscultatory findings in patients experiencing A This position accentuates the heart sounds related to the
angina
13 What is the bestare best diagnostic
initial appreciated in this position
procedure that you A left ventricular
12L ECG will rulefunction
out ACS of the
as heart,
well aswhich
themay be affected
extent of cardiac
a. Left lateral decubitus in ischemic heart disease
would request in the ER? electrical and structural involvement
b. Supine and during exhalation
c. Sitting and leaning forward Remember: Earliest sign of target organ damage in ECG
a. 12L ECG d. Seated at bedside with feet dangling is LVH
b. Troponin I
8. Cardiac troponin levels in acute MI peak at
c. 2D Echo-Doppler B Troponin
a. PA-Lat
d. Chest X-ray 36 hours • Diagnostic procedure with highest specificity in
b. 24 hours diagnosing Acute MI (>20 times higher than the
c. 6 hours upper reference limit)
d. 12 hours • can be detected 6-12 hours after onset
of myocardial injury
• peaks at about 24 hours
• Levels may remain elevated for 7-10 days after
STEMI

9. Which is an indication for Coronary Artery Bypass A Troponin determination


Indications of CABG: is not ideal in an acute setting
Graft (CABG) Surgery? 2D Echo-• willStenosis
not detect
of theelectrical changes
left main coronary artery
a. Left main artery stenosis CXR- will• only assess
Three cardiac
vessel IHD and pulmonary
(especially with diabetes structural
and/or
b. Left circumflex artery stenosis changes impaired LV function)
c. Left anterior descending artery stenosis
d. Right coronary artery stenosis
14 A 2D Echo- Doppler was requested. Considering B As the name implies, this will only determine the outflow
your PE findings, which view would be LEAST helpful tracts and not the heart’s structural integrity per se.
for you? therefore, this is among the least helpful for the
assessment of cardiac damage in ACS

a. Apical 4 chamber view


b. Great vessel view
c. Parasternal long axis view
d. Parasternal short axis view

15 What would be the best initial antihypertensive A In acute cases of coronary syndromes, MONA!
medication for this patient?
IV is the preferred route of administration
a. IV nitroglycerine IV nitroglycerin
b. IV furosemide - Preferred parenteral drug in patients suspected to
c. Sublingual clonidine have left ventricular failure (15 years
d. Sublingual captopril hypertensive, poor-compliance)

Furosemide- used in congestive states


Clonidine- usually used for Stage II HPN; poor choice if
mental status monitoring is important
Captopril- ACEi; not used in emergency situations

16 Further work-ups revealed significant Q waves in III D Q waves in III and aVF- old infarct/hypertrophy at inferior
and aVF with 3 mm ST depression in V1-V4. After ventricular wall
stabilizing the patient, what maintenance ST depression in V1-V4- anteroseptal submyocardial
antihypertensive medication would you give our infarct
patient?

a. Perindopril + Olmesartan Ideal maintenance medication for patients with


b. Candesartan + Hydroclorothiazide complicated HTN with LVH
c. Telmisartan + Amlodipine - Thiazide
d. Trandolapril + Verapamil - ACEi/ARB
MED 2- First Shifting Examination (A2022) Page 6 of 26

- Long acting CCB

Thiazide should also be avoided since the patient has


tophi (it will cause hyperuricemia)

Addition of a Non DHP CCB will target stabilize the


patient’s HR, thus lowering the workload and cardiac
demand of the suspected hypertrophied/ infarcted
structure.
MEDICINE 2 (CARDIO MODULE)

LONG QUIZ 1
17 Which of the following would most likely exacerbate B HCTZ causes hyperGlUC (Glycemia, Uricemia, and
his gouty arthritis? Calcemia)

a. Trandolapril
(Cardiac Diagnosis, Heart Failure, Cardiomyopathy, ECG, RHD)
b. Hydroclorothiazide
August 20, 2020 | First Shift A.Y 2020-2021
c. Verapamil
d. Candesartan
QUESTION ANSWER RATIONALE
have a 67 year old female patient who came in due to C Risk factors for atherosclerosis:
st pain. She 18used
A 65y to patient,
be able to walk up
known to three
diabetic, blocks but with a A
hypertensive • 1⁄2 of all deaths worldwide
ed chest pain and dyspnea after walking less than
previous STEMI presented at the ER with exertional a block • Classic anginal syndrome in the absence of other known
ce three weeks prior to admission. The pain was described cause
heaviness,dyspnea,
grade 8/10, orthopnea,
radiating to bipedal
the leftedema, distended neck
jaw, aggravated • MI in the absence of other known causes
veins and
effort and relieved bybibasal
rest. Shecrackles. Echocardiogram
is hypertensive, diabeticshowed a • Age (M > 50; F >60)
d dylipidemic. reducedShe systolic
is a function
former and segmental
smoker and wall
has motion • Hypertension- mechanical damage
abnormality.
asional alcohol intake. Which
At the heart failure
clinic, stage does
BP 160/90 the patient
regular, • Diabetes- endothelial dysfunction
98, RR 20, afebrile.
belong to? AB 6th LICS AAL, Normal S1/2 no • Dyslipidemia- fat deposition
4, (+) grade 4/6 murmur at the apex. Clear breath sound, • Smoking/ alcohol- ROS production
rest of the PE was unremarkable.
a. Stage C
at is the possible underlying cause of the patient’s cardiac
ease? b. Stage A
Congenital c. Stage B
heart disease
d. Stage
Valvular heart D
disease
Atherosclerosis
Inflammatory
recommended diagnostic modality to evaluate valvular C Cardiac MRI
ction, right ventricular function and estimate pulmonary • evaluation of patients with known or suspected CAD
ssures in patients
Format: withQuiz
[MED 2-Cardio] heart failure
3/CCC is disorders, Hypertensive heart disease, Aortic dissection,•Aorticassess
3 - Rhythm regional
stenosis/ myocardial2020)
(03 SEPTEMBER perfusion Page 7 of 7
Cardiac MRI • assess wall motion at rest & during stress- myocardial & pericardial
Nuclear Perfusion Scan • Caused by obstruction in the outflow tract
diseases
Transthoracic echocardiogram • Seen in severe AS and hypertension
Right26.heart
Which of the following is the best stroke prophylaxis for our
catheterization A *Case
Nuclear based
Perfusion Scan
patient in the CCC? • evaluate regional myocardial perfusion under rest and stress condition
a.19 NOAC
A 70y patient with atrial fibrillation with a previous A
b.history
Clopidogrel
of stroke must receive which stroke prophylaxis Transthoracic echocardiogram
c. Aspirin • RV cannot be assessed through 2D-Echo due to its anterior location
agent?
d. Warfarin (covered by the sternum)
• Valvular functions and vegetations (eg infective endocarditis)
a. Dabigatran • prosthetic valve disease
b. Aspirin • intracardiac thrombi
c. Cilostazol • assessment of congenital abnormalities (eg RHD)
d. Clopidogrel
Right Heart Catherization
• evaluate extent and severity of cardiac disease in symptomatic patient
• exclude severe disease in symptomatic patients with equivocal findings in
noninvasive studies
• for patients with chest pain syndrome of unclear etiology for whom definitive
diagnosis is necessary for management

89 27. A 20y
year oldfemale suddenly
patient withpresents
prior with a blood
history of pressure
ischemicof DC NOAC!!
Fibromuscular dysplasia
Patient already presents(FMD)
the clinical picture of heart failure, fulfilling
diomyopathy 20.came
200/100.A patient
No in
otherwith
due toobstructive
co-morbidities
dyspnea, lung
in the disease
past. and chronic A
You cough,
productive should •
the FraminghamMay present clinically
criteria. with hypertension
Therefore we request in younger
for ANP levels
screen her
andfordecrease
compensated
rmittent fever, secondary
heartinHTN due came
failure
appetite.to: OninPE, duepatient
to exertional individuals
Brain Natriuretic Peptide (BNP)(between age 15 and 50), most often
a. Pheochromocytoma • stimulus: without
women volume expansion
otherofco-morbidities.
ventricles (stretch)
s anasarcousdyspnea. He has discontinued hiswith
and cannot lay down supine, a BP of for the
medications • source: brain & ventricle
/70 HR 88b. RR Primary aldosteronism
28 cpm T 37.9 degrees examination,
Celsius. AB 6th •• One of the usual cause
bind to NP receptor → guanylate cyclase of →Large-vessel
cGMP renal
c.past
S AAL, Normal
three
Renovasculardays.
S1/2, (+)S3HTN Onduephysical
(+) to fibromuscular
grade 4/6 murmur dysplasiaBP110/60,
at the • artery occlusive
effect: diuresis, disease
natriuresis, or infarction
myocardial Renal& artery stenosis
anti-remodeling ○ uses of BNP

x. d.HR 108
Renal regular,
parenchyma RR 26
diseasecpm, O2 saturation 94%. Apex (which → is HF diagnosis, screening, predictor, prognosis
a secondary cause of hypertension)
→ HF guide to thera
beat at 7th
at is your next best step? LICS AAL, sustained and diffuse. (+) grade Further readings:→Harrison’s ACS prognosisp. 1906
28. A 3/6
Request for systolic
50y cardiac
male known murmur
MRI on the
hypertensive
with gadolinium was5-6th
noted LICS
to havetoPSL
contrast rulewith (+) D Myocarditis
• sustained and diffuse apex beat may point us to
sustained and diffuse apex beat. The rest of the
out myocarditis • LVH Presents with signs of inflammation with a history of prior/
Request cardiovascular PE is unremarkable.toWhich
for stress echocardiogram rule CXR would you
in coronary • PA on-going
view is the infection
best method among the choices in
want to request to verify your PE findings?
artery disease • assessing
Can cause fordilated cardiomyopathy
cardiomegaly and/or chamber
Request a. forChest
serumX-ray (R) lateraltodecubitus
creatinine determine viewdeterioration • enlargements.
Doesn’t necessarily warrant cardiac MRI for diagnosis;
LONG QUIZ 3 + CCC 3
(Rhythm Disorders, Hypertensive Heart Disease, Aortic Dissection, Aortic Stenosis)
MED 2- First Shifting Examination (A2022) Page 7 of 26
September 3, 2020 | First Shift A.Y 2020-2021
Carvallo’s signs, minimal crackles on both lung bases.
QUESTION ANSWE RATIONALE
At the emergency room, which diagnostic exam will you R
request to
1. rapidly rule in cardiac
A 78y hypertensive causemale
non-diabetic of the
with dyspnea?
atrial fibrillation B CHADSVASC score 1- favors anticoagulation
consults because of high blood pressure. He had a previous CHADSVASC score 1- requires anticoagulation
history of stroke 2 years ago. One of the following is true • NOAC/ Warfarin to an INR of 2-3
a. Natriureticregarding
peptides the above patient’s condition and therapeutic
b. Completemanagement:
blood count A-Cardioversion is used if patient is unstable
c. Chest X-raya. Based on the CHADSVASC score of the patient, he
carries a high risk of recurrent thromboembolic stroke
d. Echocardiogram and must be cardioverted D- patient is high-risk with score of 5
b. Based on the CHADSVASC score of the patient, he
21. About 75% carries
of thea total
high risk of recurrent
coronary thromboembolic
resistance stroke
to flow A
needs anticoagulation
occurs across three sets of arteries, EXCEPT:
c. Based on the CHADSVASC score of the patient, he
carries a high risk of recurrent thromboembolic stroke
a. left coronary and does not need anticoagulation
artery
d. Based on the CHADSVASC score of the patient, he
b. pre-arteriolarcarries
vesselsa low risk of recurrent thromboembolic stroke and
c. large epicardialdoesarteries
not need anticoagulation
2. What
d. arteriolar and isintramyocardial
the diagnostic procedure that willvessels
capillary distinguish between C A- GOLD STANDARD in identifying presence or absence of
acute MI and dissecting aneurysm at the emergency room? arterial narrowing related to atherosclerotic CAD
a. Coronary angiography
22. Among patients with acute
b. Transesophageal ST elevation myocardial
echocardiography C B- Choice For:
c. 12 lead
infarction, which of ECGthe following conditions if it • valvular vegetations (eg infective endocarditis)
d. MRI prosthetic valve disease
develops, would you favor PCI over thrombolysis? • intracardiac thrombi
• assessment of congenital abnormalities (eg RHD)
a. PVC’s in bigeminy
D- Viability assessment post MI; also:
b. BP of 160/90 • evaluation of patients with known or suspected
c. Cardiogenic Shock CAD
d. Relief of chest pain with morphine • assess regional myocardial perfusion
• assess wall motion at rest & during stress
• myocardial & pericardial diseases
3. Apatient
23. An 80y 45y malewaspatient
brought is referred because ofin loss
in unconscious the of A D Ventricular Fibrillation
consciousness. Vital signs of the patient showed BP 0, HR 0, • Can come from V-Tach
ER. Emergency measures were instituted and patient
RR 0. Chest compression is immediately initiated. Upon • Associated with coarse or fine chaotic undulations
was hookedarrival
to aof the
cardiac monitor
ECG, tracing which showed the
showed: of the ECG baseline
following tracing: • No P wave
• No true QRS complexes
• Indeterminate rate
• Patient is in cardiac arrest
• EARLY DEFIBRILLATION IS KEY!
• If in arrest. Do CPR immediately
Which isWhat
the iscorrect
the ECGstatement?
interpretation?
a. Ventricular tachycardia Based on ACLS guidelines:
b. Atrial fibrillation • CPR (Airway, oxygen, connect monitors)
a. ECG shows Ventricular
c. Atrial flutter fibrillation and defibrillation • 120-200 joules if biphasic defibrillator
must be done
d. Ventricular fibrillation
• 360 joules if monophasic defibrillator
• Continue CPR for 2 min.
b. ECG shows Atrial fibrillation and cardioversion must • Epinephrine 1mg every 3-5 min.
• Amiodarone OR Lidocaine
be done • If spontaneous circulation is still absent, go back to
defibrillation
c. ECG shows Ventricular tachycardia and amiodarone • If spontaneous circulation present, go to post
cardiac arrest case
must be infused
4. True about the management of atrial fibrillation A B- Treat the etiology of the arrythmia before giving anti-
d. ECG shows a. Atrial
Chronictachycardia and
anticoagulation is beta blockers
recommended formust
patients arrhythmic drugs
be given with risk for stroke

24. An 82y male hypertensive and diabetic consults B


ALEA, EP; DE GUZMAN, EJ Page 1 of 7
because of palpitations. He has no history of heart
failure, stroke nor coronary artery disease. Vital signs:
BP 140/100 mm Hg, HR 124 bpm irregularly irregular,
RR 19. ECG tracing showed the following:

Which of the following statements on the assessment


and management of this patient is true?
Hypertension (1)
Diabetes (1)
MED 2- First Shifting Examination (A2022) Page 8 of 26

a. The CHADSVASC score is 1 and Anticoagulation Age (2)


must be started for stroke prophylaxis
0- no therapy or aspirin
b. The CHADSVASC score is 4 and Anticoagulation 1- aspirin or oral anticoagulation
must be started for stroke prophylaxis >/= 2- oral anticoagulation

c. The CHADSVASC score is 2 and Anticoagulation


must be started for stroke prophylaxis

d. The CHADSVASC score is 3 and Anticoagulation


must be started for stroke prophylaxis

25. Case: A 19y/o female was referred for evaluation C A- seen in VSD, Mitral insufficiency,
prior to enrolment at a military academy. She has a B- non-specific sign of right and left ventricular
history of recent easy fatigability and palpitations. hypertrophy
D- seen in PDA
Which of the following is most consistent with a
diagnosis of rheumatic heart disease?

a. Systolic murmur at the left lower parasternum


b. Apex beat displaced to the left and downwards
c. Diastolic murmur at the apex
d. Continuous machinery-like murmur

26. Fibrinolysis is absolutely contraindicated in which of B Absolute Contraindications:


the following? ▪ Any prior intracranial hemorrhage
▪ Non hemorrhagic stroke or any cerebrovascular event
a. Advanced age within the past year
b. Cerebral hemorrhage ▪ Marked Hypertension (SBP >180 or DBP <110mmHG)
c. Current use of anticoagulants ▪ Suspected aortic dissection
d. Menstruation ▪ Active internal bleeding (excluding menses)

27. In STEMI patients to be given Streptokinase, the D


medicine should be given within _____ from the first
medical contact

a. 60 minutes
b. 90 minutes
c. 120 minutes
d. 30 minutes

28. In patients with dissecting aortic aneurysm, surgical B Stanford classification:


management is indicated if the involvement of the aorta
is at the

a. Thoracic
b. Ascending
c. Suprarenal
d. Infrarenal
MED 2- First Shifting Examination (A2022) Page 9 of 26

29. It is the year 2019 (pre-CoVid era). An unidentified B Dapat ABC but since cardio module to and covid pa, wag
male was rushed to the emergency room because of muna icheck yung breathing L
unresponsiveness. On initial assessment, he was assume na rhythm disorder yung case. So check pulse
unconscious and could not be aroused. What is the next muna!
thing to do?

a. Check pulse oximetry


b. Check arterial pulse
c. Check for breathing
d. Auscultate for heart sounds

30. Main pathophysiology of Stable Angina Pectoris B A- NSTEMI


C- STEMI
a. Stable plaque occluding the coronary vessels D- prinzmetal
b. Transient Myocardial Ischemia
c. Occlusion of a thrombus
d. Spasm of coronary arteries

31 NG, 75-year old hypertensive male presents with D Intermittent claudication is a peripheral disease. To
intermittent claudication of the left leg. evaluate the blood flow to these peripheral vascular
structures, doppler device can be used.
Which of the the following is a routine diagnostic test
that can be requested to work up the intermittent A,b,c- diagnostic test focusing on cardiac structures
claudication of NG?

a. 12 lead Electrocardiography
b. Magnetic Resonance Angiography
c. Computerized Tomographic Angiography
d. Doppler device to document blood flow in the dorsalis
pedis artery
32 Pharmacologic agent that will be of most benefit for C Cilostazol- a PDE3 inhibitor which is the drug of choice for
NG to improve his ability to walk before claudication is intermittent claudication
felt.
Metoprolol/ BBs- worsens intermittent claudication
a. Ramipril
b. Amlodipine ACEi- improves patient’s qualtity of life among
c. Cilostazol hypertensives, but doesn’t affect his capacity to walk
d. Metoprolol without discomfort

33 Non-transmural and intramyocardial ischemia C a- submyocardial infarction


presents with what ECG changes? b- transmural infarct
d- previous infarction/ submyocardial infarct
MED 2- First Shifting Examination (A2022) Page 10 of 26

a. Transient ST Segment depression


b. ST elevation
c. Transient T wave inversion
d. ST segment depression
34-38 RB, a 65y male, smoker consulted because of occasional chest pain accompanied by light headedness or
presyncope upon exertion. Auscultation revealed a grade 3/6 systolic crescendo-decrescendo murmur best heard over
the Erb’s point radiating to the suprasternal notch.

34 What is your diagnosis? B


Aortic Stenosis
a. Atrial septal defect • (Mid) systolic murmur that begin at a short interval after
b. Aortic stenosis S1 and ends before S2
c. Mitral regurgitation • Crescendo-decrescendo pattern
d. Tricuspid regurgitation • Low-pitched, rough and rasping in character
• Loudest at the (R) PSL 2nd ICS (base of the heart)
• Transmitted upward along the carotid arteries
• May also be transmitted to the apex (may be
confused with MR)
35 Which of the following diagnostic procedures may B Treadmill test will increase the cardiac workload of the
produce syncope when performed on RB? patient which may predispose the patient to have syncope

a. 24-hour Holter monitoring


b. Treadmill test
c. Coronary angiography
d. Echocardiogram
36 When would you recommend surgery for our patient? D Indications for surgery (accd to Doc)
If valve area is <1 cm2 with an EF >50%
a. If valve area is >1 cm2 with an EF >50% Patients for CABG with symptomatic aortic stenosis
b. Patients for CABG with asymptomatic mild aortic If valve area is <1 cm2 even if patient remain
stenosis asymptomatic
c. If valve area is >1 cm2 even if patient remain Bicuspid aortic valve with aneurysmally dilated aortic root
asymptomatic
d. Bicuspid aortic valve with aneurysmally dilated aortic
root

37 What ancillary procedure will confirm the diagnosis B Will provide the integrity of aortic structures with higher
in the shortest possible time? resolution and high turn-around time (including the
valves) – doc’s ratio
a. Coronary angiography
b. Computerized Tomographic Aortogram A and D- best for small cardiac vessel evaluation
c. Echocardiogram with Doppler C- for mitral and tricuspid evaluation
d. Magnetic Resonance Angiography
38 The pharmacologic agent that should be given to RD D IV medications should be administered
immediately
The aortic stenosis may be due to the aortic dissection
a. Nitroglycerine IV given the patient’s risk factor, therefore labetalol, which is
b. Perindopril oral the DOC, should be given
c. Amlodipine oral
d. Labetalol IV NTG- given in ACS
39 Right Ventricular Hypertrophy in a patient with D RVH can be seen in lateral upright view as retrosternal
severe mitral stenosis is best appreciated in what CXR fullness
view?

a. Oblique view
b. Anteroposterior view
c. Postero-anterior view
d. Lateral upright view

40 T.I., a 58 year old male, amputee, hypertensive for 5 A Amputee! Therefore, dobutamine test should be
years experienced intermittent chest heaviness requested
MED 2- First Shifting Examination (A2022) Page 11 of 26

aggravated by exertion. His cardiologist suspects that


he has coronary artery disease but his ECG is
inconclusive. What are you going to request to verify
presence of ischemic heart disease?

a. Dobutamine stress echo


b. Treadmill stress echo
c. Treadmill stress test
d. Coronary angiography
41 The characteristic, diagnostic flow murmur of ASD is A See #7
due to:

a. Increased flow across the pulmonic valve


b. Increase flow across the tricuspid valve
c. Back flow of blood into the right atrium
d. Increased flow across the Interatrial septal defect

42 The following are major risk factors for A


atherosclerotic disease, EXCEPT:

a. High HDL
b. Smoking
c. High LDL
d. Diabetes Mellitus

High HDL is favorable!


43 The primary sites of arterial involvement in 80-90% A Femoral and popliteal artery are the most common site of
of patients with PAD. involvement in PAD
a. Femoral and popliteal
b. Iliac
c. Abdominal aorta
d. Tibial and peroneal
44. What is the best drug for thromboembolic C In patients with atrial fibrillation and/or valvular heart
prophylaxis in an RHD patient with severe mitral disease, anticoagulation prophylaxis with warfarin is
stenosis with an irregularly irregular rhythm? recommended.

a. Dabigatran
b. Clopidogrel
c. Warfarin
d. Aspirin
MED 2- First Shifting Examination (A2022) Page 12 of 26

What is the preferred route of administration for Pen V A Note: Penicillin V or Phenoxymethylpenicillin is only
when used as prophylaxis for rheumatic fever? available in oral form. Pen G is the one given IM

a. Oral
b. Subcutaneous
c. Deep IM
d. IV bolus
Which echocardiographic finding is consistent with A B- aortic aneurysm
mitral stenosis? C- mitral regurgitation
D- non-specific
a. Diastolic doming motion of the mitral valves
b. Dilated aortic root
c. Color mosaic flow across the mitral valves on systole
d. Dilated left ventricle
Which historical information points to myocarditis as D A- drug induced cardiomyopathy
cause of a patient’s heart failure symptoms? B- residual shunt will not cause myocarditis
C- drug induced cardiomyopathy
a. Previous administration of anthracycline D- signifies infection à myocarditis
b. History of repaired atrial septal defect with residual
shunt
c. History of methamphetamine use for 6 years
d. Fever and cough one week prior to heart failure
symptoms
MED 2- First Shifting Examination (A2022) Page 13 of 26

Which is NOT included in the management of patients C


with Hypertrophic cardiomyopathy?

a. Improvement of outflow tract obstruction


b. Determining risk for sudden cardiac death
c. Anticoagulation to prevent stroke
d. Screening of family members

Which is a contraindication to exercise stress testing? D

a. Rest angina within 72 hours


b. History of infective endocarditis in childhood
c. Mild pulmonary hypertension
d. Severe aortic stenosis

Severe AS- may cause severe reduction in the systemic


perfusion
50 While you were on duty in the ICU, an 80y male was A
referred to you with the following ECG tracing on the
cardiac monitor. Her pulse rate is 35 bpm.

Which statement is correct?

a. ECG shows 3rd degree AV block and she must be


referred for possible pacing

b. ECG shows 1st degree AV block and this is a benign


rhythm Not a benign rhythm! Refer immediately!

c. ECG shows Asystole and chest compression must be


initiated

d. ECG shows 2nd degree AV block and this may be


either benign or dangerous

51. You examined the CXR of a 55 year-old male patient D Hyperinflation is usually seen in COPD.
with high suspicion of COPD. Which CXR finding is This will produce a CXR finding of a tubular heart.
compatible?
A. Unilateral, massive effusion
B. Diaphragm at the level of T8
C. Narrowed intercostal spaces
D. Tubular heart
MED 2- First Shifting Examination (A2022) Page 14 of 26

52. Which of the following findings is expected if a C Persistent fever on the 4th day of antibiotic therapy
patient admitted for CAP is still febrile while on day signifies increased severity of pneumonia à cavitations
4 of antibiotics? and multilobar involvement

A. Presence of similar infiltrates as seen on CXR on


admission
B. Blood culture growth of Streptococcus
pneumoniae, (in)sensitive to current antibiotics
C. Cavitations with air-fluid level on CXR
D. Sputum Gene Xpert is negative for TB

53. Which of the following will likely be assessed as C COPD - Typical picture of patient is a smoker, male, old,
COPD? with progressive symptoms.

A. 45 years old, male, 5 pack-year smoking history, A- More likely asthma. most asthmatics do not have
intermittent dyspnea daily symptoms, but experience intermittent
B. 45 years old, male, 30 pack-year smoking episodes of dyspnea, cough, and chest tightness
history, acute dyspnea that are usually associated with specific triggers,
C. 45 years old, male, 15 pack-year smoking B- Acute shortness of breath is usually associated
history, progressive dyspnea with sudden physiological changes, such as
D. 45 years old, male, 10 pack-years smoking laryngeal edema, bronchospasm, myocardial
history, fever, cough, dyspnea infarction, pulmonary embolism, or
pneumothorax.
D- More likely pneumonia (triad of fever, cough and
dyspnea)

54. What is the gold standard for the diagnosis of a D Restrictive Ventilatory Defect = DECREASED TLC or
restrictive ventilatory defect? FVC (however, a low FVC has many possible causes. So
A. Low FVC we use TLC to be sure of a restrictive vent. defect)
B. Low FEV1/FVC
C. Low FEV1
D. Low TLC

55. What is the most cited mechanism of infective C The most widely cited mechanism of infectious
bronchiectasis? bronchiectasis is the “vicious cycle hypothesis,”
A. Homozygous apha1 deficiency
B. Protease-antiprotease hypothesis susceptibility to infection + poor mucociliary clearance =
C. Vicious cycle hypothesis microbial colonization of the bronchial tree
D. Immune-mediated reaction
MED 2- First Shifting Examination (A2022) Page 15 of 26

Pseudomonas aeruginosa, exhibit a particular propensity


for colonizing damaged airways and evading host
defense mechanisms.

56. Which of the following is a criteria for patient B


discharge for those admitted for CAP?
A. RR 12-16/minute
B. Temperature 36-37.5 degrees Celsius
C. PR 80-12/min
D. O2 saturation 88-92%

57. Which diagnostic plans are recommended to be D Highly suspicious for pneumonia (triad of fever, cough and
done in a patient manifesting with fever, cough and dyspnea – acute)
dyspnea for 1 week, and bronchial breath sounds
on the right base? Sputum GS, CBC, Chest Xray is most appropriate
A. Spirometry, Chest CT scan, Sputum Gene
Xpert
B. BUN, CBC, sputum AFB smear
C. CBC with platelet, ABG. 12 LECG
D. Sputum GS, CBC, Chest Xray

58. Which location of pneumonia will manifest with a D


(+) silhouette sign on the right cardiac border?
A. RUL apical segment
B. RLL basal segment
C. RLL superior segment
D. RML medial segment

59. Which of the following are signs of extra C A, B and D are signs of intrathoracic spread
thoracic spread of lung cancer?
A. Horner's Syndrome
B. Pleural Effusion
C. Adrenal insufficiency
D. SVC syndrome

60. Which of the following will be the most cost- B Despite its limited sensitivity (compared to other
effective tool in the diagnosis of PTB? methods), DSSM still remains the mainstay of diagnosis
A. CXR for TB in resource limited settings.
B. DSSM
C. GeneXpert
D. MTB culture
MED 2- First Shifting Examination (A2022) Page 16 of 26

61. In stepwise management of asthma, what is the B


preferred reliever medication in patients being
treated as Step 1 and Step 2?
A. Low-dose ICS daily
B. ICS-Formoterol as needed step 1-2
C. ICS-Salbutamol as needed
D. Salbutamol as needed

62. Rita, a third-year medical student, attends your B


clinic with her stepmother. In the past month, Rita
has experienced sudden spells of breathlessness
which resolved spontaneously. Her energy levels
are lower than normal, and she is worried she won’t
do best in school. Her stepmother has asthma and
she is worried Rita might be developing it too.
Which of the following symptoms/signs would make
you consider that Rita most likely is NOT
asthmatic?
A. Symptoms often worse at night
B. Breathlessness with lightheadedness and
peripheral tingling
C. History of eczema and allergic rhinitis
D. Lack of audible wheeze on auscultation during
a symptomatic time

63. What are the clinical features of patients who A *Question to be reviewed by faculty
develop extensive thrombosis i.e. affecting 50% of
the pulmonary vasculature?
A. Dyspnea, syncope, hypotension, and cyanosis
B. Hypotension, chest pain, and palpitations
C. Leg pain, dyspnea and tachycardia
D. Dyspnea, hypotension and chest pain
MED 2- First Shifting Examination (A2022) Page 17 of 26

64. What is VTE prophylaxis recommended for patients A


with moderate risk for VTE and is admitted at the
ICU because of septic shock and has no active
bleeding?
a. LMWH
b. UFH
c. GCS
d. IPC

65. What is the classical radiographic finding in pleural B


effusion?
A. Air fluid level
B. Homogenous density with curved upper border
C. Homogenous density obscuring the cardiac
border
D. Triangular density with the apex pointing
towards the hilum

66. What is the gold standard for the diagnosis of TB? C


A. MTB/Rif Assay (GeneXpert)
B. Sputum AFB smear
C. Sputum MTB culture
D. TB-LAMP assay
MED 2- First Shifting Examination (A2022) Page 18 of 26

67. A 42 y/o male from Caloocan City was diagnosed B


with bacteriologically confirmed pulmonary TB.
Which of the following tests is recommended prior
to initiation of anti-TB treatment?
A. Serum uric acid
B. HIV screening
C. HbA1c
D. perimetry

68. 48 year old female patient was seen at the OPD C


and for re-treatment of PTB. Her sputum
GeneXpert was negative for Rifampicin resistance.
What will be given? (Based on the lecture)
A. 2HRE/4HR
B. 2HRZE/4HR
C. 2HRZES/1HRZE/5HRE
D. 4HRZE/2HR

69. A 21 year old male, has juvenile diabetes and was C


assessed to have PTB, which of the following CXR
findings will be more consistent his risk-profile?
A. Hyperinflated lungs
B. Bilateral pleural effusion
C. RUL cavitation
D. Normal CXR

70. A 22-year-old female obese student presents with B


episodic dyspnea, chest tightness, and cough. She
has a history of atopy as a child. She was seen
several months ago for these symptoms at the ER
and was given a salbutamol inhaler, which provided
symptomatic relief. She confines that she has
consumed 12 canisters of salbutamol for the past
year. For the past 4 weeks, she has been using
salbutamol over 3x a week at daytime and at least
two times per week at night when she awakens
with symptoms. Your assessment is Uncontrolled
asthma. What modifiable factor present in the
patient will make her at risk for asthma death?

A. Use of SABA
B. No ICS in her treatment
C. History of atopy
D. Obesity
MED 2- First Shifting Examination (A2022) Page 19 of 26

71. A 28-year-old male driver presents to your GP D


clinic with worsening asthma. Which of the
following observations should alert you that the
patient has acute severe asthma?

a. Talks in phrases
b. Respiratory rate 22 breaths/min
c. Audible inspiratory and expiratory wheeze
d. Visible use of accessory muscle

72. A 35-year-old female, Filipino, non-smoker A Adenocarcinoma


consulted because of right-sided pleuritic pain. Her • Most common over-all
chest radiograph showed a peripheral lung mass • Most common in non-smokers
suspicious for malignancy. Based on her clinical • Most common histologic type in women and
profile, which cell type is likely? young adults
A. Adenocarcinoma • Peripheral lesion
B. Large cell carcinoma • Metastasizes early
C. Small cell carcinoma
D. Squamous cell carcinoma
73. A 36-year-old smoker consulted because of C Increase in post bronchodilator FEV1 from pre
episodic shortness of breath. He is a current heavy bronchodilator is more than 200ml and 12% à confirms
smoker with a 30 pack-year smoking history. Which reversibility of the obstruction/airflow limitation
of the following features in his spirometry report
points to asthma as the cause of his dyspnea?
a. FEV/FVC is lower than the limit of normality
b. Flow volume loop is short with an
exaggerated concavity of the expiratory limb.
c. Increase in post bronchodilator FEV1 from
pre bronchodilator is more than 200ml and
12%
d. FVC post bronchodilator value is low.

74. A 42-year-old female consulted because of C Normal FEV1/FVC; Normal FEV1; Low FVC
progressive shortness of breath. Her spirometry = Probable Restrictive Ventilatory Defect
test showed the following: Normal FEV1/FVC; P-A-I-N-T (Pleura, Alveoli, Interstitium, Neuromuscular,
Normal FEV1; Low FVC. Based on this, which of Thoracic Cage)
the following conditions can be considered?
A. Asthma
B. COPD
C. PTB
D. Tracheal stenosis
MED 2- First Shifting Examination (A2022) Page 20 of 26

75. A 49 year-old female, previously treated for PTB D Approach to Hemoptysis (Emergency)
came in due to hemoptysis of approximately 800cc.
She appeared pale, drowsy and hypotensive. What will
you do?
A. Perform lung biopsy
B. Request for Chest CT
C. Nebulize
D. Intubate

76. A 50 y/o obese male, with DM and HTN D


(controlled) presents to the TR with sudden dyspnea
and chest pain. There are no leg signs. ECG showed
sinus tachycardia. ABG revealed respiratory alkalosis.
Chest x-ray was essentially normal. A diagnosis of
pulmonary embolism is strongly entertained. Which
among the following is the best choice regarding initial
treatment of PE?
A. An initial bolus of unfractionated heparin at 80
U/kg, followed by an initial infusion rate of 18
U/kg per h to achieve a apTT of 60–80 s.
B. Warfarin requires 3 days to take full effect and
is usually titrated empirically to achieve the
target INR of 2.0 – 3.0 s.
C. Oral anticoagulation monotherapy with
apixaban with a 1-week loading dose,
respectively, followed by a maintenance dose
without parenteral anticoagulation is not an
option for treatment.
D. Fondaparinux can be given as it does not
cause heparin-induced thrombocytopenia but
requires frequent laboratory monitoring unlike
LMWH and Unfractionated heparin

77. A 50 year old patient was diagnosed with TB 2 C Exudative Effusion d/t TB
months ago, but does not take her medications
regularly. She has been experiencing one month Egophony - increased resonance of voice sounds heard
progressive dyspnea and trepopnea, and prefers to lie when auscultating the lungs, often caused by lung
on her left side. What PE finding is most compatible consolidation or presence of fluid.
with a pleural effusion as the cause of her symptoms?
A. Apex beat in anterior axillary line Patients with trepopnea in most lung diseases prefer to
B. Dullness on the right lie on the opposite side of the diseased lung, as the
C. Egophony in right lower lung field gravitation increases perfusion of the lower lung.
D. Decreased breath sounds on the left

78. A 50 year old patient was diagnosed with TB 2 B


months ago, but does not take her medications
regularly. She has been experiencing one month
progressive dyspnea and trepopnea, and prefers to lie
on her left side. If we were to examine the fluid, what is
an expected finding, based on her clinical
manifestations?
A. Pleural fluid LDH < 2/3 serum LDH
B. Predominance of lymphocytes
C. Clear, transparent, odorless fluid
D. Numerous mesothelial cells

A, B and D – points to characteristics of transudative fluid


MED 2- First Shifting Examination (A2022) Page 21 of 26

79. A 50 year old patient was diagnosed with TB 2 D


months ago, but does not take her medications
regularly. She has been experiencing one month
progressive dyspnea and trepopnea, and prefers to lie
on her left side. If she was diagnosed to have a
“complicated parapneumonic effusion”, what is the
recommended therapeutic option?
VATS – used for loculated pleural effusion (to break up all
A. High dose IV antibiotics
locules then chest tube is inserted to drain the fluid)
B. Intrapleural instillation of medications
C. Drainage of fluid
D. Video assisted thoracoscopic surgery

80. A 50 year old patient was diagnosed with TB 2 C Pneumothorax


months ago, but does not take her medications
regularly. She has been experiencing one month
progressive dyspnea and trepopnea, and prefers to lie
on her left side. Should the patient undergo
thoracentesis, what should be considered if she has
sudden severe dyspnea, and is noted to have
compressible pulses during the procedure?

A. Re-accumulation of pleural effusion


B. Massive hemothorax
C. Tension pneumothorax
D. Re-expansion pulmonary edema

81. A 55 year-old COPD patient has mMRC of 4 and B


was admitted last January for exacerbation. He is
maintained on LABA + LAMA. He came back due to
dyspnea. His Blood Eosinophil Count is 320. What will
be your option?
A. LAMA monotherapy
B. LABA + LAMA + ICS
C. Maintain on LABA + LAMA
D. Maintain on oral steroids

82. A 57-year-old patient smoker consulted at a clinic C Copious amounts of sputum, recurrent pulmonary
because of chronic cough characterized by copious infections & airway obstruction all point us to
sputum production. He has been treated with a cocktail Bronchiectasis.
of antibiotics but still he would experience these
recurrent pulmonary infections. On PE, he has CT scan is used for diagnosing bronchiectasis.
persistent inspiratory crackles at the base. You
requested a spirometry, but the results showed no
airway obstruction. What will you do next to search for
a diagnosis?
MED 2- First Shifting Examination (A2022) Page 22 of 26

A. Methacholine bronchoprovocation test


B. ABG
C. High-resolution CT scan
D. V/Q scan
b
83. A 63 year-old male, chronic smoker, came in due B Old age, smoker, weight loss, lymphadenopathies,
to hemoptysis, weight loss and hoarseness. There rounded density on x-ray à Malignancy
were cervical and supraclavicular lymphadenopathies.
His CXR showed a rounded density on the RUL. What
is your consideration?
A. COVID-19
B. Malignancy
C. PTB
D. Bronchiectasis

84. A 65 year old male was diagnosed with small cell C


lung cancer extensive stage, what is the treatment
recommendation for this patient?

A. Radiotherapy
B. Radiotherapy + chemotherapy
C. Chemotherapy
D. Surgery + chemotherapy

85. A 65 year old female was admitted at the ICU due C


to 1 week history of cough, fever and dyspnea. She is
hypertensive, nonsmoker, no antibiotic use for the past
3 months. Which empiric antibiotic/s is/are appropriate
for the patient?
A. Amoxicillin + doxycycline
B. Piperacillin tazobactam + levofloxacin
C. Ceftriaxone + azithromycin
D. Ertapenem + clarithromycin

86. A 72-year-old male, chronic smoker consulted D


because of dyspnea, chronic cough and hemoptysis.
On chest PE there was wheezing over the entire left
lung field. If lung cancer is considered, which is the
likely presumptive cell type based on his clinical
presentation?
A. Adenocarcinoma
B. Carcinoid Tumor
C. Large Cell Carcinoma
MED 2- First Shifting Examination (A2022) Page 23 of 26

D. Squamous Cell Carcinoma

87. A post-op cancer patient manifests with sudden A Bedridden – predisposes patient to VTE and Pulmonary
dyspnea and left sided chest pain while bedridden after embolism
surgery. What radiographic finding may be expected?
A. Localized area of oligemia (+) Westermark’s Sign – localized are of oligemia;
B. R hemidiaphragm at 11th ICS decreased bronchovascular markings = signifies
C. Tram lines pulmonary embolism
D. Prominence of vasculature in upper lungs

88. Consistent with a lung auscultation findings of a C Physical Examination findings in COPD
COPD patient. • Use of accessory respiratory muscles and
A. Stridor paradoxical indrawing of lower intercostal space
B. Unilateral wheezing (Hoover sign)
C. Decreased breath sounds • In advanced disease – cyanosis, elevated JVP;
D. Bibasal crackles peripheral edema
• Pursed lip; tripod; hyperinflation (barrel chest)
• Wheezing – on forced and unforced expiration
• Diffusely decreased breath sounds (distant)
• Hyperresonance on percussion
• Prolonged expiration
• Coarse crackles beginning with inspiration

89. For patients assessed as moderate risk D


CAP, which could be an expected physical
examination finding?

A. on mechanical ventilator
B. Oxygen saturation of 86% at room air
C. tachypneic at 25/minute
D. BP 70 palpatory
MED 2- First Shifting Examination (A2022) Page 24 of 26

90. If the blood pressure at the emergency room is C


palpatory 80, and there is a high probability of VTE,
Aside from vasopressors, which treatment regimen is
recommended provided that diagnostic test shows
pulmonary embolism?

A. Enoxaparin
B. Fondaparinux + aspirin
C. Unfractionated heparin + alteplase
D. Alteplase + Clopidogrel

91. The respiratory disturbance index of a sleep study C RDI = (hypopneas + apneas) ÷ no. of hours of sleep
that had 50 apneas and 50 hypopneas over 5 hours is: (50 + 50) ÷ 5 = 20
A. 10
B. 15
C. 20
D. 25
92. Ventilatory response to chemical and mechanical D
stimuli may be transiently reduced or abolished during:

A. N1
B. N2
C. N3
D. REM
MED 2- First Shifting Examination (A2022) Page 25 of 26

93 The blood gas of a patient had an overdose of B Morphine causes respiratory depression
morphine without artificial ventilation would most likely Less CO2 blown off à CO2 retention à increased
show: PaCO2
A. Increased pH
B. Increased PaCO2
C. Increased PaO2
D. Decreased HCO3

94. The acid base interpretation of the ABG with a pH: D


7.47, paCO2: 33, paO2: 186 and a HCO3: 27
A. Normal acid base
B. Fully compensated respiratory alkalosis
C. Fully compensated metabolic alkalosis
D. Combined respiratory and metabolic alkalosis

95. A FiO2 of 100% is being administered did not C


improve the oxygenation of a patient with a widened
(A-a)O2. The mechanism of hypoxemia is most likely:
A. Low PiO2
B. High altitude
C. Shunt
D. Ventilation-perfusion mismatch

96. Which of the following is true given the ABG of a


70 year old male with difficulty of breathing:
pH = 7.28 pC02 = 68 p02 = 50 HC03= 29 Sa02 = 85%
taken at room air
MED 2- First Shifting Examination (A2022) Page 26 of 26

97. The index of oxygenation that is interpreted B


independent of age is:
A. (A-a) O2
B. a/A O2
C. PaO2
D. PaO2/FiO2

98. Type 1 respiratory failure is characterized by:

99. The patient with COVID can be considered to B


have ARDS because:
A. Respiratory distress develops three weeks
after clinical insult
B. PaO2/FiO2 is 100
C. Signs of left ventricular failure are present
D. Infiltrates are localized/unilateral

100. Contraindication to non-invasive ventilation is: A Absolute contraindications for Non-invasive


A. Respiratory arrest Mechanical Ventilation:
B. GCS of 15 • Coma
C. Intercostal muscle retractions • Cardiac Arrest
D. Pulmonary congestion • Respiratory Arrest
• Any condition requiring immediate intubation
MEDICINE 2 (CARDIOLOGY-PULMONOLOGY)

SHIFTING EXAM
16 MARCH 2021
CARDIOLOGY
QUESTION ANSWER RATIONALE
1. A 65 year old patient, known diabetic, hypertensive, and had D Doc’s ratio:
previous STEMI presented at the ER with exertional dyspnea, • Patient presents with heart failure manifestations:
orthopnea, bipedal edema, distended neck veins and Bibasal exertional dyspnea, orthopnea, bipedal edema,
crackles. BP 70/40 on three vasopressors, HR 110 bpm, RR 24 distended neck veins
cpm, O2 stats 89%. Echocardiogram showed a reduced • All three vasopressors are already being used but
systolic function and segmental wall motion abnormality. Which despite this, patient remains hypotensive.
heart failure stage does the patient belong to? • Echo showed multiple segmental wall
A. Stage A abnormality and reduced systolic functions
B. Stage B • This is intractable heart failure already
C. Stage C
D. Stage D Stages of Heart Failure:
• Stage A: at high risk for HF but without structural
heart disease of symptoms of HF
• Stage B: structural heart disease but without
signs or symptoms of HF
• Stage C: structural heart disease with prior or
current symptoms of HF
• Stage D: refractory HF

Cardiology Trans 2022: Heart Failure


2. Which historical information points to myocarditis as cause of a D Doc’s ratio:
patient’s heart failure symptoms? • Look for an infection: body malaise, fever and
A. Chemotherapy with anthracycline at >200 mg/m2 cough
B. History of repaired muscular VSD with residual shunt
C. History of tetracanabinoid use for 6 years Myocarditis: Clinical presentation
D. Body malaise, fever, and cough one week prior to • Typical symptoms of heart failure after
symptoms following a viral syndrome
• Typical signs and symptoms of infection
• Chest pain from pericarditis
• Symptoms secondary to arrhythmia may occur
• The typical patients with presumed viral
myocarditis is a young to middle aged adult who
develops progressive dyspnea and weakness
within a few days to weeks after a viral
syndrome that was accompanied by fever and
myalgias

Cardiology Trans 2022: Cardiomyopathy


3. A 76 year old male patient with COPD and history of heart D Doc’s Ratio:
failure came in due to dyspnea on ambulation, intermittent • Pick a test that will differentiate the dyspnea of
cough, and easy fatigability. He has discontinued his COPD and of heart failure
medications for the past three days. On physical examination,
BP 110/60 HR 108 regular, RR 26 cpm, O2 saturation 94%, Natriuretic peptides
apex beat at 5th LICS AAL, sustained apex beat, grade 3/6 • Are released by failing hearts and acts as a
systolic murmur on the 5-6th LICS AAL with (-) Carvallo’s signs, sensitive marker to rule in cardiac cause of
minimal crackles on both lung bases. At the emergency room, dyspnea
which diagnostic exam will you request to rapidly rule in cardiac
cause of the dyspnea?
A. Chest X-ray
B. Cardiac MRI
C. Echocardiogram Cardiology Trans 2022: Heart Failure
D. Natriuretic peptides

TEAM 18, TEAM 14, TEAM 16, TEAM 9 Page 1 of 24


MED 2 – Cardiology-Pulmonology – Shifting Exam (16 MARCH 2021) Page 2 of 24

4. A 66 year old male patient presented at the ER chest pain, D Doc’s Ratio:
dyspnea on exertion, orthopnea, paroxysmal nocturnal • (+) Heart failure manifestations
dyspnea, bipedal edema. Physical examination showed BP • Patient is congested therefore your initial
100/60 HR 110 RR 23, AB 6th LICS AAL, (+) S3, no murmurs, management is to use a diuretic on your patient
bibasal crackles from T7, grade 2 bipedal edema. Which of the at the emergency room
following is an appropriate initial management at the ER?
A. Start nitroglycerine 10 mg/hr infusion Treatment Modalities for Heart Failure:
B. Start subcutaneous heparin for anticoagulation • Congestion: IV loop diuretics
C. Give Aspirin 325 mg/tab and Clopidogrel 75 mg/tab, 4 o Congestive signs: orthopnea, jugular turgor,
tabs orally hepatomegaly, sloping edema, crackles
D. Give furosemide 40 mg IV push Cardiology Trans 2022: Heart Failure
5. A 23 year old patient admitted due to Dengue fever suddenly B Doc’s ratio:
had chest pain, intermittent dyspnea, orthopnea, paroxysmal • BP 70/40, our patient is decompensated because
nocturnal dyspnea, bipedal edema. Physical examination of heart failure but the cause is dengue
showed BP 70/40 HR 110 RR 23 AB 5th LICS MCL, (+) S3, no • There is no treatment for dengue so just
murmurs, Bibasal crackles from T7, grade 2 bipedal edema. support using inotropics for the heart function
Which of the following is an appropriate management for the of the patient hoping that it will recover
underlying cause of decompensation?
A. Percutaneous intervention for coronary Management of Dilated Cardiomyopathy (Major
atherosclerosis causes: Infective – any infection, even dengue and
B. Inotropes to support cardiac pump function leptospirosis)
C. Anti-arrhythmics to decrease heart rate • No specific therapy
D. Guideline directed antiviral agents • Immunosuppressive agents and anti-
inflammatory agents are not advocated
• Address heart failure with inotropes
Cardiology Trans 2022: Cardiomyopathy
6. Which of the following is included in the management of D Doc’s ratio:
patients with Hypertrophic cardiomyopathy? • Need to check the other family members
A. Determining stroke risk because this is hereditary
B. Achieving heart rate <70 bpm
C. Improving LV ejection fraction at peak exercise Management of Hypertrophic Cardiomyopathy:
D. Screening of family member • Family Screening
• Symptom alleviation: Lifestyle modification, use
of beta-blockers and calcium-channel blockers,
septal reduction therapy
• Sudden Cardiac death risk stratification: ICD
• For Syncope/ Presyncope, Chest pain/ Dyspnea:
o Beta blockers, DHP CCBs, Disopyramide
o Reduce LVOT obstruction by: Slowing heart
sate, enhancing diastolic filling, decreasing
contractility
• For refractory patient – Surgical myectomy or
alcohol septal ablation
• For cardiac arrest patients and even with
asymptomatic patients as long as they present
with two or more risk factors – implantable
cardioverter – defibrillator
Cardiology Trans 2022: Cardiomyopathy
7. Which ECG shows myocardial ischemia? C Doc’s ratio:
A. 1.5mm ST elevation V2, aVL, aVF • Important here is significant ST elevation (more
B. 1mm ST depression in I, aVF, V1 than 1 mm up and 0.5 mm down)
C. 1.5 mm ST depression in II, III, V1 • Look at contiguous leads
D. 1mm ST elevation in I, II, aVR
Inferior Leads II, III, aVF

Lateral Leads aVL, V5 and V6

Anterior V1, V2, V3, V4

Anterior Septal V1 and V2

Posterior No leads directly show the


depolarization but V1 and
V2 will record “mirror
images” of the posterior
region

Cardiology Trans 2022: Abnormal ECG


MED 2 – Cardiology-Pulmonology – Shifting Exam (16 MARCH 2021) Page 3 of 24

8. Indications for Coronary Angiography EXCEPT D Doc’s ratio:


A. Stable angina who are severely symptomatic despite • Patients with normal ventricular function
medical therapy
B. Troublesome symptoms that present diagnostic Coronary Angiography Indications:
difficulties • Patients with stable angina who are severely
C. Known or possible angina pectoris who have survived symptomatic despite medical therapy
cardiac arrest • Troublesome symptoms that present diagnostic
D. Patients with angina with normal ventricular function difficulties
on echocardiography • Known of possible angina pectoris who have
suffered cardiac arrest
• Patients with angina or evidence of ischemia
on non-invasive testing with clinical or
laboratory evidence on ventricular
dysfunction
• Patients judged to be at high risk of sustaining
coronary events

Cardiology Trans 2022: Ischemic Heart Disease


9. Nitrates act as an anti-anginal medication by D Doc’s ratio:
A. Dilating subendocardial coronary vessels • It’s the preload unloader therefore venodilation
B. Decreasing heart rate, arterial pressure and which reduces end diastolic volume and pressure
myocardial contractility
C. Inhibiting platelet cyclooxygenase thereby interfering Nitrates
with platelet activation • Mechanism of action: Systemic venodilation
D. Systemic venodilation with reduction in LV end- with concomitant reduction in left ventricular
diastolic volume and pressure end-diastolic volume and pressure (decrease
preload)

Cardiology Trans 2022: Ischemic Heart Disease


10. This medication reduces ventricular remodeling & mortality after B Doc’s ratio:
STEMI Medications that reduce ventricular remodeling and
A. Aspirin mortality includes:
B. Enalapril • Angiotensin converting enzyme inhibitors
C. Carvedilol (ACE inhibitors)
D. Felodipine o Enalapril
• Angiotensin II receptor blockers (ARB)
• Mineralocorticoid receptor antagonist (MRA)

Drugs that reduce mortality and morbidity


• ACE-I, Beta blockers, Nitrates, Aldosterone
antagonists

Cardiology Trans 2022: Heart Failure


11. This rhythm is considered a fatal complication of myocardial D Doc’s ratio:
infarction A fatal complication of MI is ventricular fibrillation
A. Atrial fibrillation
B. Supraventricular tachycardia Ventricular fibrillation
C. Right bundle branch block • Chaotic; No discernable QRS complexes and P
D. Ventricular fibrillation wave
• Patient in cardiac arrest
• Dreaded complication of STEMI (10-20% of
cases)
• Comprise ~50% of causes of cardiovascular
death

Cardiology Trans 2022: CCC 2: Case with Chest


pain (STEMI)
MED 2 – Cardiology-Pulmonology – Shifting Exam (16 MARCH 2021) Page 4 of 24

12. An unidentified male was rushed to the emergency room D Doc’s ratio:
because of unresponsiveness. On initial assessment, he was The tracing shows a fine ventricular fibrillation.
unconscious and could not be aroused, with zero heart rate and As you can see there are undulations but very fine.
blood pressure. On hooking to a cardiac monitor, the following Asystole is flatter.
tracing was taken. What is your ECG interpretation?
Ventricular Fibrillation
• Can come from ventricular tachycardia
A. Asystole • Associated with coarse or fine chaotic
B. Atrial fibrillation undulations of the ECG baseline
C. Ventricular tachycardia • No define complexes
D. Ventricular fibrillation • No P wave
• No true QRS complexes
• Indeterminate rate
• NUMBER 1 CAUSE OF SUDDEN DEATH
• All patients with ventricular fibrillation are in
cardiac arrest

Cardiology Trans 2022: Abnormal ECG


13. An unidentified male was rushed to the emergency room D Doc’s ratio:
because of unresponsiveness. On initial assessment, he was Patient with zero heart rate and blood pressure with
unconscious and could not be aroused, with zero heart rate and ECG showing asystole (flat line). Next best action
blood pressure. Cardiac monitor showed the tracing below. is CPR starting with chest compressions.
Which is the next best action?
Ventricular fibrillation
Tips and Tricks: Cardiac Arrest
● If rhythm is either VF or Asystole
A. Check for breathing
→ Don’t check for pulse anymore
B. Check arterial pulse
→ DO CPR
C. Do precordial thump
→ Defibrillate VF
D. Do chest compression
→ Ventricular Fibrillation or Pulseless VT
● Remember “SCREAM”
→ Shock → CPR → Rhythm check →
Epinephrine → Antiarrhythmic Medication
(Amiodarone)
→ If asystole or PEA
■ Do P-E → Pump (CPR) and Epinephrine
■ Check for reversible causes: H’s & T’s
● Reminders
→ Defibrillation
■ 200 J for biphasic defibrillator
■ 360 J for monophasic defibrillator
→ Always do chest compressions immediately
after defibrillation
→ Lidocaine IV – alternative to amiodarone

Cardiology Trans 2022: Disorders of Rhythm


14. A 50y male was brought unconscious into the emergency C Doc’s ratio:
department. Vital signs showed BP palpatory 60 mm Hg, HR 45 Patient is hypotensive (BP palpatory 60 mmHg)
bpm, RR 24. On hooking to a cardiac monitor, the following ECG findings: regular R to R, regular P to P but P is
tracing was taken. Identify the ECG rhythm: not related to the QRS (P is not associated with the
QRS). This signifies a 3rd degree AV block.

3rd degree AV block


A. 2nd degree AV block Mobitz 1 • Complete heart block or complete atrioventricular
B. 2nd degree AV block Mobitz 2 block
C. 3rd degree AV block • Impulses originate at both SA node and at an
D. High grade AV block escape
• pacemaker below the block
• Regularly occurring P waves and QRS
complexes
MED 2 – Cardiology-Pulmonology – Shifting Exam (16 MARCH 2021) Page 5 of 24

• Atrioventricular Dissociation
• P waves (atrial depolarization) not related to the
QRS complexes (ventricular depolarization);
more P waves than QRS because atrial rate is
faster than ventricular rate
• QRS complexes are often abnormal in shape,
duration and axis
• QRS morphology is constant (15- 60 beats/min)
• Atrial activity most commonly sinus initiated
• Ventricular rhythm is maintained by junctional or
idioventricular escape rhythm or a ventricular
pacemaker
• Always Regular

Cardiology Trans 2022: Abnormal ECG


15. A 50y male was brought unconscious into the emergency C Doc’s ratio:
department. Vital signs showed BP palpatory 60 mm Hg, HR 45 This is a case of a 3rd degree AV block with
bpm, RR 24. On hooking to a cardiac monitor, the following hypotension. Management will be inserting a
tracing was taken. What is the management? temporary transvenous pacemaker. But before
that while waiting we can also give Atropine.

Principles of Management
A. Do vagal maneuvers such as carotid massage • Exclude reversible causes
B. Give adenosine 6 mg IV rapid push • Emergency Management (temporizing only):
C. Insert temporary transvenous pacemaker Medical therapy is only effective as a short-term
D. Do synchronized cardioversion with 100 joules emergency measure, until pacing can be
accomplished.
→ Atropine IV push
→ Epinephrine infusion (drip and not push)
→ Dopamine infusion
→ Isoprenaline infusion
• Temporary pacemaker
→ Transcutaneous pacemaker
§ Starting point; similar equipment to
defibrillator; attachment of patch on chest
that would deliver electrical impulse to heart
§ Its duration is limited by patient discomfort
and longer- term failure to capture the
ventricle owing to changes in lead
impedance.
→ Transvenous pacemaker
§ Insert a central line into IJV, subclavian, or
catheter into femoral vein positioning to the
right ventricle to deliver your transvenous
pacing.
§ Used if a patient requires more than a few
minutes of pacemaker support

Cardiology Trans 2022: Disorders of Rhythm


MED 2 – Cardiology-Pulmonology – Shifting Exam (16 MARCH 2021) Page 6 of 24

16. A 72y male hypertensive and diabetic consults because of A Doc’s ratio:
palpitations and dyspnea. Vital signs: BP 140/100 mm Hg, HR In the ECG tracing, we can see a premature beat/
84 bpm, RR 19. His ECG tracing is shown below. Which complex but most of it is still sinus. Vital signs are
statement is correct? still stable, there is no need to do the other
interventions besides requiring further tests and
investigation.

A.This patient requires further tests and investigation


B.This patient requires immediate antiarrhythmic
medications
C. This patient requires immediate synchronized
cardioversion
D. This patient requires immediate defibrillation
17. A 33y female consults because of palpitations. Vital signs D Doc’s ratio:
showed BP 120/80, HR 190 bpm, RR 18. Her ECG tracing is For this patient, BP is 120/80 so she is still
shown below. What is the diagnosis and management? hemodynamically stable. ECG tracing presents a
ventricular tachycardia. Since she is still stable,
management will only be medication
(Amiodarone 150 mg IV) to slow down the heart
A. Sinus tachycardia, give Metoprolol 50 mg tab q12h rate but if she becomes hypotensive, we need to do
B. Supraventricular tachycardia, give Adenosine 6 mg IV cardioversion.
C. Multifocal atrial tachycardia, give Verapamil 2.5 mg IV
D. Ventricular tachycardia, give Amiodarone 150 mg IV Acute Management of Sustained Ventricular
Tachycardia
● Stable
® IV amiodarone, lidocaine, procainamide
● Unstable (Hypotensive, HF, Syncope, Angina, or
Persistent)
® Synchronized DC cardioversion
● Pulseless
® Defibrillation (treat as VF)

Cardiology Trans 2022: Disorders of Rhythm


18. An 80y old patient was brought in unconscious in the ER. Vital A Doc’s ratio:
signs zero. Emergency measures were instituted, and patient ECG findings: ventricular fibrillation
was hooked to a cardiac monitor which showed the tracing The patient’s vital signs zero, so patient is
below. Which is the correct statement? arrested. next best thing to do is defibrillate
immediately. Treatment of choice for ventricular
fibrillation is defibrillation.

A. This is a shockable rhythm and immediate Shockable rhythms


defibrillation must be done • Ventricular fibrillation
B. ECG shows ventricular tachycardia and defibrillation • Pulseless ventricular tachycardia
must be done
C. This is a nonshockable rhythm and requires 1. On initial recognition of such rhythms, deliver a
immediate chest compression shock (200J if biphasic, 360J if monophasic), then
D. ECG shows asystole and defibrillation must be done initiate chest compressions immediately for 2 mins.
2. If the rhythm is still shockable upon checking,
deliver another shock with the same joules, and
resume CPR after for 2 mins
● Administer IV epinephrine every 3-5 mins
● Consider advanced airway and capnography
3. If the rhythm is still shockable upon checking,
deliver another shock with the same joules, and
resume CPR after for 2 mins
● Administer amiodarone 300mg for refractory VF
(anti-arrhythmic) or Lidocaine as an alternative
for refractory VF
● Treat reversible causes
● If the patient is still in VT/VF, continue
defibrillation and CPR until patient is revived.

Cardiology Trans 2022: CCC 2: Case with Chest


pain (STEMI)
MED 2 – Cardiology-Pulmonology – Shifting Exam (16 MARCH 2021) Page 7 of 24

19. One of the following is an indication for permanent pacing: C Permanent Pacemaker Indications
A. First degree AV block ● Sick sinus syndrome
B. Sinus bradycardia ● Symptomatic heart block
C. Complete heart block ® 2nd degree, Mobitz II
D. Sinus arrhythmia ® 3rd degree AV block
● Symptomatic sinus bradycardia due to long term
drug therapy of a type and at a dose for which
there is no accepted alternative
● Keyword: “Symptomatic Bradycardia”

Cardiology Trans 2022: Disorders of Rhythm


20. A 72y hypertensive diabetic patient presents with shortness of C Doc’s ratio:
breath and palpitations. ECG showed: The ECG shows an atrial fibrillation:
● Irregular RR interval
● No definite T for every QRS
There is uncoordinated activity in the atria thus it
will not contract properly. If there is no atrial
Which physical examination finding is consistent with the above contraction, the atrial “kick” will also be lost. No
rhythm? atrial kick = no A wave.
A. Prominent A waves on the JVP
B. Prominent V waves on the JVP Atrial fibrillation
C. Absent A waves on the JVP ● No discernable P waves
D. Absent V waves on the JVP ● Low amplitude chaotic f (fibrillatory) waves
● Irregular RR interval
● Most common sustained cardiac arrhythmia
● Increased incidence in elderly and those with
structural heart disease
Bedside Examination of AF
● Pulse deficit
● Irregularly irregular pulse
● Loss of the A wave in JVP
● Varying intensity of S1
● Varying amplitude of pulse

Cardiology Trans 2022: Disorders of Rhythm


21. Which of the following patients with AF has the highest risk of C CHA2DS2-VASc Score
cardioembolic stroke? Risk Score
A. A 70y hypertensive diabetic male with no history of Congestive heart failure/LVEF ≤ 40% 1
failure and no previous stroke Hypertension 1
B. A 70y hypertensive nondiabetic male with no history Aged ≥75 years 2
of failure and with no previous stroke Diabetes Mellitus 1
C. A 70y hypertensive diabetic female with no history of Stroke/TIA/TE 2
failure and no previous stroke Vascular disease (prior MI, PAD, or 1
D. A 70y hypertensive nondiabetic female with no history aortic plaque)
of failure and no previous stroke Aged 65-74 1
Sex category (female) 1
Maximum Score 9

0: No treatment
1: Favor anticoagulant (NOAC or Warfarin INR 2-3)
2: Must give anticoagulant (NOAC or Warfarin INR
2-3)

Patient A Score = 3
Patient B Score = 2
Patient C Score = 4
Patient D score = 3

Cardiology Trans 2022: Disorders of Rhythm


MED 2 – Cardiology-Pulmonology – Shifting Exam (16 MARCH 2021) Page 8 of 24

22. A 63y known hypertensive for 20 years but poorly compliant C Doc’s ratio:
with medications presents with sudden onset of severe downing Patient with elevated blood pressure presenting
sensation that awakened the patient. This was accompanied by with acute pulmonary edema is considered
diaphoresis. Vital signs showed BP of 210/120, CR 110bpm hypertensive emergency.
regular, RR 32cpm, O2 sat 89% at room air. Crackles were
noted on both lung fields. JNC 7 Guidelines for Classification of Hypertension in Adults
How would you classify the hypertension of the patient? Classification SBP DBP
A. Stage 1 HTN (mmHg) (mmHg)
B. Stage 2 HTN Normal <120 and <80
C. Hypertensive emergency Prehypertension 120-139 or 80-89
D. Hypertensive urgency Stage 1 140-159 or 90-99
Stage 2 >160 or ≥100

Hypertensive emergency
● SBP >180 or DBP 120mmHg associated with
impending or progressive organ damage
Hypertensive Urgency
● Isolated large BP elevations without acute organ
damage

Cardiology Trans 2022: Hypertensive Vascular


Diseases
23. A 63y known hypertensive for 20 years but poorly compliant B Doc’s ratio:
with medications presents with sudden onset of severe downing In order to confirm if the crackles are from
sensation that awakened the patient. This was accompanied by pulmonary edema, request for CXR to see for
diaphoresis. Vital signs showed BP of 210/120, CR 110bpm congestion bat wing -like configurations, or even
regular, RR 32cpm, O2 sat 89% at room air. Crackles were pleural effusion.
noted on both lung fields. You are highly suspecting acute
pulmonary edema. Which diagnostic procedure will help
confirm your impression?
A. 12L ECG
B. Chest X-ray PA
C. 2D Echo-Doppler
D. Troponin I
24. 12L ECG showed 2-3 mm horizontal ST depression in II, III, D Doc’s ratio:
aVF with a 3-4 mm ST elevation from V1-V6, I, aVL. This is If you have a ST depression and ST elevation, then
consistent with: you look at the ST elevation. So that tells you that it
A. Inferior wall ischemia is a transmural injury pattern.
B. Lateral wall AMI Leads V1-V6 so anterolateral, and I, aVL is lateral
C. Anterior wall AMI so the answer is Anterolateral wall AMI.
D. Anterolateral wall AMI
25. The best initial antihypertensive medication to give to our A Doc’s ratio:
patient is: Our patient is congested, acute pulmonary edema,
A. IV Nitroglycerin and IV Furosemide with very high blood pressure so give venodilator to
B. IV Nicardipine and HCTZ tablet reduce preload (nitrates) and it will also dilate your
C. IV Labetalol plus Captopril tablet coronaries and give diuretic to unload.
D. IV Nitroglycerin and IV Labetalol Contested answer: B.

Doc’s explanation
If the patient is in heart failure, without ST elevation,
px is elderly and hypertensive and awakened
because of a severe drowning sensation which
means the patient has acute pulmonary edema,
with coughing and diaphoresis with very much
elevated BP, crackles on both lung field. So, in this
case you give furosemide (a diuretic) but since the
patient has high BP it needs to be given via IV route
and not PO (therefore HCTZ tablet should not be
given). Between a Loop and a thiazide diuretic, the
thiazide diuretic is not a very good diuretic, it is
used more as an anti-HTN medication unlike
Furosemide it truly will diurese the patient, so just
by looking at Furosemide you already know that is it
the answer since you need to give it. Nitroglycerin is
placed also for patients with ACS. Furosemide is
combined with nitroglycerin to act as a preload
unloader.
(Doc’s source is Harrison’s chapter on Hypertensive
Emergency – look for pulmonary edema or heart
MED 2 – Cardiology-Pulmonology – Shifting Exam (16 MARCH 2021) Page 9 of 24

failure) – for this part I only saw the table for the
preferred drugs (see text below)

Preferred Parenteral Drugs for Selected


Hypertensive Emergencies
● Hypertensive encephalopathy – Nitroprusside,
Nicardipine, Labetalol
● Malignant Hypertension (when IV therapy is
indicated) – Labetalol, Nicardipine, Nitroprusside,
Enalaprilat
● Stroke – Nicardipine, Labetalol, Nitroprusside
● MI/ Unstable angina – Nitroglycerin, Nicardipine,
Labetalol, Esmolol
● Acute left ventricular failure – Nitroglycerin,
Enalaprilat, Loop diuretic
● Aortic dissection – Nitroprusside, Esmolol,
Labetalol
● Adrenergic crisis – Phentolamine, Nitroprusside
● Postoperative hypertension – Nitroglycerin,
Nitroprusside, Labetalol, Nicardipine
● Preeclampsia/ eclampsia of pregnancy –
Hydralazine, Labetalol, Nicardipine

Harrison’s 20th edition Table 271-9 p. 1905


26. 2D Echo Doppler showed multiple wall motion abnormalities A Doc’s ratio:
with EF of 35%. The best maintenance antihypertensive In essence, just remember that you do not combine
medication for our patient is: a ACEI/ ARB and a renin blocker.
A. Losartan + HCTZ+ Carvedilol In B, Aliskiren is a renin blocker so you cannot give
B. Perindopril + Aliskiren + Furosemide it with Perindopil
C. Candesartan + Propanolol + Olmesartan In C, there are two ARBs
D. Propanolol + Enalapril + Bisoprolol In D, there are two beta blockers
In this case, you may start the patient with an ACEI
or ARB and a diuretic (thiazide) since, this is now
for maintenance you can now give a beta blocker
but you do not give a beta blocker initially (you have
to wait until the patient is already stable).
27. Drug recommended to be added in resistant HTN D Doc’s ratio:
A. ACE-inhibitors How to know if the patient has resistant HTN?
B. Angiotensin receptor blockers ● If the patient is on triple HTN medications one of
C. Calcium channel blockers which is a diuretic at full dose
D. Spironolactone In this case the patient already has resistant HTN
and according to guidelines, Spironolactone can be
given.
28. Case: A 17yo female consulted because of palpitations and B Doc’s ratio:
shortness of breath. Her history revealed recurrent sore throat In rheumatic mitral stenosis look at which PE will
with progressive easy fatigability over her teenage years. Which point to it and in this case, it will be the Diastolic
is a finding best points to rheumatic mitral stenosis? rumbling murmur at the apex.
A. Apex beat diffuse and sustained at the 6th ICS LAAL
B. Diastolic rumbling murmur at the apex
C. Loud S1, soft S2 at the apex
D. (+) Carvallo’s sign
29. To best document the cause of the palpitations in this patient, B Doc’s ratio
the following is most helpful: Request ECG to know what is causing the
A. 2D Echo – Doppler palpitation
B. 12L ECG
C. Chest radiograph
D. Trans-esophageal echocardiogram
30. What chamber is likely to be involved in an MS patient in sinus A Doc’s ratio
rhythm, presenting with a very strong pulsation over the 4th ICS In this case there is a strong pulsation over the 4th
during diastole. ICS during diastole meaning that this is not from the
A. Left atrium ventricles, this is LA lift, so the answer is Left
B. Left ventricle atrium.
C. Right ventricle
D. Right atrium
MED 2 – Cardiology-Pulmonology – Shifting Exam (16 MARCH 2021) Page 10 of 24

31. An RHD patient with an irregularly irregular rhythm is at risk for A “Valvular heart disease has always been one of the
the following: leading causes of cardioembolic stroke. Atrial
A. Cardioembolic stroke fibrillation associated with mitral stenosis leads to
B. Infective endocarditis left atrial clot and forming thrombo-embolic
C. Myocardial infarction condition.”
D. Pulmonary embolism
Prajapati, P.S. (2015). Rheumatic heart disease as
a cause of cardioembolic stroke (young stroke).
Health Care: Current Reviews 2015, 3:3
(http://dx.doi.org/10.4172/2375-4273.C1.012)

People living with rheumatic heart disease (RHD)


can be at risk for ischemic and hemorrhagic stroke.
Ischemic stroke can occur when clots form in the
heart – from heart failure, around damaged heart
valves, or from heart valve infections. Clots can
also form in the heart when it is pumping in an
abnormal rhythm. People with RHD are at risk of
AF.
People with RHD are also at risk of hemorrhagic
stroke as a result of medication some people need
following a heart surgery. When mechanical heart
valve replacements are used during the surgery,
clots can form around the valve – blood thinning
medications is used to prevent this. However, with
close monitoring, there is a risk of hemorrhagic
stroke from bleeding into the brain.

Rheumatic Heart Disease and Stroke


(https://rhdaction.org/sites/default/files/Rheumatic%
20Heart%20Disease%20and%20Stroke_Brief.pdf)
32. The drug of choice for secondary prevention of rheumatic fever B Best antibiotic for secondary prophylaxis:
in a patient who is allergic to penicillin. Benzathine Penicillin G (1.2 million units or
A. Cotrimoxazole 600,000 units if <27kg) q 4 weeks
B. Erythromycin Erythromycin (250mg) BID for Penicillin-allergic
C. Amoxycillin patients
D. Quinolones
Cardiology Trans 2022:CCC1: Case with Dyspnea
(RHD)
33. TRUE regarding the management of STEMI at the emergency C Doc’s ratio:
room: A. Not all patients with MI require oxygen. You
A. Oxygen support is beneficial to all patients only give it if the O2 stat if it is <90% (according
B. Nitrates are the drug of choice in RV failure to guidelines) but usually we maintain the O2
presenting with chest pain stat above 94% so if it is less than that, that is
C. 12-lead ECG should be done within 10 minutes of first when we give oxygen.
medical contact B. Nitrates are NOT given when there is RV failure
D. Morphine may cause tachycardia further increasing C. Request ECG immediately if you’re
oxygen demand suspecting MI in the patient
D. Morphine does NOT cause tachycardia, rather
it may cause slowing of the heart
34. This drug blocks ADP-mediated platelet aggregation: B Aspirin
A. Aspirin ● Irreversible inhibitor of platelet cyclooxygenase
B. Clopidogrel and thereby interferes with platelet activation
C. Ranolazine
D. Streptokinase Clopidogrel
● Block P2Y12 ADP receptor mediated platelet
aggregation

Ranolazine
● Inhibits late inward sodium current (INa) in the
cardiac muscle in voltage-gated sodium channels

Streptokinase: fibrinolytic

Cardiology Transes 2022: Ischemic Heart Disease


Acute Coronary Syndrome & Prinzmetal Angina
MED 2 – Cardiology-Pulmonology – Shifting Exam (16 MARCH 2021) Page 11 of 24

35. Target systolic BP as part of long-term therapy in STEMI C STEMI Long Term Therapy
patients: ● Key lifestyle interventions include cessation of
A. <120 mmHg smoking, optimal BP control, diet advice &
B. <130 mmHg weight control, and encouraging physical activity
C. <140 mmHg ● Antihypertensive medication with SBP target
D. <150 mmHg of <140mmHg

Cardiology Trans 2022: Case with Chest Pain


36. In a patient with possible unstable angina/NSTEMI with an D Doc’s ratio:
initially inconclusive ECG result, what should be done next? For patients whom you’re suspecting to have ACS
A. Immediate thrombolysis but the ECG is inconclusive, you do serial ECG.
B. Follow-up on OPD A. You do not do thrombolysis immediately
C. Give patient sedative B. You do not simply do a follow up (take note:
D. Do serial ECG follow-up OPD is usually after 1 month)
C. You do not just simply sedate
37. Not all patients with widely split S2 have ASD. Which of the D Doc’s ratio:
following will make you think, it is due to ASD? Look for sign of pulmonic stenosis murmur:
A. The presence of a holosystolic murmur at the apex. crescendo decrescendo murmur over the pulmonic
B. The presence of a holosystolic murmur at the left area (2nd LICS)
parasternal area.
C. The presence of a crescendo decrescendo murmur at
the 2nd right ICS
D. The presence of a crescendo decrescendo murmur at
the 2nd left ICS
38. A 2D Echocardiography with color doppler studies confirms B Doc’s ratio:
your suspicion of ASD. It also reported that on color doppler, Normal flow: Left to right
the flow is from the right atrium to the left atrium. This Since the flow is already from right to left, you know
means that: that the right-sided pressure is already very high
A. The pressure on the left side of the heard is still
greater than that on the right
B. The pressure on the right side of the heart already
exceeded that on the left.
C. The pressures on both sides of the heart are equal.
D. The flow of blood to the pulmonic circulation is greater
than the flow to the systemic circulation.
39. The management that will give the best prognosis is: B Doc’s ratio:
A. ACEI and spironolactone Best intervention is to close the shunt because it
B. Intervention to close the defect will not close by itself anymore
C. Digoxin and furosemide
D. Spontaneous closure of defect
40. This is an irreversible inhibitor of platelet cyclooxygenase and A Aspirin
thereby interferes with platelet activation: ● Irreversible inhibitor of platelet
A. Aspirin cyclooxygenase and thereby interferes with
B. Clopidogrel platelet activation
C. Ranolazine
D. Diltiazem Clopidogrel
● Block P2Y12 ADP receptor mediated platelet
aggregation

Ranolazine
● Inhibits late inward sodium current (INa) in the
cardiac muscle in voltage-gated sodium channels

Diltiazem: NDHP CCB

Cardiology Trans 2022: Ischemic Heart Disease


MED 2 – Cardiology-Pulmonology – Shifting Exam (16 MARCH 2021) Page 12 of 24

41. A relative contraindication to beta-blocker therapy in the C Doc’s ratio:


treatment of ischemic heart disease: A. Pneumonia – not a contraindication
A. Pneumonia B. Tachycardia – indication
B. Tachycardia C. AV Block – contraindication
C. AV block D. Aortic aneurysm – we actually give beta
D. Aortic aneurysm blockers to patients with this condition

Contraindications to beta-blockers:
● Heart blocks
● COPD patients (can go into bronchospasm)
● PR interval >0.24s
● Heart rate <50bpm
● Systolic pressure <90mmHg
● Shock
● LV failure
● Severe reactive airway disease

Cardiology Transes 2022: Ischemic Heart Disease


Acute Coronary Syndrome & Prinzmetal Angina
42. Which of the following is a modifiable risk factor for cardiac A Modifiable risk factors:
disease? ● Smoking
A. Smoking ● Alcohol intake
B. Age ● Traditional CV risk factors
C. Male Gender ● Co-morbidities and well-known syndromes
D. Parents with HTN Non-modifiable risk factors:
● Age
● Sex
● Race
● Family history

Cardiology Trans 2022: Cardiac Diagnosis (page 1)


43. EG, an 80y hypertensive male smoker with diabetes presents C Doc’s ratio:
with calf pain while climbing stairs. The ABI is 0.75 in the left The case is Peripheral Arterial Disease (PAD) and
lower extremity and 0.8 in the right lower extremity. This is a the routine diagnostic test to be requested is
routine diagnostic test that should be requested for EG Arterial Duplex Scan.
A. CT angiography
B. Fluoroscopic angiography The ratio of the ankle and brachial artery pressures
C. Arterial Duplex scan (termed the ankle:brachial index, or ABI) is 1.00-
D. MR angiography 1.40 in normal individuals. ABI values of 0.91-0.99
are considered “borderline”, and those <0.90 are
abnormal and diagnostic of PAD.

Harrison’s Principles of Internal Medicine, 20th ed.,


p.1923
44. EG, an 80y hypertensive male smoker with diabetes presents D The primary sites of involvement are the abdominal
with calf pain while climbing stairs. The ABI is 0.75 in the left aorta and iliac arteries (30% of symptomatic
lower extremity and 0.8 in the right lower extremity. In 80-90% patients), the femoral and popliteal arteries (80-
of these cases, the primary site/s of arterial involvement 90% of patients), and the more distal vessels,
A. External & internal Iliac including the tibial and peroneal arteries (40-50% of
B. Tibial and peroneal patients).
C. Abdominal aorta
D. Femoral and popliteal Harrison’s Principles of Internal Medicine, 20th ed.,
p.1923
45. EG, an 80y hypertensive male smoker with diabetes presents C Doc’s ratio:
with calf pain while climbing stairs. The ABI is 0.75 in the left We give statins (to reduce the risk of myocardial
lower extremity and 0.8 in the right lower extremity. The infarction, stroke, and death), ACEI/ARB (since our
pharmacologic agent that has NO benefit for EG and may even patient has both hypertension and PAD), and
cause harm is Cilostazol (it can improve the time to claudication).
A. Atorvastatin
B. Cilostazol The anticoagulant warfarin is as effective as
C. Warfarin antiplatelet therapy in preventing adverse
D. Ramipril cardiovascular events but causes more major
bleeding, therefore it is not indicated to improve
outcomes in patients with chronic PAD.

Harrison’s Principles of Internal Medicine, 20th ed.,


p.1924
MED 2 – Cardiology-Pulmonology – Shifting Exam (16 MARCH 2021) Page 13 of 24

46. BG an 80y male hypertensive and a heavy smoker was rushed C Doc’s ratio:
the hospital because of sudden shortness of breath. He The patient has an aortic dissection involving the
complained of severe chest and back pain an hour prior to ascending aorta that is why the grade 3/6 diastolic
consult. PE revealed cold clammy skin, orthopnea, a BP of murmur comes from aortic regurgitation.
100/60, PR of 120/m and RR of 28/m. Coarse crackles are
heard over both lung fields. He has a 3/6 diastolic murmur in
the 3rd left and 2nd right ICS parasternal line. What acute
valvular abnormality happened to BG?
A. MS
B. AS
C. AR
D. MR
47. The aortic dissection was determined to be type A after CT D Stanford classification of aortic dissections:
aortogram. This means that the following is involved? ● Type A – ascending aorta
A. Infrarenal ● Type B – transverse and/or descending aorta
B. Thoracic DeBakey classification:
C. Suprarenal ● Type I – ascending to descending aorta
D. Ascending ● Type II – ascending or transverse aorta
● Type III – descending aorta only

Harrison’s Principles of Internal Medicine, 20th ed.,


p.1920
48. What is the definitive management for BG? C Doc’s ratio:
A. Beta blockers and nitrates For most dissections, we can do conservative
B. PCI management, maximizing medical management. An
C. Aortic surgery exception would be when dealing with an ascending
D. Trans catheter aortic valve replacement dissection (Type A). The definitive management
would be to do an immediate aortic surgery.
49. The characteristic peripheral arterial pulse in aortic stenosis is D Doc’s ratio:
A. Bisferiens The peripheral arterial pulse in aortic stenosis is
B. Alternans parvus et tardus (carotid arterial pulse rises slowly
C. Corrigan to a delayed peak).
D. Parvus et tardus
Chronic severe / advanced AR:
● Corrigan’s or water-hammer pulse – the carotid
upstroke has a sharp rise and rapid fall-off
● Bifid or bisferiens pulse – two systolic peaks are
appreciated
Pericardial effusion / tamponade:
● Pulsus alternans – pulse waves alternating
between those of greater and lesser amplitude
with successive beats

Harrison’s Principles of Internal Medicine, 20th ed.,


pp. 512 and 1670
50. The diagnostic modality which will provide the most information C Doc’s ratio:
in aortic stenosis. To diagnose aortic stenosis, request for
A. Chest X-ray PA-Lateral echocardiography with doppler.
B. 12L Electrocardiogram
C. Echocardiography w/ Doppler Echocardiography is useful for identifying coexisting
D. Coronary angiography valvular abnormalities, differentiating valvular AS
from other forms of LV outflow obstruction, and
measuring the aortic root and proximal ascending
aortic dissections.

Harrison’s Principles of Internal Medicine, 20th ed.,


p.1805
MED 2 – Cardiology-Pulmonology – Shifting Exam (16 MARCH 2021) Page 14 of 24

PULMONOLOGY
QUESTION ANSWER RATIONALE
1. A 20 year old patient has 3 weeks low grade fever, cough, and The patient presents with an infection due to the
progressive dyspnea. Physical exam: Lagging of the left, decreased presence of the fever cough and dyspnea
tactile and vocal fremiti, dullness, and decreased breath sounds
from T7 down on the left. What is expected on chest x-ray? A homogenous density with a curved upper border
a. Midline structures shifted to the left would suggest pleural effusion (no air at all)
B
b. Homogenous density with curved upper border
c. Visible visceral pleural line with no bronchovascular Dr. Visperas (Review of Answers)
markings
d. Inhomogenous density with air bronchograms with a (-)
silhouette sign
2. How do we determine if the chest xray is properly penetrated? Penetration
a. The right hemidiaphragm is at the level of the 10th rib • How black or how white the film is
b. The right descending pulmonary artery is easily delineated • You should just be able to see the
c. The lungs show bronchovascular markings only on the vertebral bodies behind the level of heart
mid 2/3, but none in the periphery • Over-penetrated
d. The vertebra are just visible behind the heart shadow o See vertebral bodies CLEARLY
D behind heart
o Vertebral bodies are seen at level
of hemidiaphragm
• Underpenetrated
o No longer see vertebral bodies
behind the heart
Chest imaging Batch 2022 Trans page 1
3. A 60-year-old retired fireman consults for chronic cough, with At first the blood-streaked sputum, chronic cough
blood-streaked sputum and significant weight loss. his chest x-ray and the weight loss would point to TB or lung cancer
reveals an S sign of golden on the right. What PE findings will be The S sign of golden would point that there is a
most associated with the x-ray result? mass or atelectasis on the area.
a. Lagging on the right upper chest The atelectasis would be interpreted as a fissure of
b. Increased tactile fremiti on the right interscapular area the lower border of the lung being pulled up
c. Egophony on the right anterior chest, at the level of 3rd - The mass would be interpreted as an obstruction of
5th ICS A the right upper lobe bronchus
d. Decreased breath sounds on the right base The increased tactile fremitus and the egophony
would need a patent airway which would not be seen
on this patient
Decreased breath sounds on the base is not
possible since there is no x-ray finding that would
point to such
Chest imaging Batch 2022 Trans page 9
4. A 30-year-old chronic heavy smoker consulted because of How to look at a spirometry finding for asthma?
dyspnea for the past year. Two of his siblings has asthma and his
father was diagnosed with COPD at the age of 45. Which of the Check if there is a significant response to a
following spirometric features would point to ASTHMA as the cause bronchodilator:
of dyspnea?
a. Change from pre to post-bronchodilator FEV1 is 220mL. A 12% change from pre to post BD
Percent change is 15% 200mL criteria
b. Post bronchodilator FEV1/FVC ratio is below the lower
limit of normality
c. Post bronchodilator FEV1 is 50% of the predicted value Spirometry Application Batch 2022 Trans (Feb 18 2021), p. 2
d. Pre-bronchodilator FVC is below the limit of normality
5. Which of the following is the expected appearance of the flow A patient with kyphoscoliosis would be expected to
volume loop of a patient with kyphoscoliosis? have a restrictive ventilatory defect and a low FVC
a. Exaggerated concavity of the expiratory limb The volume would be affected while the flow would
C
b. Flattened expiratory and inspiratory limb be normal. It would resemble a witch's hat since the
c. Narrow loop is narrow
d. Short Spirometry Batch 2022 Trans, p. 6
6. A 45-year-old female consulted because of shortness of breath. A patient with COPD would present with a
Her spirometry test showed the following obstructive ventilatory defect and a low FEV1/FVC
LLN ACTUAL ACTUAL % change
PRE POST It is not asthma since it does not fulfill:
FEV/FVC 63 57 65 12% change from pre to post BD
FVC 1.98 2.44 2.47 1 200mL criteria
B
FEV1 1.22 1.39 1.52 10
What is the likely diagnosis? It is not PTB since it should show and restrictive
a. Asthma ventilatory defect and a low FVC
b. COPD
c. PTB
Spirometry Batch 2022 September 10 2020 page 1
d. Heart Failure
MED 2 – Cardiology-Pulmonology – Shifting Exam (16 MARCH 2021) Page 15 of 24

7.What is the acid base interpretation of the ABG with a pH: 7.47, Looking at the ABG flow chart given by Dr. Moral
paCO2: 43, paO2: 186 and a HCO3: 32? first look at the pH, this would be interpreted as
a. Fully compensated respiratory alkalosis alkalosis, Next look at the paCO2, since in between
b. Fully compensated metabolic alkalosis 35-45 this would now be interpreted as
c. Uncompensated respiratory alkalosis uncompensated metabolic alkalosis
d. Uncompensated metabolic alkalosis

WS2 CRC2 CXR & ABG Batch 2022 Trans (February 18 2021)
Appendix
8. How many liters per minute of oxygen given via nasal cannula Flow Rate L/min FiO2
are needed for an FiO2 of 28%? 1 0.24
a. 1 2 0.28
b. 2 3 0.32
c. 3 4 0.36
d. 4 5 0.40
B 6 0.44
Or if you do not want to memorize the table you can
compute it
FiO2 = 20 + (liters per minute x 4)
ABG Made Easy Anesthesiology Lecture Slides
Workshop on ABG September 13, 2018 Batch 2020 p. 1
9. What is the mechanism of hypoxemia in a patient whose ABG Mechanism of hypoxemia
shows pH: 7.30, PaCO2: 30, PaO2:55, HCO3: 17 at 100% FiO2,
P(A-a)O2 of 675?
a. Hypoventilaation
b. Low PiO2
c. Shunt
d. V/Q mismatch

In this case, the patient has a low paCO2 and the


paO2 is increased then the patient was given
oxygen (100% FiO2) but the CO2 did not improve
hence it’s a shunt
WS2 CRC2 CXR & ABG Batch 2022 February 18 2021
Appendix
10. What is the index of oxygenation that does not need to be a/AO2: advantage of not being affected by age. No
adjusted for a 65 year old patient? adjustments needed

a. a/AO2 Normal Values


b. (A-a)O2 (A-a)O2 if ≤ 30 < 15
c. PaO2 (A-a)O2 if > 30 15 + (3 x decades after 30)
d. PaO2/FiO2 A
PaO2 if < 60 y.o. 80 - 100
PaO2 if > 60 y.o. 80 – years above 60

PaO2/FiO2 if < 60 y.o. 400 - 500


PaO2/FiO2 if > 60 y.o. 400 – (years above 60 x 5)
Arterial Blood Gas Sept. 2018 Trans p. 5
MED 2 – Cardiology-Pulmonology – Shifting Exam (16 MARCH 2021) Page 16 of 24

11. A 70 year old female presented with 3 day history of cough, Patients who are immunocompromised, elderly, on
anorexia and malaise. What typical symptoms for CAP are not immunosuppressant drugs may not have fever. Their
usually found in the elderly immune system may not be competent enough to
a. Difficulty of breathing B mount an immune response to an infection.
b. Fever
c. Changes in mental status
d. Chest pain TBL 2 Dyspnea Integration March 2021 Trans p. 4
12. A 70 year old female presented with 3 day history of cough, Pneumonia
anorexia and malaise. On PE, she was stretcher-borne, with use of Symptoms
accessory muscles of respiration, speaks in phrases. She was TRIAD: Fever, Cough, Dyspnea
tachypneic, BP 100/70 mmHg, CR 100/min, febrile at 38 °C. Signs
Crackles were apparent over the right anterior and posterior chest. Tachypnea, Crackles, Evidence of consolidation
Which diagnostic tests should be prioritized? (increased tactile fremitus, dullness, increased
a. Anaerobic culture, Chest CT scan, bronchoscopy breath sounds)
b. BUN, Serum Na, CBG
c. Sputum GS/CS, CXR, ABG C Since pneumonia is highly suspected and with the
d. UAT, CXR, ESR patient is in respiratory distress, requesting for the
following will help in the diagnosis and management:
Sputum GS/CS – specific etiologic organism
CXR – infiltrates, air bronchograms, extent and
location of infection
ABG – acid base balance, oxygenation status and
ventilation status
Community Acquired Pneumonia Sept. 2020 Trans p. 1
13. A 70 year old female presented with 3 day history of cough, CRB 65
anorexia and malaise. On PE, she was stretcher-borne, with use of Scores 1 point for each:
accessory muscles of respiration, GCS 8 (E2,V2,M4). She was • Confusion
tachypneic with RR 35/min, BP 100/70 mmHg, CR 100/min, febrile • Respiratory Rate > 30/min
at 38 °C. Crackles were apparent over the right anterior and • Blood pressure
posterior chest. Using CRB 65, define the severity and o Systolic BP <90 mmHg
recommended site of care for this patient? o Diastolic BP < 60 mmHg
a. Low risk, Out patient • Age >65
b. Moderate pneumonia, in patient hospitalization
c. Moderate pneumonia, closely supervised out-patient Higher score = higher risk of mortality
treatment
d. Severe pneumonia, hospitalize and consider admitting at SCORE RISK RECOMMENDATION
D 0 Very Low Usually does not
ICU
Risk require hospitalization
1-2 Increased Consider
Risk hospitalization
3-4 High Risk Urgent hospitalization
of Death
The patient has a score of 3:
Confusion due to GCS8 E2V2M4
RR 35/min
Age 70

Community Acquired Pneumonia Batch 2022 Trans p. 3


14. A 70 year old female presented with 3 day history of cough,
anorexia and malaise. On PE, she was stretcher-borne, with use of
accessory muscles of respiration, speaks in phrases. She was
tachypneic, BP 100/70 mmHg, CR 100/min, febrile at 38 °C.
Crackles were apparent over the right anterior and posterior chest.
On Past Medical History, patient has been admitted a 2 months ago
and was given IV antibiotics. Which of the following medications
should be given empirically?
a. Cefotaxime + Azithromycin
b. Ceftaroline + Clarithromycin C
c. Cefepime + Levofloxacin
d. Vancomycin + Ciprofloxacin

Patients with antibiotic use in the past 3 months


should receive a respiratory fluoroquinolone such as
levofloxacin

Harrison’s Principles of Internal Medicine, p. 913


MED 2 – Cardiology-Pulmonology – Shifting Exam (16 MARCH 2021) Page 17 of 24

15. Which of the following may be reasons for clinical failure in the Factors to consider in nonresponding
management of CAP, EXCEPT? pneumonia or failure to improve:
a. Bronchial obstruction to cause postobstructive pneumonia
b. Concomitant treatment with glucocorticosteroids 1. Incorrect diagnosis of a complicating non-
c. New diagnosis of HIV infection
infectious condition (e.g. pulmonary embolism,
d. Progression of initial infection to empyema
CHF, MI)
2. A resistant microorganism or an unexpected
pathogen that is not covered by the antibiotic
choice
3. Antibiotic is ineffective or causing an allergic
reaction (e.g. poor absorption of oral antibiotic,
certain drug interactions, inadequate dose,
patient not taking or receiving the prescribed
antibiotic).
4. Impaired local or systemic host defenses (e.g.
B aspiration, endobronchial obstruction,
bronchiectasis, systemic immune deficiency)
5. Local or distant complications of pneumonia
(e.g. parapneumonic effusion, empyema, lung
abscess, ARDS, metastatic infection,
endocarditis)
6. Overwhelming infection
7. Slow response in the elderly patient: S.
pneumoniae and L. pneumophila may cause
slow resolution of pneumonia in the elderly
8. Exacerbation of co-morbid illnesses
9. Nosocomial infection

A and C – explained by 4
D – explained by 5 and needs further investigation

Community Acquired Pneumonia Sept. 2020 Trans p. 6


16. What is an indication for non-invasive ventilation? Patient Inclusion Criteria
a. Cardiac arrest
b. Glasgow Coma Scale of 4 • Patient cooperation - Essential component
c. Poor cough reflex Excludes: agitated, belligerent, and comatose
d. Pulmonary congestion • Dyspnea
Moderate to severe, but short of respiratory
failure
• Tachypnea 24 breaths/min
• Increased Work of Breathing
Accessory muscle use
Pursed lip breathing
D • Hypercapnic respiratory acidosis
pH:7.10 - 7.35
• Hypoxemia
PaO2/FiO2 < 200mmHg
Best: rapidly reversible cause of hypoxemia
Absolute Contraindications
• Coma
• Cardiac Arrest
• Respiratory Arrest
• Any condition requiring immediate intubation

ER 1: Acute Respiratory Failure Sept. 2019 Trans p. 2


17. Which physical examination finding indicates the need for B is a sign of with respiratory depression
ventilatory support? A, C, D are signs of respiratory distress but do not
a. Alar flaring always indicate intubation or ventilatory support
b. Respiratory rate of 6 cycles per minute B
c. Expiratory wheezes
d. Coarse crackles
Dr. Visperas (Review of Answers)
MED 2 – Cardiology-Pulmonology – Shifting Exam (16 MARCH 2021) Page 18 of 24

18. Which finding points to respiratory distress? Signs of respiratory failure:


a. Abdominal paradox Abdominal Paradox
b. Altered sensorium Central Cyanosis
c. Cyanosis of the nail beds Altered Sensorium
d. Increased respiratory rate D *** pakicheck, kasi cyanosis of the nail beds is
peripheral cyanosis, I don’t know why di sya kasama
since this is a “choose any that applies” type of
question
Dr. Visperas (Review of Answers)
19. A 22-year-old female student presents with episodic dyspnea, Since GINA score is 3, and there were no other
chest tightness, and cough. You decided to treat her at home. What information regarding previous medications, then at
would be the preferred medication to give? least a Step 2 medication is given
a. Low-dose LABA+ICS as needed Dr. Visperas (Review of Answers)
b. High dose LABA+ICS every 12 hours
c. Low-dose LABA+ICS every 12 hours
d. Maintenance ICS every 12 hours

20. What combination of inhaled drugs is utilized for MART Maintenance and reliever therapy (MART) is
therapy? combined ICS and LABA treatment (preferably fast-
a. Low-dose Formoterol+Budesonide acting LABA) e.g., Budesonide+Formoterol
A
b. Low-dose Salmeterol+Fluticasone
c. Low-dose Formoterol+Tiotropium TBL 1 Bronchial Asthma Batch 2022 Trans, p. 8
d. Low-dose+Beclomethasone+Tiotropium
21. In this time of Covid-19 pandemic a 17-year-old asthmatic was Patient presents with an asthma exacerbation.
rushed to the ER. Patient is in tripod position with use of accessory During the pandemic, nebulization is not done since
muscles. You have started oxygen inhalation. What should be your the patient’s Covid status in not yet confirmed.
next step in managing the patient? Instead, Salbutamol (SABA) puffs via a spacer is
a. Salbutamol via nebulizer now then repeat after 20 mins given. LABA and LAMA are not given in the spacer/
b. Salbutamol 4 puffs via a spacer then repeat after 20 B nebulizer form.
minutes
c. Salmeterol+Fluticasone 4 puffs via a spacer then repeat
after 20 minutes Dr. Visperas (Review of Answers)
d. Salbutamol+Tiotropium via a nebulizer now then repeat
after 20 minutes
22. Variable airway obstruction in asthma is assessed by requesting Spirometry is a respiratory function test that
for: measures the ventilatory capacity of the respiratory
a. Spirometry system (ability to bring a volume of air in and out of
A
b. FeNO the lungs).
c. Sputum eosinophil count
d. MMRC dyspnea score Spirometry Batch 2022 Trans, p. 2
23. A 23-year-old male from Caloocan City was recently diagnosed We want to determine the risk of the patient prior to
with bacteriologically confirmed PTB. Which of the following tests initiating treatment. For the other choices, you may
should be done prior to initiation of treatment? forego with the other screening tests.
a. CBC with platelets B
b. HIV screening
c. Serum Creatinine Dr. Visperas (Review of Answers)
d. Serum Uric Acid
24. Which of the following regimens is recommended for new cases
of bacteriologically confirmed TB?
a. Ethambutol, pyrazinamide, streptomycin, rifampicin
b. Amikacin, isoniazid, pyrazinamide, streptomycin
c. Isoniazid, rifampicin, ethambutol, pyrazinamide
d. Ciprofloxacin, isoniazid, streptomycin, ethambutol
C

Patient is classified as Category I.


Tuberculosis Batch 2022 Trans, p. 2
MED 2 – Cardiology-Pulmonology – Shifting Exam (16 MARCH 2021) Page 19 of 24

25. 38-year-old male with chronic cough, minimal hemoptysis, Refer to the table above (#24). Patient is classified
weight loss and night sweats. His sputum AFB was positice. as Category II.
Sputum GeneXpert is positive and negative for RiF resistance. He
was previously treated for in 2014. What is your option for this
patient using the treatment regimen discussed in the lecture and the
C
workbook?
a. 2HRZE/4HRZE
b. 2HRZE/4HR
c. 2HRZE/1HRZE/5HR
d. 2HR/4HR
26. After 3 weeks on anti-TB meds, the 55-year old male diabetic Optic neuritis is due to Ethambutol toxicity.
developed eye pain and blurring of vision. Which of the following is
the likely culprit? Harrison’s Principles of Internal Medicine 20th Ed., p. 1252
a. Isoniazid D
b. Rifampicin
c. Pyrazinamide
d. Ethambutol
27. A 65-year-old male, hypertensive and with history of gout. He Serum uric acid testing is not recommended. It can
will be treated for TB. Which of the following should be included in be included if the patient presents with previous risk
his screening tests? factors, and history of gout.
a. Request for HBA1c C
b. Request for liver function test
c. Request for BUA Tuberculosis Batch 2022 trans, p. 2
d. No need to request additional tests
28. 38 y/o male, came in the ER because of repeated episodes of Patient had previous TB treatment, there may be
minimal hemoptysis. He is otherwise asymptomatic. He had already structural lung damage due to TB and may
previous TB treatment 5 years ago. The bleeding is likely secondary be correlated with bronchiectasis. Thus, patient have
to? minimal bouts of hemoptysis.
B
a. Pneumonia
b. Structural lung damage
Dr. Visperas (Review of Answers)
c. Lung malignancy
d. Emphysema
29. A 35-year-old call center agent had hemoptysis of 80mL in 24 Patient is stable (BP was 110/70, CR 98). You need
hours. BP was 110/70, CR 98. Which of the following tools will best to characterize where the bleeding is coming from
aid in the diagnosis? before you undergo to bronchoscopy. Chest CT is
a. Chest Xray the best answer.
b. MRI
c. CBC PC Dr. Visperas (Review of Answers)
D
d. Chest CT Scan Use of early chest CT has been advocated to help
localize the bleeding site and diagnose the cause of
hemoptysis. Requires temporary movement of
patient away from intensive care.

ER2 Hemoptysis Batch 2022 trans, p. 3


30. A 65-year-old female was referred due to 1 week history of Massive PE (Circulatory collapse)
unilateral leg swelling and acute dyspnea. Which physical Acute PE with:
examination finding would make one suspect massive PE? • Sustained hypotension = SBP <90 mmHg
a. Bradycardia CR <60/minute for at least 15 minutes or requiring
b. Tachycardia CR >100/minute inotropic support.
c. Pulselessness C • Pulselessness
d. SBP = 90 mmHg on inotropic support • Persistent Profound Bradycardia = Heart
rate <40 bpm with signs or symptoms of
shock
ER3 Massive PE Batch 2020 trans, p. 1
MED 2 – Cardiology-Pulmonology – Shifting Exam (16 MARCH 2021) Page 20 of 24

31. A 65-year-old female was referred due to 1 week history of First line drug: Heparin Anticoagulation
unilateral leg swelling and acute dyspnea. Venous duplex scan Gold standard: Recombinant tPA = Alteplase
showed findings compatible with DVT. VQ scan showed unmatched
perfusion to ventilation. While transporting patient back at
emergency room, patient had palpatory BP at 60 mmHg and was
given 4 vasopressors without any change in BP. Which treatment
should be given?
a. Aspirin and Urokinase
b. Heparin drip and Alteplase
c. Sulodexide and Apixaban
d. Tinzaparin and Dabigatran

ER3 Massive PE Batch 2020 trans, p. 2


32. A patient has been experiencing fever, cough, and dyspnea for Do not do thoracentesis when there are multiple
the past 2 weeks. She has a chest x-ray done and it shows pleural loculations seen since you cannot drain all
effusion on the left. On Ultrasound, there were septations noted. loculations. Refer to TCVS
What is the next best option?
a. Aspirate a sample from one of the locules and send them B
to the Lab. Pleural Diseases Batch 2022 trans, p. 4
b. Refer to TCVS
c. Insert a catheter and instill a sclerosing agent
d. Increase the dose of the antibiotics and shift to IV route.
33. A 65-year-old chronic smoker consults because of cough, Effusion may be secondary to malignancy/metastatic
significant weight loss, and progressive shortness of breath for the disease (chronic smoker and weight loss). To
past 2 months. On PE, you note that he has clubbed fingers. Chest investigate for malignancy, best diagnostic test
x-ray reveals pleural effusion on the right. What examinations will would be cytology of pleural fluid.
best confirm the most probable etiology of the fluid? C
a. pH Dr. Visperas (Review of Answers)
Pleural Diseases Batch 2022 trans, p. 2
b. Differential count
c. Cytology
d. Gene Xpert
34. A patient is undergoing thoracentesis. The doctor has already Re-expansion pulmonary edema is a manifestation
aspirated around 1200 cc of fluid when the patient starts of aspirating large amounts of fluid. B lines is a sign
complaining of increasing dyspnea. A re-expansion pulmonary of Pulmonary edema.
edema is highly suspected. What do we expect on ultrasound? Dr. Visperas (Review of Answers)
a. Numerous B lines A
b. Absence of lung sliding Numerous B lines is an implication of too much fluid
c. Bar code sign found in the interstitium.
d. Hepatization
Chest Imaging Batch 2022 trans, p. 12
35. What finding points to a complicated parapneumonic effusion? Factors of a Complicated Parapneumonic
a. Foul smelling aspirate Effusion:
b. pH < 7.2 • Loculated pleural fluid
c. Pleural fluid protein : serum protein >0.5 • pH < 7 (<7.2 in the book)
d. Turbid appearance • Glucose < 3.3 mmol/L (<60mg/dL)
B
• Positive Gram stain or culture of the pleural
fluid
• Presence of gross pus in the pleural
space.
Pleural Diseases Batch 2022 Trans, p. 2
MED 2 – Cardiology-Pulmonology – Shifting Exam (16 MARCH 2021) Page 21 of 24

36. What is the most appropriate treatment for a sleep apnea Take note that the patient is obese, with more than
patient with a BMI of 32, diabetes mellitus and apnea hypopnea +5 hypopnea index that’s why do CPAP and weight
index of 20/hour? loss via lifestyle change and pharmacotherapy
a. Oral/mandibular appliance and weight reduction via
lifestyle change
C
b. CPAP and weight loss via lifestyle change
c. CPAP and weight loss via lifestyle change and
pharmacotherapy
d. CPAP and weight loss via lifestyle change,
Dr. Visperas (Review of Answers)
pharmacotherapy and bariatric surgery
37. What sleep stage are apneas most prominent because REM and Breathing
ventilatory response to increasing carbon dioxide levels are • Greatest likelihood that you will have sleep
transiently abolished? apnea.
a. N1 • Characterized by a low-amplitude, mixed
b. N2 frequency EEG similar to that of N1
c. N3 • EOG tends to occur in flurries or bursts
d. R • EMG activity is absent in nearly all skeletal
muscles except those involved in
respiration = reflects brainstem-mediated
muscle paralysis (characteristic of REM)
• More significant because there’s more
disturbance that can happen in REM sleep
D • Irregular respiratory drive
• Ventilatory response to chemical and
mechanical stimuli may be transiently
reduced or completely abolished even
with increase PaCo2, you may not
increase breathing efforts.
• Short periods of central apnea occur
• Decreased intercostal and accessory
muscle activity
• Generalized inhibition of skeletal muscle
tone including pharyngeal muscles
• Decreased thoraco-abdominal coupling

Sleep Disordered Breathing Batch 2022 Trans, p. 2


38. Which characteristic in a male patient is a risk factor for sleep STOP BANG
apnea? • Snoring
a. BMI of 20 • Tiredness when waking up
b. 52 years old • Obstruction noticed to stop breathing or
c. Neck circumference of 16 inches choking episodes at night
d. Epworth sleepiness score of 5 • Pressure - high BP
• BMI (>30)
• Age (>50 y/o)
• Neck circumference – 17 inches for men,
B 16 inches for women
• Gender – male > female

Epworth sleepiness – not diagnostic for sleep apnea


but can help in predicting if patient has SBD.
• Score >10 = excessively somnolent –
caused by lack of sleep; if with other risk
factors may point to sleep apnea.

Sleep Disordered Breathing Batch 2022 Trans, p. 5


39. A 42-year-old female, Filipino, non-smoker consulted because Adenocarcinoma
of incidental finding of a peripheral lung mass suspicious for • Non-smoker
malignancy. Based on the clinical profile, which cell type is likely? • Female
a. Adenocarcinoma A • Peripheral lung mass
b. Mesothelioma
c. Large cell carcinoma
Squamous cell carcinoma Dr. Visperas (Review of Answers)
MED 2 – Cardiology-Pulmonology – Shifting Exam (16 MARCH 2021) Page 22 of 24

40. A 75-year-old male with a lung mass on the right upper lobe Since there’s affectation of vessels presented as
was observed to have facial edema, swelling of the right arm, distended neck veins and chest vein collaterals,
distended neck veins and chest vein collaterals. What is the most Superior Vena Cava Syndrome is highly suspected
likely cause of these findings? and that is in the intrathoracic extension of the
a. Cushing syndrome C tumor.
b. Extra-thoracic metastasis
c. Intrathoracic extension of the tumor
d. Lambert-Eaton syndrome
Dr. Visperas (Review of Answers)
41. A 55-year-old male, previous smoker was admitted because of To investigate the findings of the right upper lobe,
anorexia and weight loss of about 10 kgs for the past 6 months. the best to use is chest CT with contrast since it
Chest x-ray showed a right upper lobe homogenous opacification. didn’t mention about the central signs, eliminate
Which of the following diagnostic modality is recommended at this bronchoscopy as the answer. You have to see first
time? C the map and navigate first to delineate further chest
a. ABG x-ray findings.
b. Bronchoscopy
c. Chest CT scan with contrast Dr. Visperas (Review of Answers)
d. Sputum cytology
42. A 55-year-old male, previous smoker was admitted because of The extent of mediastinal lymph node involvement is
anorexia and weight loss of about 10 kgs for the past 6 months. important in determining the appropriate treatment
Chest x-ray showed a right upper love homogenous opacification. strategy: surgical resection followed by adjuvant
While admitted, patient coughed out 200 ml of blood and underwent chemotherapy versus chemoradiation alone.
fiberoptic bronchoscopy (FOB) with findings of endobronchial mass
consistent with squamous cell lung carcinoma and with Stage IIB. D Stage II B – surgery + adjuvant chemotherapy
Which treatment regimen is suitable to this patient?
a. Chemoradiotherapy
b. Chemoradiotherapy and surgical resection
c. Cisplatin based chemotherapy Lung Cancer Batch 2022 Trans, p. 10
d. Surgical resection
43. A 50 year-old hypertensive male, presented with progressive Dyspnea in COPD is more of the progressive type
shortness of breath. He stopped smoking 10 years ago. On PE, Spirometry Application Batch 2022 Trans, p. 1
diffuse wheezing was noted. Which of the following will likely point
to COPD? GOLD: COPD is a common preventable and
a. 50 years-old treatable disease that is characterized by persistent
b. Diffuse wheezing respiratory symptoms and airflow limitation that is
c. Progressive shortness of breath due to airway and/or alveolar abnormalities usually
d. Hypertension caused by significant exposures to noxious particles
of gases
Key points:
• COPD is preventable and curable
• Airway limitation is usually progressive
• Symptoms are persistent
• Main risk factor is tobacco smoking.
Elemental exposures are contributory
• COPD punctuality periods of the worsening
of symptoms

COPD Batch 2021 Trans, p. 1-2


44. Which of the following is a typical radiographic sign of A tubular or slender or narrow heart is expected in
hyperinflation? patients with hyperinflation or emphysema
a. Tubular heart • Apex beat not at MCL
b. Meniscus sign CRC 2: CXR & ABG Batch 2022 Trans, p. 1
c. Water-bottle heart
d. Narrowed ICS Meniscus sign – signifies pleural effusion
A Water bottle heart – indicates the presence of
pericardial effusion
Narrowed ICS – seen in atelectasis and trapped lung
Chest Imaging Batch 2022 Trans, p. 4
Foley, J., Tong, L. P., & Ramphul, N. (2016). Message in a
bottle. The use of chest radiography for diagnosis of pericardial
effusion. African Journal of Emergency Medicine, 6(3), 148-150.
doi:10.1016/j.afjem.2016.01.004
MED 2 – Cardiology-Pulmonology – Shifting Exam (16 MARCH 2021) Page 23 of 24

45. If you are suspecting COPD in a 65-year-old male, which of the Obstructive lung disease is suggested by a barrel
following PE finding is likely? chest deformity on PE (increased AP diameter)
a. Bronchial breath sounds
b. Increased AP diameter Harrison’s Principles of Internal Medicine, p. 1667
c. Resonant on percussion
d. Asymmetry in chest expansion Asymmetry in chest expansion is seen in the PE
findings of pleural effusion, atelectasis, and
B pneumothorax

Chest Imaging Batch 2022 Trans

Resonant (hyper-resonant) on percussion indicates


pleural effusion

Harrison’s Principles of Internal Medicine, p. 1944


46. Mr. Jun, diagnosed COPD since June 2018. He was maintained
on salmeterol + fluticasone. Though he is compliant, he was
admitted last 2019, 2020, for an exacerbation and pneumonia.
What will be the best option for him?
a. Maintain ICS + LABA
b. Shift to LABA + LAMA
c. Shift to LABA
d. Start oral steroids
B

The patient already had 2 previous exacerbations,


so the best option would be to give a LABA + LAMA
(Group D classification).
Dr. Visperas (Review of Answers)
CCC 1: Exacerbated COPD Batch 2022 Trans, p. 5
47. A 65-year-old female presents with a history of copious sputum Primary Diagnosis of Bronchiectasis:
production. What PE findings on the lungs will make you suspect of
a possible bronchiectasis? Physical Examination
a. Persistent basal crackles A • Persistent basal coarse crackles and
b. Transient basal crackles wheezing on auscultation
c. Expiratory wheeze
d. Inspiratory wheeze Bronchiectasis Batch 2022 Trans, p. 4
48. A 75-year-old male diagnosed to have bronchiectasis on CT For this case, we will not experiment on what drugs
scan and now readmitted for the 4th time in the same year because to give to the patient, instead a referral to a specialist
of an exacerbation (increased sputum volume and purulence). team (infectious diseases & pulmonology) is done.
What is the best way to manage him?
a. Refer to a specialist team A
b. Start anti-fungal medications
c. Treat for multiple drug-resistant tuberculosis
d. Administer the antibiotics following CAP high risk Dr. Visperas (Review of Answers)
guidelines
49. Which is TRUE regarding treatment of deep vein thrombosis • IVC filter is not indicated unless there is active
(DVT): bleeding
a. Primary therapy consists of anticoagulation or placement • Treatment for severe DVT consists of clot
of an inferior vena caval (IVC) filter. dissolution with low-dose catheter-directed
b. Secondary prevention consists of clot dissolution with low- thrombolysis
dose catheter-directed thrombolysis. This approach is • Vascular compression stockings prevent the
reserved for patients with extensive femoral, iliofemoral, or development of postthrombotic syndrome in
upper extremity DVT. symptomatic DVT
C
c. Below-knee graduated compression stockings, usually
with applied pressure of 30-40 mmHg, may be prescribed
for patients with swelling of the legs when acute DVT is
diagnosed to lessen patient discomfort, and should be
replaced every 3 months.
d. Prescription of vascular compression stockings is
asymptomatic newly diagnosed acute DVT patients Dr. Visperas (Review of Answers)
prevents the development of postthrombotic syndrome.
MED 2 – Cardiology-Pulmonology – Shifting Exam (16 MARCH 2021) Page 24 of 24

50. In which o the following patients who present to the emergency Patient presented with a risk factor for DVT
room complaining of acute dyspnea and a elevated/ positive d- (prolonged air travel)
Dimer value, prompt additional testing for pulmonary embolism?
a. A 24-year old woman who is 34 weeks pregnancy
b. A 48-year old man with no past medical history who
B
present with calf pain following a prolonged air travel
c. A 56-year old woman undergoing chemotherapy for breast
cancer
d. A 72-year old man with acute myocardial infarction 2 Dr. Visperas (Review of Answers)
weeks ago

~ GOOD LUCK! ~
MEDICINE 2 (PULMO)

PULMO SHIFTING EXAM (2ND SHIFT)


15 DEC 2020

This ratio only includes the pulmo part of the exam since neither the answer nor the choices were provided for the cardio part. Please use at your own risk and good luck!
QUESTION ANSWER RATIONALE
1. A patient has 5 days fever, cough, and dyspnea. Physical D Triad of pneumonia (fever, cough, dyspnea) points to
exam of the posterior chest is normal but the x-ray shows a possible consolidation on chest radiograph. In order to
“consolidation”. What lobe of the lung is probably involved? differentiate lobar involvement on PA view:
A. Left lower Right middle lobe
B. Lingula Obscured cardiac border
C. Right lower (+) Silhouette sign – signifies a possible middle
lobe pneumonia
D. Right middle
Right lower lobe
Right border of the heart still demarcated
Chest Imaging (Batch 2022) p.7
Community Acquired Pneumonia (Batch 2022) p.1
2. How do we determine if a patient is properly positioned for a B Acceptable positions in a properly taken CXR:
chest x-ray? Clavicle heads
A. The right hemidiaphragm is at the level of the 10th rib Distance to spinous process must be equidistant
B. Both clavicles are equidistant to vertebral spine If not: patient is not centered
C. The apex of the heart is along the midclavicular line Trachea - not as accurate
D. The cupola of both lungs are visible above the clavicle Chest Imaging (Batch 2022) p.1
3. A 60 year old retired fireman has been having progressive D Chronic smoking predisposes the patient to COPD and
dyspnea for the past 5 years. He admits to smoking at least a can manifest with progressive dyspnea. Furthermore,
pack a day since he was in his 20s. What is expected on chest severe airflow obstruction can functionally reduce the
film? compliance of the respiratory system leading to dynamic
A. Blunted costophrenic sulci hyperinflation.
B. Layering >10 mm Normally, the right hemidiaphragm is one ICS higher
than the left because of the liver. Therefore, among the
C. Apex of the heart displaced laterally and downwards
choices, a right and left hemidiaphragm at the same
D. Right hemidiaphragm at level of 11th rib
level (11th rib) indicates hyperinflation.
Chest Imaging (Batch 2022) p.1
Harrison’s 20E p.229
4. A 30-year-old chronic heavy smoker consulted because of B Upon spirometry, a patient with asthma will manifest
dyspnea for the past year. Two of his siblings has asthma and with significant response to bronchodilators, while a
his father was diagnosed with COPD at the age of 45. Which COPD patient does not. A significant response to
of the following spirometric features points to COPD as the bronchodilators must fulfill the criteria that FEV1 shows
cause of dyspnea? both:
A. Change from pre- to post-bronchodilator FEV1 is 220 mL, > 12% increase from pre- to post bronchodilator value
> 200 mL increase from pre- to post bronchodilator
percent change is 15%
If not able to fulfill the previous criteria, check FVC (it
B. Post-bronchodilator FEV1/FVC ratio is below the lower limit of
must also show both).
normality In addition, the normal FEV1/FVC ratio is 70-80% in
C. Post-bronchodilator FEV1 is 50% of predicted value normal adults; thus, a value or LLN less than that may
D. Pre-bronchodilator FVC is below the limit of normality also indicate a possible COPD.
Spirometry (Batch 2022) p. 5,8
5. Which of the following is the expected appearance of the flow C Pulmonary fibrosis is a type of restrictive lung disease.
volume loop of a patient with pulmonary fibrosis? A narrow flow volume loop means that there is a
A. Exaggerated concavity of the expiratory limb restrictive lung disease while a loop with short &
B. Flattened expiratory and inspiratory limb exaggerated concavity can be seen in obstructive lung
C. Narrow disease.
D. Short Spirometry (Batch 2022) p. 6
6. A 42-year-old female consulted because of shortness of A The FEV1/FVC ratio pre-bronchodilator value is lower
breath. Her spirometry test showed the following: than the LLN,
57 (actual) < 63 (LLN)
LLN ACTUAL ACTUAL % This indicates an obstructive ventilatory defect.
(PRE) (POST) change Additionally, there is significant response to
bronchodilator (see criteria in #4).
FEV/FVC 63 57 65 14% > 12%
1610 (POST) - 1390 (PRE) = 220 mL > 200 mL
FVC 1.98 2.44 2.47 1 Thus, the most likely condition is Asthma.
Spirometry (Batch 2022) p. 7,8
FEV1 1.22 1.39 1.61 14
What is the likely diagnosis?

TIME TO TRANS Page 1 of 11


[MED2-CARDIOPULMO] – 2nd Shift Finals (15 DEC 2021) Page 2 of 11

A. Asthma
B. COPD
C. PTB
D. Tracheal Stenosis
7. The value in the ABG that is computed and therefore less D paCO2 is the best and most reliable parameter, while
reliable is: HCO3 is not always reliable.
A. pH Blood gas analyzers do not have the capacity to
B. PaCO2 directly measure bicarbonate; instead, it is calculated
C. PaO2 from measured pH and pCO2, using the Henderson-
D. HCO3 Hasselbalch equation that relates all three
parameters.
Serum total carbon dioxide and gas panel-derived
plasma bicarbonate are often used interchangeably
for clinical purposes. When they disagree, there is a
tendency to accept total carbon dioxide and discredit
gas panel-derived plasma bicarbonate values.
ABG (Batch 2020) p. 2
Goldwasser, P., Manjappa, N. G., Luhrs, C. A., & Barth, R. H.
(2011). Pseudohypobicarbonatemia caused by an endogenous
assay interferent: a new entity. American journal of kidney diseases
8. A 65 year old patient has a gas exchange dysfunction as C
evidenced by this ABG finding: Indices Normal Value
A. PaO2 of 78 at room air
B. a/AO2 of 0.77 PaO2 (> 60 y.o.) 80
C. (A-a)O2 of 60
D. PaO2/FiO2 of 385 a/AO2 > 0.75

(A-a)O2 15 ± 5

PaO2/FiO2 375
(> 60 y.o.) 400 - (# of years above 60
years x 5)
= 400 - 25

The Alveolar-Arterial Oxygen Difference (p[A-a]O2)


reflects the gas exchange function of the lungs.
Normally, it ranges from 10-20 torr, but it becomes wider
(>20) with age because the oxygen is not transferred
well from the alveolar to the arterial system. An (A-a)O2
of 60 is too high, thus implying gas exchange problems.
ABG (Batch 2020) p. 5
9. An ABG would be helpful before deciding to mechanically B Acute respiratory failure requires intubation or
ventilate a patient in this scenario: mechanical ventilation.
A. Abdominal paradox Signs of acute respiratory failure:
B. Accessory muscle use Abdominal paradox
C. Altered sensorium Central cyanosis
D. Circumoral cyanosis Altered sensorium

The use of accessory muscles signifies increased


work of breathing, as in respiratory distress. An ABG
will be helpful in telling us if the symptom is already
part of a progression to ARF. Else, this would only be
part of the patient inclusion criteria for non-invasive
mechanical ventilation (i.e., helmet, full-faced mask).
Pulmo Workbook
Emergency 1: ARF (Batch 2020), p. 2
10. The individual whose hypoxemia would show the most C Rapidly reversible causes of hypoxemia such as high-
improvement with oxygen supplementation: altitude hypoxemia shows the most improvement to
A. Flail chest secondary to trauma oxygen supplementation. At high altitudes, especially
B. Guillain Barre Syndrome when tissue O2 demands are higher during athletic
C. High altitude hypoxemia activities, the marked reduction in the pressure gradient
D. Myasthenia gravis and available oxygen can lead to tissue hypoxia
(hypobaric hypoxia).
The rest of the choices are examples of condition that
may actually predispose an individual to respiratory
failure. GBS and MG can lead to ARF by failure to
ventilate; while flail chest can cause it by failure of gas
flow.
Gallagher, et al. (2019). High altitude illness. In UptoDate.
Emergency 1: ARF (Batch 2020), p. 2
[MED2-CARDIOPULMO] – 2nd Shift Finals (15 DEC 2021) Page 3 of 11

11. A 42 y/o female consulted because of productive cough for 1 C Community Acquired Pneumonia
week accompanied by fever and difficulty of breathing. On Triad Symptoms: Fever, Cough, Dyspnea
physical examination: BP 100/70, RR 26/min, CR 95/min Temp. Physical Examination: Tachypnea, Crackles,
38 degrees Celsius with crackles over the right Posterior T8 Evidence of Consolidation (increased tactile fremitus,
down. Given this clinical scenario what is the most likely dullness, increased breath sounds)
diagnosis? Community Acquired Pneumonia (Batch 2020) p. 1
A. Acute Bronchitis
B. Bronchiectasis
C. CAP
D. PTB
12. A 60 y/o female consulted because of productive cough for 5 D The diagnostic test for CAP is chest x-ray.
days, accompanied by decrease in appetite and fever. At the Chest Radiograph
clinic she had stable vital signs and minimal crackles over the Demonstrable infiltrate by chest x-ray
left base. She has no previous antibiotic intake. What is an Air Bronchograms = consolidation on PE
appropriate test to evaluate for this patient's condition? Good sound transmission on consolidation areas
A. ABG Findings may include risk factors for increased
severity:
B. Sputum AFB Smear
Cavitation
C. Chest ultrasound Multilobar involvement
D. Chest X ray May suggest an etiologic diagnosis
Pneumatoceles – Staphylococcus aureus
Upper-lobe cavitation - Tuberculosis
Community Acquired Pneumonia (Batch 2020) p. 2
13. Which of the following findings which will warrant admission B Based on the flowchart, pleural effusion will stratify the
in a patient with CAP: patient to moderate or high-risk which will warrant
A. CR 100/minute admission either way. The others choices are classified
B. Pleural effusion as low-risk CAP (based on the PCPG Algorithm on Risk
C. SBP 90 mmHg Stratification of CAP).
D. RR 28/min

Community Acquired Pneumonia (Batch 2020) p. 2


14. A 72 year old male presented at the clinic due to 3-day history C Suspect CAP due to the presence of the CAP triad.
of anorexia, weakness, occasional dyspnea and non- CRB 65
productive cough with febrile episodes. He is maintained on British Thoracic Society classification
amlodipine and salmeterol + fluticasone inhaler. GCS 15, BUN was omitted (from CURB 65) because usually
ambulates with assistance, BP 140/70 CR 102/min 30/min T: you don’t have BUN data right away.
38.5 oC, O2 saturation 94% at room air, with crackles Right T7 Higher score = higher risk of mortality
Scores 1 point for each:
down, no cyanosis, no edema. Which of the following should
Confusion
be the next step?
Respiratory Rate > 30/min
A. Give oral antibiotics and ask the patient to follow-up after 1 Blood pressure
week Systolic BP <90 mmHg
B. Do CXR and ask patient to come to you for follow up the next Diastolic BP < 60 mmHg
day Age >65
C. Admit the patient and treat with broad spectrum antibiotics
D. Work up the patient for COPD
0 Very Low Risk.
Usually does not require hospitalization

1-2 Increased Risk. Consider hospitalization.

3-4 High Risk of Death. Urgent hospitalization

Community Acquired Pneumonia (Batch 2020) p. 3


[MED2-CARDIOPULMO] – 2nd Shift Finals (15 DEC 2021) Page 4 of 11

15. In a patient with moderate risk CAP, which of the following will C The patient is classified with Moderate Risk CAP.
most likely be an expected finding and the appropriate Give IV Non pseudomonal B-lactam (BLIC,
treatment? Cephalosporin) + Extended Macrolides or
A. Sputum GS Gram Positive cocci in pairs, give oral Co- Respiratory Fluoroquinolones
amoxiclav and Ciprofloxacin
B. Chest X-ray findings of cephalization bilateral, give
Levofloxacin per IV once a day
C. WBC count elevated with predominance of segmenters, start
Ceftriaxone IV plus azithromycin
D. Sputum Gen Xpert MTB detected, sensitive to RIf; give INH +
Rif + Eth + Pyr tablets

Community Acquired Pneumonia (Batch 2020) p. 3-4


16. ARDS patient will have this presentation: C Acute Respiratory Distress Syndrome
A. Symptoms 2 weeks after clinical insult More severe form of ARF in which the alveoli get filled
B. Findings of fluid overload up with fluid and debris, adding to the dead space.
C. PaO2/FiO2 of 160
D. Unilateral opacities

Emergency 1: ARF (Batch 2020), p. 2


17. This patient is a candidate for non-invasive ventilation: A Patient Inclusion Criteria for Non-Invasive
A. Pulmonary congestion patient with a PaO2/FiO2 of 200 Mechanical Ventilation
B. Stroke patient with altered sensorium Patient cooperation
C. Extremely restless COPD patient Moderate to severe dyspnea, short of ARF
D. Pneumonia patient with PaCO2 of 95 and a pH of 7.0 Tachypnea (> 24 cpm)
Increased work of breathing (accessory muscle use,
pursed lip breathing)
Hypercapnic respiratory acidosis (pH = 7.10 to 7.35)
Hypoxemia (PaO2/FiO2 < 200 mmHg)
Emergency 1: ARF (Batch 2020), p. 2
18. The management most likely to improve the hypoxemia in B Positive End Expiratory Pressure (PEEP) prevents
ARDS is: the alveoli from collapsing. Its main mechanism in ARDS
A. Maintaining FiO2 at 100% for one week is that high levels of PEEP are able to recruit atelectatic
B. Adding positive end expiratory pressure alveoli, thereby increasing oxygenation.
C. Increasing the respiratory rate This is preferred in the treatment of ARDS because
D. Increasing the dead space volume mechanical ventilation may further lung injury.
Emergency 1: ARF (Batch 2020), p. 2
19. A 35-year-old male comes back to the ER because of dyspnea. C Adventitious Breath Sounds
He had a history of near fatal asthma and he was intubated for Stridor: isolated, inspiratory, monophonic wheeze
5 days a month ago. What in your PE would suggest an upper that gets louder upon auscultation when heard over
airway obstruction? the trachea.
A. Inspiratory and expiratory wheeze Suggests upper airway obstruction
B. Expiratory polyphonic wheeze An isolated expiratory wheeze that is musical or
polyphonic in nature suggests asthma.
C. Inspiratory monophasic wheeze
When the asthma progresses, you may have both
D. Squawk
inspiratory and expiratory wheezes.
Squawk is made up of short inspiratory wheezes that
are usually preceded by crackling sounds.
Heard in interstitial fibrosis and allergic alveolitis
TBL: Lung Sounds (Batch 2022), pp. 4, 9
[MED2-CARDIOPULMO] – 2nd Shift Finals (15 DEC 2021) Page 5 of 11

20. A 35-year-old male traffic aide comes to you with cough of 10 A Asthma is a disease of the airways due to chronic
weeks duration. What information in the history will you be inflammation, leading to characteristic signs and
asking that would increase the probability that he may be symptoms like wheezes, shortness of breath, chest
having asthma? tightness, and cough. These symptoms often occur at
A. Cough is nocturnal, waking patient from sleep at 2 am night or early morning, and are often triggered by viral
B. Cough is productive described as tenacious infections, allergens, and even exercise.
Productive, tenacious (dark brown) cough is seen in
C. Cough is accompanied by progressive dyspnea
cystic fibrosis.
D. Cough is reported when he is orthopneic Cough accompanied with progressive dyspnea and
orthopnea may be due to congestion; further work-up
will be needed to identify the cause of congestion in
this case.
Bronchial Asthma (Batch 2022), p. 2
21. An 18-year-old asthmatic patient reports back at the clinic B
seeking an employment clearance. He is maintained on a
moderate dose of ICS taking 800 mcg/day. His GINA symptom
asthma score is 3. How would you best manage the patient?
A. Increase the ICS dose to 1200 mcg/dose and use prn a short-
acting B2-agonist
B. Maintain the ICS and add a long-acting B2-agonist
C. Decrease the ICS and give short course of oral prednisone
(40 mg/day for 4 days)
D. Maintain the ICS and add a leukotriene cysteine antagonist

22. True statements regarding management of patients with mild A


asthma (on Step II), EXCEPT:
A. SABA alone is the preferred reliever medication Figure 3: Selecting initial controller treatment in
B. As needed ICS-Formoterol can be the controller medication adults and adolescents with a diagnosis of asthma
C. Daily low-dose ICS can be the controller medication © 2020 Global Initiative for Asthma
www.ginasthma.org
D. SABA as a reliever is an option together with an ICS
23. 28 year-old male with chronic cough, weight loss and night B Patient is classified as Category I, new case of PTB.
sweats. His sputum AFB was positive. He does not have any
previous TB treatment. The most appropriate management is?
A. 2HRZE/6HRE
B. 2HRZE/4HR
C. 2HRZES/1HRZE/5HRE
D. 2HR/4HR

Tuberculosis (Batch 2021) p.6


24. After 2 weeks on anti-TB meds, the 60-year old female diabetic C Hypersensitivity reactions usually require the
developed joint pains and tenderness. Which of the following discontinuation of all drugs and rechallenge to determine
is the likely culprit? which agent is the culprit. Because of the variety of
A. Isoniazid regimens available, it usually is not necessary although
B. Rifampicin it is possible to desensitize patients. Hyperuricemia and
C. Pyrazinamide arthralgia caused by pyrazinamide can usually be
managed by the administration of acetylsalicylic acid;
D. Ethambutol
however, pyrazinamide treatment should be stopped if
the patient develops gouty arthritis.
Harrison’s 20E p.#1252
25. A 45-year-old male, on TB treatment for 3 weeks developed C The most common adverse reaction of significance
yellowish discoloration of the skin and sclerae. Which of the among people treated for drug-susceptible TB is
following is most appropriate? hepatitis. Patients should be carefully educated about
A. Start on silymarin the signs and symptoms of drug-induced hepatitis and
B. Request for liver function test should be instructed to discontinue treatment promptly
C. Hold anti-TB medications and see their health care provider if these manifestations
occur.
D. Continue anti-TB medications, this is likely transient
For patients with symptomatic hepatitis and those
with marked (five- to sixfold) elevations in serum
levels of aspartate aminotransferase, treatment
should be stopped and drugs reintroduced one at
a time after liver function has returned to normal.
Harrison’s 20E p.#1252
[MED2-CARDIOPULMO] – 2nd Shift Finals (15 DEC 2021) Page 6 of 11

26. A 52 y/o male, consulted because of hemoptysis. He has been B Identification of Presumptive TB
having productive cough for 3 weeks accompanied with Cough of at least 2 weeks duration
dyspnea, episodes of low-grade fever, back pain and weight Most important; does not mean that your patient
loss. Which of his symptoms strongly suggest TB? has TB, but warrants the need for further
A. Back pain investigation
B. Cough for 3 weeks Unexplained cough of any duration in a close
contact
C. Episodes of fever
Chest X-ray findings suggestive of PTB, ± Sx:
D. Hemoptysis Cough of any duration
Significant unintentional weight loss
Fever
Bloody sputum or hemoptysis
Chest pains not referable to msk disorders
Easy fatigability or malaise
Night sweats
Shortness of breath or Difficulty of breathing
Tuberculosis (Batch 2021), p. 2
27. A 60 y/o female consulted because of productive cough for 2 D XpertR MTB/Rif
months, accompanied by weight loss and back pain. She has 1st step in diagnosing
no previous treatment for PTB. According to the algorithm on Has a false negative of 18% and a sensitivity of 89%,
adults suspected for tuberculosis (15 years and above), what therefore it should never be interpreted alone
is the initial test of choice to evaluate for pulmonary Consider the clinical profile of the patient, AFB
tuberculosis? smear results, and chest x-ray results
A. Sputum GS/CS Tuberculosis (Batch 2021), p 3
B. Sputum AFB Smear
C. Sputum cytology
D. Sputum Xpert MTB/Rif
28. 38 y/o male, came in at the ER because of repeated episodes B All of the choices present with crackles on auscultation,
of minimal hemoptysis. He had previous TB treatment 5 years but further probing into the patient profile will guide us
ago. On PE, you appreciate crackles which is likely secondary more to the diagnosis of bronchiectasis.
to? Bronchiectasis is one of the infectious sequelae of
A. Pleural effusion PTB, along with cavitary pneumonia and
B. Bronchiectasis aspergilloma.
Most common symptoms:
C. Consolidation
Chronic cough with copious amount of thick mucus
D. Bronchitis
Hemoptysis
Crackles and wheezing on auscultation
Unexplained weight loss
Thickening of the skin under the nails, clubbing
Frequent respiratory infections
Bronchiectasis (Batch 2022), p. 3
Hemoptysis (Batch 2022), p. 1
29. A 35 year-old call center agent had hemoptysis of 880 mL in D Signs of Respiratory Failure
24 hours. BP was 100/70, CR 110, altered sensorium. Which of Abdominal Paradox
the following data will prompt you to intubate the patient? Central Cyanosis
A. 35 years old Altered Sensorium
B. BP 100/70 Emergency 1: Acute Respiratory Failure Batch 2021 page 1
C. CR 110
D. Altered sensorium
30. A 50 y/o anesthesiologist presented herself to the ER A Anticoagulation
complaining of 1 episode of sudden chest pain while Mainstay of therapy since the introduction of Heparin
attending to an abdominal surgery. She also noticed right calf Phases:
swelling which on duplex disclosed a noncompressible Initial: unfractionated heparin (UFH) injected
popliteal vein, R. The correct statement pertinent to this case during the first 5 to 7 days, while
is: warfarin/coumadin is initiated on day 2 or 3 (note:
this warrants frequent monitoring using INR)
A. An initial bolus of unfractionated heparin at 80 U/kg, followed
Mx:
by an initial infusion rate of 18 U/kg per h to achieve an apTT Long-term: spans 7 days to months
of 60-80 s Extended: prolonged treatment of around 3
B. Warfarin requires 3 days to take full effect and is usually months to an indefinite period.
titrated empirically to achieve the target INR of 2.0-3.0 s Vitamin K antagonist, LMWH, or other agents
C. Oral anticoagulation monotherapy with apixaban with a 1- given during long-term and extended phase
week loading dose, respectively, followed by a maintenance Venous Thromboembolism (Batch 2022), p 7
dose without parenteral anticoagulation is not an option for
treatment
D. Unlike LMWH or UFH, fondaparinux does not cause heparin-
induced thrombocytopenia but requires frequent laboratory
monitoring
[MED2-CARDIOPULMO] – 2nd Shift Finals (15 DEC 2021) Page 7 of 11

31. ALL of following is TRUE regarding the use of B Transthoracic echocardiography can identify right
echocardiography in diagnosing pulmonary embolism: ventricular hypokinesis with moderate-to-large PE
A. It is a reliable diagnostic imaging tool for acute PE but it is not typically useful for diagnosing the presence
B. Hypokinetic RV free wall on echo may be seen of a PE.
C. It is not useful in ruling out conditions that may mimic PE Harrison’s 20e - Chapter 135 p. 755
D. Thrombus formation on transthoracic echocardiography
32. A 60 year old female with breast CA went to the emergency C Massive PE
room due to 7-day history of unilateral leg swelling and pain Sustained hypotension (SBP <90 mmHg for 15
over Right lower extremity. BP 110/70, CR 80 RR 22 T 36.8C. mins or requiring inotropes)
(+) leg swelling R calf 42 cm, L calf 38 cm. Pulseless
If while at the emergency room, she develops sudden dyspnea Persistent and profound bradycardia ( According
and BP goes down to 80/50 and the CR 110/min, tachypnea at to Doc, these are the one who’ll you give
thrombolysis, not just heparin)
30/min, which of the following will definitely be a clinical
Necessitates thrombolysis and anticoagulants
presentation of massive pulmonary embolism given the
(heparin)
clinical background of the patient? Examples of thrombolytics:
A. Tachypnea Alteplase
B. Tachycardia Streptokinase
C. Hypotension Urokinase
D. Dyspnea Reteplase
33. A 60 year old female with breast CA went to the emergency B
room due to 7-day history of unilateral leg swelling and pain Venous Thromboembolism (Batch 2022), p 3
over Right lower extremity. BP 110/70, CR 80 RR 22 T 36.8C.
(+) leg swelling R calf 42 cm, L calf 38 cm.
If while at the emergency room, she develops sudden dyspnea
and BP goes down to 80/50 and the CR 110/min, tachypnea at
30/min and on CT angiography a filling defect over the right
pulmonary artery is seen, which would be the management for
this case?
A. Fondaparinux
B. Alteplase
C. Enoxaparin
D. Heparin
34. A patient has been experiencing low grade fever, cough, and D Contralateral deviation
progressive dyspnea for the past month. She says that she Volume containing abnormalities
prefers to sleep on her left side as this makes her less Seen as a contralateral deviation or pushing of the
dyspneic. She had a chest x-ray done 3 days ago and it shows midline structures
pleural effusion.
Pleural effusion CXR:
Homogenous density at the (L) with an upper border
What is expected on the chest film?
curve
A. Air fluid level Meniscus sign (indicative of pleural effusion)
B. Barely visible right cardiac border Trachea is midline proximally but becomes deviated
C. Apex displaced to the anterior axillary line at upper thoracic segment
D. Trachea displaced to the right Chest imaging (Batch 2022), p.13
35. A patient has been experiencing low grade fever, cough, and B Light’s criteria (if met any = EXUDATE)
progressive dyspnea for the past month. She says that she Pleural fluid protein > 0.5
prefers to sleep on her left side as this makes her less Pleural fluid LDH/serum >0.6
dyspneic. She had a chest x-ray done 3 days ago and it
shows pleural effusion.
Exudative Type
What is expected if we have the fluid examined? Usually Produced by inflammatory conditions
A. Low pH Usually unilateral
B. High protein Local disease
Causes:
C. (+) newspaper test
Infectious (bacterial, TB, fungal, viral, parasitic)
D. Predominance of neutrophils
Neoplastic disease (Metastatic, mesothelioma
Collagen Vascular disease (SLE, Rheumatoid
pleurisy)
Pulmonary embolism
Pleural Diseases (Batch 2022), pp 3-4
36. A patient has been experiencing low grade fever, cough, and B Patient presents with pneumothorax.
progressive dyspnea for the past month. She says that she Probable cause: thoracentesis procedure
prefers to sleep on her left side as this makes her less c/c after procedure: sudden chest pain and
dyspneic. She had a chest x-ray done 3 days ago and it dyspnea
shows pleural effusion. PE: severe respiratory distress, hypotension,
compressible pulses, lagging of the left chest,
hyperresonant on percussion
[MED2-CARDIOPULMO] – 2nd Shift Finals (15 DEC 2021) Page 8 of 11

The patient underwent drainage of pleural fluid via Characteristic Ultrasound Findings
thoracentesis. During the procedure, the patient had sudden Barcode sign
chest pain and dyspnea. PE revealed a patient in severe Also called stratosphere sign
respiratory distress, with hypotension and compressible if (+), indicates pneumothorax
pulses, lagging of the left chest, with hyperresonance. What Loculations
do we expect on ultrasound? Indicate Pleural effusion
Hepatization
A. Loculations
Indicates alveolar consolidation where the lungs
B. Barcode sign take on the appearance of the liver (hepatization)
C. Hepatization in ultrasound
D. Numerous B lines Numerous B lines
Comet tail artifacts perpendicular to parietal pleura
Usually normal: implies fluid found in the lung
interstitium
Numerous/too many B lines: too much fluid in
interstitium (probably Congestive Heart Failure)
Chest Imaging (Batch 2022) p. 11-14
37. A patient has been experiencing low grade fever, cough, and B Patient presents with iatrogenic pneumothorax
progressive dyspnea for the past month. She says that she Leading causes are manipulation procedures such as
prefers to sleep on her left side as this makes her less transthoracic needle aspiration, thoracentesis,
dyspneic. She had a chest x-ray done 3 days ago and it insertion of central intravenous catheters
shows pleural effusion. After thoracentesis, the patient had Management for Iatrogenic pneumothorax
sudden chest pain and dyspnea. PE revealed a patient in Close observation (if stable)
Supplemental oxygen or aspiration
severe respiratory distress, with hypotension and
if unsuccessful, do tube thoracostomy.
compressible pulses, lagging of the left chest, with
Pleural Diseases (Batch 2022), pp 5-6
hyperresonance.

What is the immediate management?


A. Refer to TCVS for VATS
B. Needle aspiration
C. Empiric antibiotics
D. Loop diuretic
38. A patient with 10 apneas and an RDI of 5 during five hours of B AHI (Apnea Hypopnea Index) or RDI (Respiratory
sleep, has this number of hypopneas Distress Index):
A. 10 Averaged frequency of apnea and hypopnea events
B. 15 per hour of sleep
C. 20 Formula: AHI = (Apneas + Hypopneas) / hours of
D. 25 sleep
Manipulate the formula with the given values:
5 = (10 + hypopneas)/5 hours of sleep
Hypopneas = 15
Interpretation of AHI or RDI:
Normal Cessation of Breathing during sleep: 5
times per hour of sleep.
5 - 15 = Mild
6-30 = Moderate
>30 = Severe
Sleep Disordered Breathing (Batch 2022) p.5
39. The patient who would benefit from a mandibular appliance in B Clinical Variables for Oral appliance (Criteria):
OSA: Younger age
A. BMI of 36 kg/m2 Lower BMI
Smaller neck circumference
B. Apnea-Hypopnea index of 8/hour
Positional OSA (AHI higher when supine)
C. With combined central apnea Lower AHI – not a consistent predictor
D. 70 years old Increased amount of protrusion by appliance

r/o choice A: BMI of 36 is classified obese


People with increased body weight will experience
difficulty in moving the jaw forward.
r/in choice B: Smaller neck circumference and mild
apnea (AHI of 8 per hour)
r/o choice C: not included in the criteria
CPAP is gold standard for treatment of sleep apneas
r/o choice D: patient is 70 y/o
Oral devices do not work for everyone since it only
works for younger people with complete teeth.
Sleep Disordered Breathing (Batch 2022), p.9
[MED2-CARDIOPULMO] – 2nd Shift Finals (15 DEC 2021) Page 9 of 11

40. OSA may benefit and show improvement with: D Sleeping in prone position could be effective in the
A. Reduced sleep time management of obstructive sleep apnea (OSA)
B. Use of stimulants like caffeine syndrome by reducing the gravity effect on the upper
C. Increased alcohol intake airway and hence collapsibility.
D. Prone positioning Afrashi, A., & Ucar, Z. Z. (2015). Effect of prone positioning in mild
to moderate obstructive sleep apnea syndrome.
41. A 62-year-old male, chronic heavy smoker consulted because D Squamous Cell Carcinoma
of hemoptysis. On PE, there were wheezes on the entire right Strongly associated with smoking
lung field. His chest radiograph showed a centrally located Central lesion-tend to cause obstruction
mass suspicious for malignancy. Based on his clinical Appearance of a central tumor. blocking the airways
profile, which cell type is likely? Cough
A. Adenocarcinoma Dyspnea
Hemoptysis
B. Mesothelioma
Wheeze
C. Large cell carcinoma
Post-obstructive pneumonitis
D. Squamous cell carcinoma Chest discomfort
Lung Cancer (Batch 2022) pp.5-6
42. A 72-year-old male with a lung mass on the left upper lobe was C Intrathoracic extension of tumor
observed to have ptosis, miosis and anhidrosis. What is the Either by direct extension or lymphatic spread: nerves
most likely cause of these findings? chest wall and pleura , vascular involvement,
A. Extra-thoracic metastasis pericardium, heart, other viscera including
B. Hypercalcemia esophagus.
C. Intrathoracic extension of the tumor Intrathoracic Spread: Pancoast Tumor
Superior sulcus tumor
D. Lambert-Eaton syndrome
Involves the C8 and T1, T2 nerves
shoulder and arm pain, muscle wasting
Destruction of the 1st and 2nd ribs
Intrathoracic Spread: Horner’s Syndrome
Sympathetic nerve paralysis
Syndrome:
Endophthalmos
Ptosis
Miosis
Anhidrosis
Intrathoracic Spread: Superior Vena Cava
Syndrome
Engorged neck veins and precordial veins
Facial edema
Arm edema
Lung Cancer (Batch 2022) p.5
43. A 58-year-old male, chronic smoker consulted because of B Diagnosis of Lung Cancer
dyspnea and back pain. The chest Xray showed a Left upper In patients, with a suspected malignant pleural
lobe lobulated opacification measuring 7 x 7 cm. Another effusion, if the initial thoracentesis is negative, a
opacification of the left lower lung from T6 down with repeat thoracentesis is warranted. Although the
meniscus sign was also seen on CXR. Which approach is majority of pleural effusions are due to malignant
recommended for the case? disease, particularly if they are exudative or bloody,
some may be parapneumonic.
A. Open lung biopsy with frozen section
B. Thoracentesis and send fluid for cytology (+) Meniscus SIgn
C. Pleuroscopy and mediastinoscopy Homogeneous density w/ curved upward border
D. CT guided biopsy of the left lower lobe mass Signifies pleural effusion, fluid outside lungs between
parietal & visceral pleura
Harrison’s 19E p.512
Chest Imaging (Batch 2022) p.4
44. A patient with 6X4 cm lung mass underwent fiberoptic D Non-small Cell Lung Cancer
bronchoscopy endobronchial ultrasound (EBUS) Adenocarcinoma (50%)
transbronchial needle aspiration (TBNA) and showed
adenocarcinoma with mediastinal lymph node metastasis. Management of Metastatic NSCLC
PET scan showed activity over the thoracic and pelvic bones. Chemotherapy palliates, improves the quality of life,
Which treatment regimen is possible for this patient? and improves survival in patients with stage IV
NSCLC, particularly in patients with good
A. Surgery
performance in status. In addition, economic analysis
B. Neoaduvant chemotherapy then surgery has found chemotherapy to be cost-effective
C. Radiotherapy palliation for stage IV NSCLC.
D. Chemotherapy
[MED2-CARDIOPULMO] – 2nd Shift Finals (15 DEC 2021) Page 10 of 11

Harrison’s 19E p.516, 519


45. A 50 year-old male, presented with progressive shortness of C Chronic Obstructive Pulmonary Disease
breath. He stopped smoking 10 years ago. On PE, diffuse A common preventable and treatable disease that is
wheezing was noted. What is your impression? characterized by persistent respiratory symptoms
A. Bronchial asthma and airflow limitation that is due to airway and/
B. PTB alveolar abnormalities usually cause by significant
C. COPD exposure to noxious particles of gases
D. Pneumonia
Risk factors for COPD
Tobacco smoking
Biomass fuel exposure
AIr pollution

Symptoms of COPD
Chronic cough
Dyspnea
Sputum production
Wheezing
Chest tightness
Chronic Obstructive Pulmonary Disease (Batch 2021) p.1,4,9
46. Which of the following is a typical radiographic sign of A Hyperinflation is characterized as having too much air
hyperinflation? in the lungs, which pushes down the hemidiaphragm into
A. Flattened hemidiaphragm a flattened position as seen in the radiograph.
B. Meniscus sign This is typically seen in conditions such as COPD and
C. Globular heart emphysema.
D. Narrowed ICS Harrison’s 20E. p. 1992
[MED2-CARDIOPULMO] – 2nd Shift Finals (15 DEC 2021) Page 11 of 11

47. If you are suspecting COPD in a 55 year-old female, which of B In COPD there is often “air trapping” (increased
the following PE finding is likely? residual volume and increased ratio of residual
A. Bronchial breath sounds volume to total lung capacity) and progressive
B. Increased AP diameter hyperinflation (increased total lung capacity) late in
C. Resonant on percussion the disease.
D. Asymmetry in chest expansion
Physical Examination findings in COPD
Use of accessory respiratory muscles and
paradoxical indrawing of lower intercostal space
(Hoover sign)
In advanced disease – cyanosis, elevated JVP;
peripheral edema
Pursed lip; tripod; hyperinflation (barrel chest)
Seen as an increase in AP diameter in lateral
view of Chest Imaging.
Wheezing – on forced and unforced expiration
Diffusely decreased breath sounds (distant)
Hyperresonance on percussion
Prolonged expiration
Coarse crackles beginning with inspiration
Harrison’s 20E p. 1992
COPD (Batch 2021) p. 4
48. Central in the management of COPD? B In general, bronchodilators are the primary
A. ICS treatment for almost all patients with COPD and are
B. Bronchodilator used for symptomatic benefit and to reduce
C. PDE4 inhibitor exacerbations.
D. Antibiotics Harrison’s 20E, p. 1997

49. Aside from an idiopathic cause, the second most common D In areas where tuberculosis is prevalent,
cause of non-CF bronchiectasis is: bronchiectasis more frequently occurs as a sequela
A. Allergic bronchopulmonary aspergillosis of granulomatous infection. Apart from cases
B. Aspiration/GERD associated with tuberculosis, an increased incidence
C. Ciliary dyskinesia of non-CF bronchiectasis with an unclear underlying
D. Post-infectious causes mechanism has been reported as a significant
problem in developing nations.
Furthermore, epidemiological data for
bronchiectasis showed that TB and idiopathic
causes are the only common causes of non-CF
bronchiectasis written.
Harrison’s 20E p 1984
Additional: Based on the table of causes of
bronchiectasis from the book Respiratory Therapist
as Disease Manager. Post infectious is the second
leading cause of non-CF bronchiectasis.
Leen, H. Respiratory Therapist as Disease Manager, p. 30
50. A 65-year-old female patient presents with a history of A Clinical Manifestation of Bronchiectasis
copious sputum production. What PE findings on the lungs Most common symptoms:
will make you suspect of a possible bronchiectasis? Chronic cough
A. Persistent basal crackles Coughing out blood
B. Transient basal crackles Crackles and wheezing on lung auscultation
C. Expiratory wheeze Coughing large amounts of thick mucus
D. Inspiratory wheeze Unexplained weight loss
Thickening of the skin under the nails and
clubbing - Frequent respiratory infections
History
Suspect bronchiectasis in patient with recurrent
productive cough
Persistent productive cough with thick tenacious
sputum
Physical Examination
Persistent basal coarse crackles and wheezing on
lung auscultation
Examine upper airway for signs of rhinosinusitis
Clubbing of nails
Schamroth Sign
Bronchiectasis (Batch 2022) p. 4
MEDICINE 2
CARDIOLOGY-PULMONOLOGY SHIFTING EXAM
First Shift (Section A) | AY 2019-2020

QUESTION ANSWER RATIONALE

1. Which of the following clinical manifestations is a result A An elevated LVEDP is a hallmark of uncompensated
of elevated Left Ventricular End Diastolic Pressure congestive heart failure. Common symptoms include
(LVEDP)? dyspnea, fatigue, orthopnea, and PND.
A. Dyspnea
B. Dependent edema
C. Elevated JVP
D. Hepatomegaly

2. What stage of heart failure does a patient with structural A I think this question was considered bonus because the
disorder of the heart but who has never developed (Bonus) choices given were wrong. Dapat Stages A, B, C, D
symptoms of heart failure belong to stage _____? based on the ACC/AHA stages of heart failure.
A. 1
Based on the question, the answer should’ve been
B. 2
Stage B.
C. 3
D. 4 Memorize niyo nalang yung stages because this same
type of question was given last year sa lahat ng sections
ng Batch 2020! (1 point din yan!)

3. Kerley B lines are seen in this imaging modality in A Kerley B/Septal lines signifies lymphatic engorgement or
patients with heart failure: edema of connective tissues of the interlobular septa.
A. Chest X ray
B. CT scan X-ray findings in Heart Failure:
C. Doppler studies ● Kerley B lines (left-sided)
D. Echocardiography ● Cephalization
● Bronchial cuffing
● Hilar vasculature congestion
● Cardiomegaly

4. It is the initial drug of choice in patients with HFrEF: A All patients with HFrEF (ejection fraction <50%) should
A. ACE inhibitors be started on a low-dose ACE inhibitor, unless this is not
B. Angiotensin receptor neprilysin inhibitor tolerated or is contraindicated. ACEIs prolong survival in
C. Beta blocker patients with New York Heart Association Class II-IV HF;
D. Ivabradine improve patient symptoms and exercise tolerance; and
reduce hospitalizations for worsening HF

Beta-blockers should only be initiated after a patient’s


condition has stabilized to avoid precipitating HF
decompensation. They are often commenced in hospital
with a plan for up-titration every 2-4 weeks until the target
dose is reached

UST-FMS Section B 2021 1 of 28


CARDIOLOGY-PULMONOLOGY SHIFTING EXAM (SECTION A)

5. The primary imaging modality to assess cardiac C 2D Echo - primary imaging method of assessing cardiac
structure & function in heart failure: structure and function (cheaper, widely available)
A. Cardiac magnetic resonance
B. Cardiac CT Cardiac MR, Cardiac CT - more accurate (however,
C. 2-D Echo - Doppler more expensive and not always available)
D. Myocardial perfusion scintigraphy

6. The most likely cause of death of a 20-year old B Hypertrophic cardiomyopathy is the leading cause of
basketball player who suddenly succumbs to cardiac sudden death in athletes in both white and black
arrest while playing is: Americans. LV hypertrophy that develops in the absence
A. Dilated cardiomyopathy of causative hemodynamic factors like hypertension.
B. Hypertrophic cardiomyopathy Clinical manifestations include:
C. Ischemic cardiomyopathy - Cardiac arrest
D. Restrictive cardiomyopathy - From V-tach
- Syncope/Presyncope and Chest
pain/Dyspnea
- LVOT obstruction
- Reduced diastolic filling
- Mitral Regurgitation
- Asymptomatic
*Syncope is a very important manifestation

7. This is indicated in a patient suffering from hypertrophic C Hypertrophic cardiomyopathy (HCM) is a common and
cardiomyopathy with history of sudden death, heterogeneous disorder that increases an individual’s
spontaneous ventricular tachycardia and syncope: risk of sudden cardiac death (SCD).
A. Cardiac resynchronization therapy
B. Heart transplant Implantable cardioverter-defibrillator (ICD) therapy
C. Implantable cardioverter-defibrillator is the cornerstone of modern treatment for individuals
D. Left ventricular assist device at high risk of SCD. ICDs are now also widely used in
patients who survive sustained VT or VF that is not
attributable to a transient correctable cause, or who are
at high risk for recurrent arrhythmia.

Source: American Heart Association

8. Addition of this drug to ACE inhibitor should be avoided D In patients with bilateral renal artery stenosis, diffuse
in order to prevent renal dysfunction and hyperkalemia atherosclerosis and functional single kidney, ACEI and
in the treatment of heart failure: ARB can significantly worsen renal function as they are
A. Beta blocker dependent on RAAS to maintain glomerular filtration.
B. Ivabradine
C. Mineralocorticoid receptor antagonist By inhibiting formation of circulating angiotensin II or
D. Angiotensin receptor antagonist blocking angiotensin II binding to the adrenal receptor,
ACEi or ARB, respectively, interfere with the stimulatory
effect of angiotensin II on aldosterone secretion in the
adrenal gland and as a consequence impair kidney
excretion of potassium. There is also evidence that ACEi
and ARB interfere with angiotensin II generated within
the adrenal cortex.

Main mechanisms contributing to hyperkalemia with


ACEi/ARB include decreased aldosterone
concentrations, decreased delivery of sodium to the
distal nephron, abnormal collecting tubule function, and
excessive potassium intake.

Source:
https://www.ncbi.nlm.nih.gov/pubmed/21883995

UST-FMS Section B 2021 2 of 28


CARDIOLOGY-PULMONOLOGY SHIFTING EXAM (SECTION A)

Case (For Nos. 9-13): A 30 y/o male with RHD with mitral stenosis was referred for clearance prior to tooth extraction

A. Apex beat at the 6th ICS left midclavicular line


9. Which PE finding is consistent with mitral stenosis? ● cardiomegaly/ left ventricular hypertrophy:
A. Apex beat at the 6th ICS left mid-clavicular line inferior displacement of the left ventricle due to
B. S2 is split during inspiration at the base hypertrophy of the myocardium or dilatation of
C. Systolic thrill at the apex radiating at the back D the left ventricular chamber.
D. An opening snap followed by a low-pitched B. S2 is split during inspiration at the base
diastolic murmur ● normal S2 sound: due to the early closure of the
aortic valve and the delayed closure of the
pulmonic valve as an effect of the negative
pressure in the thoracic cavity during inspiration
C. Systolic thrill at the apex radiating at the back
● Pulmonic valve stenosis: Ejection click at LUSB
with valvular PS—click intensity varies with
respiration, decreasing with inspiration and
increasing with expiration S2 may split widely
with P2 diminished in intensity SEM (2–5/6) ±
thrill at LUSB with radiation to the back and
sides
D. An opening snap followed by a low-pitched
diastolic murmur
- Mitral stenosis:
● loud S1
● Opening snap: may be audible and move
closer to S2
● Mid-diastolic murmur: turbulent flow
produces characteristic low-pitched sound

10. This will confirm the presence of rheumatic heart C


disease
A. Chest X-ray
B. CT Angiography
C. Echocardiogram
D. 12 Lead ECG

11. May be responsible for left parasternal heave: D (+) Left parasternal heave or RV Heave - RV
A. Abdominal aortic aneurysm Hypertrophy, apex beat 5th LICS AAL
B. LVH
C. Pectus Excavatum In severe tricuspid valve regurgitation, pressure can
D. Severe Tricuspid Regurgitation rise in your right ventricle due to blood flowing backward
into the right atrium and less blood flowing forward
through the right ventricle and into the lungs. Your right
ventricle can expand (RVH) and weaken over time,
leading to heart failure.

UST-FMS Section B 2021 3 of 28


CARDIOLOGY-PULMONOLOGY SHIFTING EXAM (SECTION A)

12. Antibiotic prophylaxis prior to dental procedure, is B Infective endocarditis is a serious infection occurring
required to prevent this complication on the endothelial surfaces of the heart, especially at the
A. Bacteremia valves. Oral commensal bacteria are the important
B. Infective Endocarditis etiologic agents in this disease. Common dental
C. Recurrent rheumatic fever procedures, even non-surgical dental procedures, can
D. Sepsis often cause bacteremia of oral commensals. (Ho, et al.,
2006)

The guidelines recommend 2 grams of amoxicillin


given orally as a single dose 30-60 minutes before the
procedure as the drug of choice for infective
endocarditis prophylaxis. Amoxicillin has been
shown to be effective in reducing bacteraemia related to
dental procedures

13. Drug of Choice for the secondary prevention of A Benzathine Penicillin G is the drug of choice for the
Rheumatic Fever secondary prevention of Rheumatic Fever
A. Benzathine penicillin G ● Erythromycin - for patients allergic to Penicillin
B. Quinolone
C. Co-trimoxazole
D. Cefuroxime

14. A palpable substernal outward thrust is felt in a patient C (+) RV heave = RVH
suspected of having ASD. This is due to:
A. LVH
B. Left Atrial enlargement
C. Right Ventricle enlargement
D. Right atrial enlargement

15. The most commonly encountered form of ASD: C


A. Ostium Primum
B. Sinus venosus
C. Ostium secundum
D. Sinus arteriosus

16. The widely split second heart sound in ASD is due to: A A split S2 is caused physiologically during inspiration
A. Increased flow across the pulmonary artery because the increase in venous return overloads the
B. Increased flow across the tricuspid valve right ventricle and delays the closure of the pulmonary
C. Increased flow across the ASD valve. With an atrial septal defect, the right ventricle can
D. Increased flow across the aorta be thought of as continuously overloaded because of the
left to right shunt, producing a widely split S2.

Case (For Nos. 17-20): A 57 year old female government employee was brought to ER because of substernal chest pain of 3 hours
duration. She is hypertensive with 15 pack year smoking history. On cardiac auscultation, there was soft S1 at the apex and clear breath
sounds.

17. What is the diagnostic test you would request C


immediately?
A. Troponin I
B. Chest X-Ray
C. 12 Lead ECG
D. 2D Echocardiogram

UST-FMS Section B 2021 4 of 28


CARDIOLOGY-PULMONOLOGY SHIFTING EXAM (SECTION A)

18. This cardiac biomarker rises within 4-8 hours and D


generally returns after 48-72 hours
A. Troponin C
B. Troponin I
C. Myoglobin
D. CK-MB

19. If the 12L ECG showed > 2mm St segment elevation in B


leads V4, V5, V6, I and AVL, which wall of the
myocardium is involved?
A. Antero-septal
B. Antero-lateral
C. Inferior
D. Posterior

20. What is the characteristic of an atherosclerotic plaque B As the lipid core increases, the fibrous cap on top thins
that is vulnerable to rupture? out
A. Huge amount of RBC and during stress in the coronary arteries, it will rupture
B. Large lipid core the
C. Thick fibrous cap plaque and the lipid will exude out of the blood vessel.
D. Scant inflammatory cells Platelets will try to close down the defect and there is
formation of thrombus
A = huge amt of rbc = should be thrombus, RBC just
passes through the bloodstream
B = LARGE lipid core = true
C = Thick fibrous cap = should be THIN
D = Scant inflammatory = SCANT/few is false

Case (For Nos. 21 - 23): A 73-year-old male businessman consulted at the outpatient department because of chest heaviness, radiating
to the left shoulder, noted after walking 2 blocks and relieved by rest. He has been having these episodes for the past 2 months. BP
140/80 HR 91 RR 18. Cardiovascular and pulmonary examination was unremarkable.

21. What does the patient have? A


A. Stable ischemic heart disease
B. Unstable angina
C. Acute myocardial infarction
D. Prinzmetal angina

Angina Pectoris/Ischemic Heart Disease/Chronic Stable


Angina
A. Clinical Presentation
● Typically, in a man > 50 years old or a woman > 60 years
old
○ Natural estrogens have anti-inflammatory and
vasoprotective effects against heart disease
● Discomfort described as:
○ Heaviness
○ Pressure
○ Squeezing
○ Choking
● Usually occurs with exertion, relieved by rest, and does
not progress

UST-FMS Section B 2021 5 of 28


CARDIOLOGY-PULMONOLOGY SHIFTING EXAM (SECTION A)

22. What is the most common cause of myocardial A Coronary Atherosclerosis


ischemia? ● Single most important factor that causes
A. Atherosclerosis of epicardial coronary arteries obstruction to flow
B. Acute thrombosis of coronary arteries ● Epicardial coronary arteries are the major site
C. Cardiomyopathy of atherosclerotic disease
D. Inflammatory reaction

23. Which of the following is true regarding anti-platelets in B ● Aspirin is an irreversible inhibitor of cyclooxygenase
the treatment of chronic stable angina? activity.
A. Aspirin is a reversible inhibitor of cyclooxygenase ● Aspirin therapy should almost always be continued
activity indefinitely in patients with CAD.
B. Clopidogrel blocks P2Y12 ADP receptor-mediated ● In patients with stable ischemic heart disease
platelet aggregation (SIHD) treated with DAPT after drug-eluting stent
C. Combined treatments with aspirin & clopidogrel (DES) implantation, P2Y12 inhibitor therapy with
indefinitely is recommended for patients with stents clopidogrel should be given for at least 6 months
D. Warfarin is a recommended substitute if aspirin is (Class I).
not available ● Moderate-intensity oral anticoagulation alone or
combined with low-dose aspirin does not appear to
be superior to low-dose aspirin (Hyunh et al, 2001).

Source: American College of Cardiology, American


Heart Association

CASE (For Nos. 24-26). BA, a 65yo male came in to the emergency room due to chest pain, grade 10/10 crushing in character, which
awakened him from sleep. He self-medicated with 3 doses of isosorbide dinitrate sublingually which did not provide relief. ECG showed:

24. Based on the ECG, BA had an occlusion on the blood B Inferior- Abnormal ECG on leads II, III, AVF, as shown
supply on which wall of the heart?
A. Anterior
B. Inferior
C. Septum
D. Lateral

25. What is the treatment of choice for BA this patient? C Reperfusion Therapy: LIMITATION OF INFARCT
A. Anticoagulation SIZE
B. Oxygen Therapy ● ST Segment elevation of 2 mm in 2 contiguous
C. Reperfusion precordial leads and 1 mm in 2 adjacent limb
D. Sedation leads
● Golden hour of 60 minutes
● Total ischemic time of 120 minutes

UST-FMS Section B 2021 6 of 28


CARDIOLOGY-PULMONOLOGY SHIFTING EXAM (SECTION A)

● TIMI Grading System


○ Grade 1: Some penetration of contrast
material beyond obstruction but without
perfusion
○ Grade 2: Delayed perfusion up to the distal
bed
○ Grade 3: Full perfusion of infarct vessel with
normal flow
● Pharmacologic (Fibrinolysis)
● Percutaneous Coronary Intervention (PCI)
● Coronary Artery Bypass Graft (CABG)

26. If BA will undergo fibrinolytic treatment, what is an B Absolute Contraindications


absolute contraindication to it? ● Any prior intracranial hemorrhage
A. Hemorrhagic diabetic retinopathy ● Non hemorrhagic stroke or any cerebrovascular
B. Cerebral hemorrhage event within the past year
C. Active peptic ulcer disease ● Marked Hypertension (SBP >180 or DBP
D. Transient Ischemic Attack a year ago <110mmHg)
● Suspected aortic dissection
● Active internal bleeding (excluding menses)

27. Which of the following is TRUE regarding angina A


pectoris?

A. Angina pectoris is chest discomfort primarily


caused by myocardial ischemia, which is
precipitated by an imbalance between myocardial
oxygen requirements and myocardial oxygen
supply, resulting in insufficient delivery of oxygen
to meet their heart’s metabolic demands.

B. Myocardial oxygen demand decreases by


increased heart rate (HR), ventricular wall stress,
and myocardial contractility, whereas myocardial
oxygen supply is determined by coronary blood
flow and coronary arterial oxygen content.

C. When asked for the location, angina may present


anywhere in the chest area not uncommon, it may
also present as epigastric pain.

D. Angina usually occurs acutely/sudden, then it


persists for hours. It can radiate to either shoulder
and to both arms (especially the ulnar surfaces of
the forearm and hand)

UST-FMS Section B 2021 7 of 28


CARDIOLOGY-PULMONOLOGY SHIFTING EXAM (SECTION A)

28. What diagnostic test/s are helpful for initial assessment A


of patient with suspected ischemic heart disease?
A. 12-L ECG, Treadmill Stress Test, 2D-Echo
B. 12-L ECG, Treadmill Stress Test, CT Coronary
Angiogram
C. 12-L ECG, 2D-Echo, Coronary Angiogram
D. 12-L ECG, Treadmill Stress Test,Cardiac MRI

*second pic from trans

Coronary Angiography
● Expensive ₱₱₱₱₱
● The radiographic visualization of the coronary
vessels after the injection of radiopaque
contrast
● “Gold standard” for identifying the presence or
absence of arterial narrowing related to
atherosclerotic coronary artery disease
● Provides the most reliable anatomical
information for determining the
appropriateness of medical therapy,
percutaneous coronary intervention or
coronary artery bypass graft surgery in patients
with ischemic CAD
● Invasive

Cardiac MRI indications:


● Viability assessment after MI
● Mural thrombosis
● Arrhythmogenic RV cardiomyopathy with fatty
infiltration
● Marfan’s Syndrome or Severe dilatation of
ascending aorta o Partial anomalous
pulmonary venous drainage

29. Given the clinical data and if the 12-LECG for this C Commonly used IHD Drugs:
patient is as shown (anterolateral wall ischemia), what ● Nitrates
MEDICAL management would you start this patient on? ● Beta- Blockers
A. Aspirin, Clopidogrel, Beta-Blockers, RAAS ● CCB- used only if beta-blockers are
Blockers, High-dose Statin contraindicated
B. Aspirin, Clopidogrel, Calcium Channel Blockers, ● Antiplatelet Drugs
Nicorandil, High dose Statin
C. Aspirin, Clopidogrel, Beta-Blockers, ISMN, RAAS Other Therapies:
Blockers, High-dose Statin ● ACEI
D. Aspirin, Clopidogrel, Beta-Blockers, ISMN, High ● Nicorandil
dose Statin ● Ivabradine

Risk Reduction Drug:


● Statin

UST-FMS Section B 2021 8 of 28


CARDIOLOGY-PULMONOLOGY SHIFTING EXAM (SECTION A)

CASE (For Nos. 30-31): A 68y female known hypertensive and diabetic presents to the emergency room wit palpitations and mild
shortness of breath. She has no heart failure and no history of stroke. Vital signs showed BPP 120/80, HR 127 bom, RR 24. On PE,
there are no murmurs on auscultation. ECG showed the following:

30. What is the ECG interpretation? A Atrial Fibrillation


A. Atrial Fibrillation ● Irregularly irregular narrow complex
B. Supraventricular Tachycardia tachycardia with no discernible P waves
C. Ventricular tachycardia ● Chaotic irregular atrial fibrillatory waves
D. Ventricular fibrillation ● Irregularly irregular heart beat
● Pulse Deficit
● Absent A wave
● Varying intensity of S1

B. Supraventricular Tachycardia

C. Ventricular Tachycardia

D. Ventricular Fibrillation

31. The first priority in the treatment of this patient is: C Rate Control and Rhythm Control
A. Rhythm control through DC Cardioversion with 100 ● Beta-blocker
joules ● Verapamil
B. Rhythm control through pharmacologic ● Diltiazem
cardioversion with amiodarone ● Digoxin
C. Rate control with a beta blocker or calcium channel ● Also consider chronic rate control due to increased
blocker chance of recurrence
D. Anticoagulation with intravenous heparin ● Digoxin (rhythm control)

Case (For Nos. 32-33) An unidentified male was rushed to the emergency room because of loss of consciousness. On hooking to the
cardiac monitor, the tracing showed the following:

UST-FMS Section B 2021 9 of 28


CARDIOLOGY-PULMONOLOGY SHIFTING EXAM (SECTION A)

32. Identify the above rhythm: Ventricular Fibrillation


A. Atrial fibrillation D ● Can come from V-Tach
B. Supraventricular Tachycardia ● Associated with coarse or fine chaotic undulations
C. Ventricular Tachycardia of the ECG baseline
D. Ventricular fibrillation ● No P wave
● No true QRS complexes
● Indeterminate rate
● Patient is in cardiac arrest

33. What is the next appropriate thing to do? Ventricular Fibrillation/ Pulseless VT
A. Do synchronized cardioversion with 100 joules ● EARLY DEFIBRILLATION IS KEY!
followed by chest compression ● If in arrest. Do CPR immediately
B. Do synchronized cardioversion with 200 joules ● Based on ACLS guidelines:
followed by chest compression D ○ CPR (Airway, oxygen, connect monitors)
C. Do defibrillation with 100 joules followed by chest ○ 120-200 joules if biphasic defibrillator
compression ○ 360 joules if monophasic defibrillator
D. Do defibrillation with 200 joules followed by chest ○ Continue CPR for 2 min.
compression ○ Epinephrine 1mg every 3-5 min.
○ Amiodarone OR Lidocaine
○ If spontaneous circulation is still absent, go
back to defibrillation
○ If spontaneous circulation present, go to post
cardiac arrest case

Case (For Nos. 34-35): A 90y male was referred to you in the ICU because of unresponsiveness. On checking the cardiac monitor,
the tracing was showed of the following.

34. identify the rhythm


A. Asystole
B. Course V fib A
C. Fine V fib
D. Pulseless electrical activity

UST-FMS Section B 2021 10 of 28


CARDIOLOGY-PULMONOLOGY SHIFTING EXAM (SECTION A)

35. What is the next immediate appropriate action? A


A. Chest compression
B. Defibrillate with 200 joules
C. Intubate patient and hook to ventilator
D. Transcutaneous pacing

● For asystole/PEA, no need to shock!


● Just do CPR and give epinephrine

CASE (For nos. 36-37): After your intervention , the ECG tracing taken showed the following

36. Identify the above rhythm: C Features common to any broad complex tachycardia
A. Atrial fibrillation ● Rapid heart rate (> 100 bpm).
B. Supraventricular tachycardia ● Broad QRS complexes (> 120 ms).
Features suggestive of VT
C. Ventricular tachycardia
● Very broad complexes (>160ms).
D. Ventricular fibrillation ● Absence of typical RBBB or LBBB morphology.
● Extreme axis deviation (“northwest axis”) — QRS is
positive in aVR and negative in I + aVF.
● AV dissociation (P and QRS complexes at different
rates).
● Capture beats — occur when the sinoatrial node
transiently ‘captures’ the ventricles, in the midst of AV
dissociation, to produce a QRS complex of normal
duration.
● Fusion beats — occur when a sinus and ventricular beat
coincide to produce a hybrid complex of intermediate
morphology.
● Positive or negative concordance throughout the chest
leads, i.e. leads V1-6 show entirely positive (R) or
entirely negative (QS) complexes, with no RS complexes
seen.
● Brugada’s sign – The distance from the onset of the QRS
complex to the nadir of the S-wave is > 100ms.
● Josephson’s sign – Notching near the nadir of the S-
wave.
● RSR’ complexes with a taller “left rabbit ear”. This is the
most specific finding in favour of VT. This is in contrast
to RBBB, where the right rabbit ear is taller.

How to differentiate Vtach from Vfib


Vfib is rapid totally incoordinate contraction of ventricular fibers;
the EKG shows chaotic electrical activity and clinically the
patient has no pulse. Vtach is defined by QRS greater than or
equal to 120 millisecs and a rate of greater than or equal to 100
beats per minute.

UST-FMS Section B 2021 11 of 28


CARDIOLOGY-PULMONOLOGY SHIFTING EXAM (SECTION A)

37. What is the next immediate appropriate action? B Following the algorithm, the next step should be to
A. Cardiovert determine if there is a pulse.
B. Defibrillate
C. Give Epinephrine 1mg IV
D. Give amiodarone 300 mg IV

However, a MONOMORPHIC type of Ventricular


Tachycardia indicates that the patient also has NO
PULSE. This means that the next appropriate step is to
defibrillate.

CASE (For no. 38-39): A 28y female presents to the emergency room with palpitations. Vital signs showed BP 120/80, HR 170 bpm,
RR 24. ECG showed the following:

38. What is the ECG interpretation? B Supraventricular tachycardia


A. Atrial fibrillation ● Re-entry around the AV node tissue or
B. Supraventricular tachycardia involving the normal AV nodal conducting
C. Ventricular tachycardia system and an accessory pathway
D. Ventricular fibrillation ● Tachycardia with narrow QRS complex
● Sudden onset and termination
● 150-250 beats/min
● Regular rhythm
● QRS complex is normal in contour and duration
● No discernable P waves (buried within QRS)
● FAST, NARROW, REGULAR, NO P WAVE

39. Which of the following is the first line pharmacologic C


agent to give in the emergency management of this
patient?
A. Atropine
B. Atenolol
C. Adenosine
D. Amiodarone

UST-FMS Section B 2021 12 of 28


CARDIOLOGY-PULMONOLOGY SHIFTING EXAM (SECTION A)

40. The following is a routine diagnostic test than can be D Doppler Device in the dorsalis pedis artery is a routine
requested to work up the intermittent claudication of NG diagnostic test for the work up for intermittent
A. CT Angiography claudication
B. MR Angiography
C. 12 Lead ECG
D. Doppler Device in the dorsalis pedis artery

41. The primary site(s) of arterial involvement in 80-90% of C Peripheral Artery Disease
patients with PAD ● Stenosis or occlusion in the aorta or the arteries of
A. Abdominal aorta the limbs
B. Iliac arteries ● Primary sites of involvement:
C. Femoral and popliteal arteries ● Abdominal aorta and iliac arteries: 30% of
D. Tibial and peroneal arteries symptomatic patients
● Femoral and popliteal arteries:80-90%
● Tibial and peroneal: 40-50%

42. Pharmacologic agent that will improve the symptoms of B Cilostazol is used to treat the symptoms of intermittent
NG claudication by keeping platelets in the blood from
A. Amlodipine sticking together and clotting thus improving the
B. Cilostazol circulation
C. Metoprolol
D. Ramipril

CASE (For nos. 43-45): A 45 y/o male mechanic consulted for employment clearance. He smokes 10 sticks of cigarettes a day since
25 y/o and drinks 2 bottles of beer everyday. He had BP elevations in the past but no formal consult was made because he claims that
he is totally asymptomatic. PE showed BP 145/100, PR 115bpm regular, with the apex beat at 5th LICS MCL not sustained and not
diffused. There are noticeable tophi on his metacarpo-phalangeal joints. The rest of the PE is unremarkable.

43. Based on JNC 7, patient has C


A. Pre-HPN
B. Stage I HPN
C. Stage II HPN
D. Stage III HPN

44. The patient brought his CXR PA done yesterday. What B The patient’s apex beat is found at the 5th LICS MCL,
do you expect to see? therefore we can assume that there is no cardiac
A. Cardiomegaly enlargement. The expected finding is a normal Cardio-
B. Cardio-thoracic ratio of 0.50 Thoracic ratio of Less than or Equal to 0.50
C. Rounding of the cardiac apex
D. Loss of cardiac waistline

UST-FMS Section B 2021 13 of 28


CARDIOLOGY-PULMONOLOGY SHIFTING EXAM (SECTION A)

45. What Antihypertensive medication is best suited for the A


patient?
A. Metoprolol
B. Nifedipine
C. Hydrochlorothiazide
D. Spironolactone

Since the patient is hypertensive and tachycardic (PR


of 120), a Beta Blocker such as metoprolol is highly
suggestive

CONT. (For Nos. 46-48):: Patient was low to follow up and returns 5 years after for another employment clearance. He claims he
stopped taking the medicine you gave him after he consumed the 30 tablets you prescribed. He claims he is asymptomatic. PE showed
BP of 200/100. CR 90 bpm, apex beat is at the 6th iCS LAAL, sustained and diffuse. The rest of the PE is unremarkable.

A V1 very deep S and V6 very deep R indicates left


46. You did a 12 L ECG on patient. What finding will you ventricular hypertrophy
expect to see?
A. Lead II- P wave 0.14 sec, V1 very deep S, V6 very
tall R
B. Lead II- P wave 0.4 mV, V1 very tall R, V6 very
deep S
C. Lead II- P wave 0.14 sec, V1 very tall R, V6 very
deep S
D. Lead II- P wave 0.4 mV, V1 very deep S, V6 very
tall R

47. Based on JNC7, patient has: C


A. Stage II HTN
B. Stage III HTN
C. Hypertensive Urgency
D. Hypertensive Emergency

48. How will you manage the patient? D Treatment of modality of choice for hypertensive patients
A. Admit the patient to the ICU and start furosemide is combination therapy of ACE-I or ARBS with CCBs or
40 mg IV push every 4 hours thiazides based on 2013 ESH/ESC Hypertension
B. Admit the patient to a regular room in the hospital Guidelines. For hypertensive urgency situations, with
and start nicardipine intravenous drip prior treatment re-intensification of drug therapy is a
C. Put clonidine 150 sublingual q 15 mins until BP is must.
120/80 then send the patient home?
D. Start combination with Losartan and amlodipine
and follow up next week.

UST-FMS Section B 2021 14 of 28


CARDIOLOGY-PULMONOLOGY SHIFTING EXAM (SECTION A)

CASE (For Nos. 49-50): RB, a 65 year old, male, smoker consulted because of occasional chest pain accompanied by lightheadedness
or presyncope upon exertion. Auscultation revealed grade 3/6 systolic crescendo-decrescendo murmur best heard over the Erb’s point
radiating to the suprasternal notch.

49. What is you diagnosis? B Aortic stenosis - (+) 3/6 mid-systolic crescendo -
A. Atrial Septal Defect decrescendo murmur best heard over the Erb’s point
B. Aortic Stenosis radiating to the suprasternal notch
C. Mitral Regurgitation Exertional syncope - decline in arterial pressure
D. Tricuspid Regurgitation caused by vasodilation in the exercising muscles and
inadequate vasoconstriction in non-exercising muscles
in the face of a fixed CO, or from a sudden fall in CO
produced by an arrhythmia

Aortic stenosis is the narrowing of the aortic valve. It is


the most common valvular heart disease in the
developed world. The condition is described as a harsh
crescendo-decrescendo systolic murmur heard best at
the second right and 3rd LICS which is transmitted
upward along the carotid arteries.
As a result, the outflow of blood from the left ventricle
into the aorta is obstructed. This leads to chronic and
progressive excess load on the left ventricle and
potentially left ventricular failure.
The carotid arterial pulse rises slowly to a delayed peak
(pulsus parvus et tardus) - can occur in the presence of
LV systolic dysfunction

50. Which of the following diagnostic procedures may A Stress testing can be performed with exercise or by
produce syncope when performed on RB? pharmacological means. Exercise testing is a method for
A. Treadmill stress test risk stratifying ONLY patients with AS who are able to
B. 2D Echo Doppler exercise. It is important to rule out the following
C. 24 hours Holter Monitoring contraindications before performing the test:
D. Coronary Angiography ● an established indication for AVR (aortic valve
replacement)
● uncontrolled hypertension
● symptomatic or hemodynamically significant
arrhythmia (#49)
● inability to perform the test such as orthopedic
limitations or global disabilities

51. A patient had fever, cough, dyspnea. On PE, he has C Patient has consolidation
symmetrical chest expansion with increased fremiti, Choice A: Pleural effusion
dullness, and bronchial breath sounds on the right Choice B: Pleural effusion
posterior chest, from T8 down. What is the expected Choice C: Consolidation
chest radiography? Choice D: No shifting seen in consolidation
A. Homogeneous density obscuring the right cardiac
border
B. Meniscus sign
C. Air Bronchograms
D. Shifting of mediastinum to the left

52. A patient was diagnosed to have long-standing COPD, D COPD phenotypes:


predominantly Emphysema. Which of the following is ● Chronic Bronchitis: cough and sputum 3
expected on his chest X-ray? months per year for 2 years
A. Right hemidiagphragm at the level of T10 ● Emphysema: HYPERINFLATIONS; dyspnea,
B. Cardiac apex displaced to the left exercise intolerance
C. Localized area of oligema ● Asthma: allergens; eosinophils;
D. Flattened hemidiaphragms bronchoconstriction; hyperinflation

UST-FMS Section B 2021 15 of 28


CARDIOLOGY-PULMONOLOGY SHIFTING EXAM (SECTION A)

EMPHYSEMA
● Acinus (respiratory bronchiole, alveolar ducts,
alveoli): loss of elastic recoil
● Destruction of alveoli means less surface area
for gas exchange

53. What is the hallmark of the diagnosis of A Visible visceral pleura is pathognomonic of
Pneumothorax? pneumothorax
A. Visible visceral pleura ● Other radiologic findings of pneumothorax
B. Contralateral shift of the midline structures ○ Absent bronchovascular markings
C. Localized area of oligemia ○ Contralateral deviation of mediastinal
D. Obtuse angle with the chest wall structures
○ Widened ICS
● Ultrasound:
○ Barcode/Stratosphere sign
○ No lung sliding, no comet tails

Choice B: Contralateral shift of midline structures is not


a hallmark of diagnosis because this can also be seen in
Pleural effusion.

Choice C: (+) Westermark’s sign - localized area of


oligemia; decreased bronchovascular markings. This is
seen in PULMONARY EMBOLISM

Choice D: This finding can’t be found in the trans but


according to a journal, a pleural neoplasm shows
obtuse angles with the lateral chest wall with tapered
margins displaces the pulmonary vasculature, changes
its location on respiration, and may show incomplete
border sign on chest radiograph (Sureka, et al., 2013)

54. A 28 y/o female, known case of SLE, consulted C Magic numbers:


because of breathlessness. Spirometry showed a FEV1/FVC: >70%
FEV1/FVC ratio 85%, FEV1 80%, and FVC 55%. What FVC or FEV1: >80%
is most likely the pathology? Patient showed FVC of 55%, so this is abnormal
A. Bronchial hyperresponsiveness
B. Chronic Obstructive Pulmonary Disease Low FVC = restrictive (PAINT)
C. Interstitial lung disease
D. Neuromuscular weakness According to Johns Hopkins.net, lung presentation in
SLE patients is LUNG SCARRING —> INTERSTITIAL

55. What is the volume of air remaining in the lungs after B


maximal expiration?
A. Tidal volume
B. Residual volume
C. Inspiratory Reserve volume
D. Expiratory Reserve volume

UST-FMS Section B 2021 16 of 28


CARDIOLOGY-PULMONOLOGY SHIFTING EXAM (SECTION A)

56. A 55 year old male consulted because of 4 week history C RESTRICTIVE = Low FVC
of cough. His chest X-ray showed findings compatible
with Pulmonary TB. What is the expected spirometric
parameter of this patient?
A. Low FEV1
B. High FEV1
C. Low FVC
D. High FVC

57. A 35 y/o male, came in at the ER because of shortness B ABG would provide the most information on the status
of breath. He is a known asthmatic, RR at 29, O2 sat at of the patient. Let’s review the indication for ABG:
88% with wheezing. Which of the following would be ● To know the ventilatory status of the patient
your priority (pCO2)
A. Intubate with ET size 8 ● Acid-base balance (pH)
B. ABG ● To know the oxygenation status of the patient
C. Spirometry (pO2)
D. Chest CT scan
Since the patient is not in a state of altered sensorium,
invasive mechanical ventilation such as intubation is
uncalled for.

With the patient having a low O2 saturation. We can not


do spirometry. But here’s a review of its indications
Indications for spirometry:
● Aids in the diagnosis of respiratory diseases
● characterizes the pathophysiology
● assess the severity
● monitors response to therapy
● May be used as a screening process

58. 69 year old male COPD patient is in exacerbation ABG B


showed pH 7.23 PaCO2 58 PaO2 88 HCO3 32. Which
of the following is possible?
A. Hypoxemic Respiratory Failure
B. Hypercapnic Respiratory Failure
C. Ischemic Respiratory Failure
D. Normal

59. 28 years old female asthmatic patient is in A pH = 7.47 (Alkalotic)


exacerbation. Her ABG at the ER showed pH of 7.47, PaCO2 = 23 (Low; NV 35-45 mmHg)
PaCO2 23, PaO2 89, HCO3 25. What is the HCO3 = 25 (Normal)
interpretation?
A. Uncompensated Respiratory Alkalosis
B. Compensated Respiratory Alkalosis
C. Uncompensated Respiratory Acidosis
D. Compensated Respiratory Acidosis

UST-FMS Section B 2021 17 of 28


CARDIOLOGY-PULMONOLOGY SHIFTING EXAM (SECTION A)

60. 48 years female has fever, cough and dyspnea. Her C C: PaO2 = 88, FiO2 = 21% (FiO2 at room air)
ABG at room air while at the ER showed pH of 7.7, PaO2/FiO2 = 419
PaCO2 28, PaO2 88, HCO3 25. Which of the following Normal PF ratio for <60 y.o. Is 400 - 500
is true?
A. ABG interpretation is uncompensated respiratory A: Acid base abnormality present is uncompensated
acidosis. respiratory alkalosis (low CO2)
B. The patient has inadequate oxygenation. B: Available indices of oxygenation (PaO2, P/F ratio) are
C. She is not hypoxemic by PF ratio all normal.
D. An interstitial pulmonary fibrosis is likely D: Interstitial fibrosis would lead to a low tidal volume,
thus leading to hypoventilation.

61. An 80 year old woman is in the emergency department. B


Her daughter says that she has been confused (which
is new for her). She has had a cough. She is known to
have COPD and angina. Examination reveals a
respiratory rate of 28/minute and right sided crackles. A
chest X-ray shows shadowing in the right lower zone.
Which of the following statements is correct?
A. The patient can be managed at home
B. The patient need to be admitted for assessment
C. The patient only needs amoxicillin
D. The patient’s confusion will not improve

Patient is confused and has unstable co-morbid


conditions (COPD and angina)

62. A 65-year old man is admitted in hospital having C If the patient has pneumonia and the test is positive,
returned from his holiday in Europe 5 days ago. He is then you should consider the patient to have
confused with dry cough. He has a respiratory rate of Legionnaires' disease. (www.cdc.gov)
30 and a BP of 60/40. On PE there is dullness to
percussion the RUL. Bronchial sounds and crackles Investigate for specific pathogens only when the
were appreciated on the RUL. Crackles also notes at etiology would significantly alter standard empirical
the left lower lobe. Which of the following statements in management decisions:
correct? ● Recent travel
● (+) Legionella UAT result

A. The patient should be started on anti-TB


medications.
B. The patient needs to have a beta-lactam
lactamase inhibitor only
C. Testing for Legionella is indicated
D. The patient can be managed in the hospital ward

63. A 22 year old varsity swimmer has been complaining of A Macrolides are indicated for gram negative
dry cough and chest tightness for the past week.he was microorganisms and are indicated for atypical
seen at a clinic where amoxicillin 500g capsules every pneumonia (pharma notes)
8 hours was started. There was no relief of symptoms.
On PE, RR=22/min, BP 120/80, T=38C. Fine end Amoxicillin alone does not have a good gram negative
inspiratory crackles were appreciated bilaterally. The x- coverage. Its better to combine it with clavulinic acid (co-
ray showed interstitial pneumonia bilaterally. Which of amoxiclav) or a macrolide for a broader coverage.
the following statements is correct?

UST-FMS Section B 2021 18 of 28


CARDIOLOGY-PULMONOLOGY SHIFTING EXAM (SECTION A)

A. add a macrolide
B. add a second generation cephalosporin
C. add a quinolone
D. add oseltamivir

64. A 30-year-old man diabetic is admitted to the hospital. A A - Patient is hypotensive (SBP <90 mmHg) &
He is confused with a fever and a dry cough. His RR is tachypneic (RR ≥30/min). CRB65 score is 2.
40/min and BP is 70/50. Which of the following CURB65 score of 2 = Intermediate mortality (9.2%),
statements is correct? admit, antibiotics, supportive care (*Refer to CURB65
A. His estimated mortality is 10% algorithm)
B. Amoxicillin and clarithromycin would be an B - Not a high risk patient
appropriate choice C - We must admit the patient
C. Admission to critical care is necessary D - Presentation is not atypical
D. Mycoplasma is the likely organism Author’s notes: Aside from hypotension and
tachypnea, the patient is also confused. This should
make the CURB65 score 3. Following the CURB65
algorithm, group 3 is high mortality (15-40%), must be
admitted, and given antibiotics and supportive care (ICU
only required for ≥4 CURB65 score). The answer may
actually be B if we follow Harrison’s management
guidelines for medium to high severity CAP which
includes B-lactam + Macrolide.

65. A 72-year-old male smoker with moderate severity A


pneumonia has been on antibiotics for five days. He is
improving clinically. His observations are: temperature
37.3 C, heart rate 80/minute, blood pressure 120/80
mmHg, respiratory rate of 14/minute. Which of the
following statements is correct?
A. It is appropriate to discharge the patient
B. The patient should remain in the hospital
C. He should stop his antibiotics
D. He does not a repeat chest x-ray

66. Which of the following will help recruit alveoli to B Positive End expiratory Pressure (PEEP) - Prevents
participate in the gas exchange process? the alveoli from collapsing
A. SIMV
B. PEEP
C. AC Mode
D. High tidal volume

67. 21 year old female asthmatic did not respond to the C Patient is showing signs of Respiratory Failure
initial SABA and IV steroids. You noted presence of ● Bluish discoloration of the lips (central
bluish discoloration of the lips with intercostal cyanosis)
retractions. What will you do? ● Abdominal retractions
A. Request for ABG ● Altered Sensorium
B. Request for portable CXR
C. Intubate the patient Must intubate the patient
D. SABA for 3 more doses

UST-FMS Section B 2021 19 of 28


CARDIOLOGY-PULMONOLOGY SHIFTING EXAM (SECTION A)

68. 65 years old diabetic had severe pneumonia and was C


admitted at the ICU due to septic shock, intubated with
PEEP at 5 cm H2O, FiO2 100%. Her ABG PaO2 68.
What is your consideration?
A. Mild ARDS
B. Moderate ARDS
C. Severe ARDS
D. Very severe ARDS

P/F = 68/1 = 68
Fits under severe criteria

69. A 22 year old female student, presents with episodic A


dyspnea, chest tightness, and cough. She has a history
of atopy as a child. She was seen several months ago
for these symptoms at the ER and was given a
salbutamol inhaler, which provided symptomatic relief.
She confides that she has consumed 12 canisters of
salbutamol for the past year. For the past 4 weeks, she
has been using salbutamol over 3x a week at daytime
and at least two times per week at night when she
awakens with symptoms. How would you assess her
asthma according to symptom control?
A. Uncontrolled
B. Partly controlled
C. Controlled
D. Mild Persistent

Take note of 12 canisters of salbutamol for the past


year

70. A 45 year old seaman consulted because of nocturnal A When managing your patient, review their response after
cough. He had spirometry 2 years ago. He has been 2-3 months and adjust treatment based on the stepwise
placed on Salbutamol inhaler which he uses as needed. approach. Consider stepping down if the asthma has
Which of the following is indicated for this patient for been well-controlled for 3 months.
daily use to improve asthma control?
A. Inhaled corticosteroids
B. Inhaled long-acting B2 agonist (LABA)
C. Inhaled long-acting anticholinergic
D. Inhaled long-acting B2 agonist (LABA) + Inhaled
long-acting anticholinergic

UST-FMS Section B 2021 20 of 28


CARDIOLOGY-PULMONOLOGY SHIFTING EXAM (SECTION A)

71. A 28 year old male driver presents to your GP clinic with B


worsening asthma. Which of the following observations
should alert you that the patient has acute severe
asthma?
A. Talks in sentences
B. Prominent sternocleidomastoid muscle when
breathing
C. Assumes tripod position
D. Peak flow 70% of best or predicted

Talks in sentences - Mild


Prominent SCM when breathing - Moderate to severe
Assumes tripod position -
Peak flow 70% of best or predicted - mild

72. Which of the following risk factor for asthma with B


increase mortality?
A. Rhinosinusitis
B. High SABA use
C. Tobacco smoke
D. Post Bronchodilator FEV1 65%

73. A 45 year old baker goes to the ER with asthma B


exacerbation. He has been confined 6 months ago for
severe asthma. Which of the following admitting orders
is correct?
A. Ipratropium bromide nebulization now then every
20 minutes for 3 doses
B. Hydrocortisone 100 mg IV now
C. Diazepam 10 mg IV now then every 4 hours for
restlessness
D. Clarithromycin 500 g once a day for 7 days

UST-FMS Section B 2021 21 of 28


CARDIOLOGY-PULMONOLOGY SHIFTING EXAM (SECTION A)

74. A 35 year old female presented with 3 week history of A Identification of Presumptive Pulmonary Tuberculosis:
productive cough with yellowish sputum. There was 1. Cough of at least 2 weeks duration
accompanying anorexia, low grade fever, 2-3 bouts of ○ Most important; does not mean that
hemoptysis amounting to 2 tablespoons per episode your patient has TB but warrants the
and 5 lbs. weight loss since 1 month ago. She has had need for further investigation
one week intake with co-amoxiclav, however cough 2. Unexplained cough of any duration in a close
persisted. Which of the symptoms is most commonly contact
associated with PTB? 3. Chest X-ray findings suggestive of PTB +/-
A. Cough symptoms
B. Hemoptysis 4. Any of the following:
C. Low grade fever ○ Cough of any duration
D. Weight loss ○ Significant unintentional weight loss
○ Fever
○ Bloody sputum or hemoptysis
○ Chest pains not referable to any
musculoskeletal disorders
○ Easy fatigability or malaise
○ Night sweats
○ Shortness of breath or difficulty of
breathing

75. What is the next step to ascertain the diagnosis? D Xpert MTB/Rif:
A. Bronchoscopy ● WHO standard for rapid TB Diagnosis
B. Mantoux test ● Indications:
C. Chest CT Scan ○ Initial diagnostic test for presumptive
D. Sputum Gene Xpert TB
○ Initial diagnostic test for Drug-resistant TB
(DR-TB/Rifampicin resistant)
○ Immunocompromised, elderly patients
○ Follow-on test for sputum negative but with
positive CXR findings
● Patient should be able to provide sputum
samples

**Gold Standard is still CULTURE but results take too


long, access is limited, and it is expensive.

76. Chest radiograph was done. Which chest radiograph C In active pulmonary TB, infiltrates or consolidations
findings is expected taking into account the patient’s and/or cavities are often seen in the upper lungs with or
presentation? without mediastinal or hilar lymphadenopathy.
A. Air fluid level on the right paracardiac area
B. Bilateral Pleural effusion
C. Right upper lobe infiltrates with cavities
D. Prominent pulmonary artery

77. If this patient has had a previous treatment for PTB in C Indications for performing sputum TB culture with drug
2006 which she took for 6 mos, which diagnostic test is susceptibility testing (DST):
warranted to determine the disease activity? a. Retreatment cases
A. Blood culture b. Treatment failure
B. IGRA c. contacts of known DR-TB cases
C. MTB Culture and Sensitivity -DST should not be routinely performed among new
D. Quantiferon Assay cases of PTB
-Those who are positive for GeneXpert whill undergo
DST
-Those who tested negative but are more likely active will
undergo DST

UST-FMS Section B 2021 22 of 28


CARDIOLOGY-PULMONOLOGY SHIFTING EXAM (SECTION A)

78. Sputum MTB Culture was positive for Mycobacterium C


tuberculosis; Sensitive to all medications. She has
completed treatment at DOTS center in 2016. Which
treatment regimen should be given?
A. 2HREZ/10HR
B. 2HREZ/4HRE/3HR
C. 2HREZS/1HRZE/5HRE
D. 3HREZS/2HRZE/4HRE

79. If you are suspecting the airway bleeding is coming C The patient must assume a right lateral decubitus
from the right lung, what is the initial best maneuver to position in order to protect/prevent the left lung from
consider? spillage.
A. Perform bronchoscopy
B. Nebulize with salbutamol GOOD LUNG UP.
C. Assume a right lateral decubitus position
D. Intubate using ET size 8

80. A 38 year old male had TB treatment in 2017. Latest D In the Philippines, the most common cause of
GeneXpert is negative. He came in with hemoptysis bronchiectasis is Post-infective, usually from TB.
and on PE you heard crackles on the right upper to mid
lung field. Which one is the assessment? CAP-moderate risk - The common triad of pneumonia is
A. CAP, MR cough, dyspnea, and fever, which are not found in the
B. Pulmonary congestion patient.
C. TB reactivation Pulmonary congestion - while crackles may suggest
D. Bronchiectasis congestion, it would manifest bilaterally.
TB reactivation - recent GeneXpert results were
negative.

81. What is the most common sign/ symptom for VTE- A Characteristic signs and symptoms such as
PULMONARY EMBOLISM? tachycardia, dyspnea, chest pain, hypoxemia, and shock
a. Tachypnea and dyspnea are non-specific and are present in many other
b. Crackles and pleuritic pain conditions, such as acute MI, congestive heart failure, or
c. 4th heart sound and cough pneumonia. In the Prospective Investigation of
d. Accentuated P2 and hemoptysis Pulmonary Embolism Diagnosis II (PIOPED II) trial,
patients with PE had a range of signs and symptoms.
Common signs were tachypnea (54%) and tachycardia
(24%).
The most common symptoms were dyspnea, usually of
onset within seconds, at rest or with exertion (73%),
pleuritic pain (44%), calf or thigh pain (44%), calf or thigh
swelling (41%), and cough (34%).

82. Which treatment is appropriate for a 35 yr old woman B Deep vein thrombosis
with right leg swelling and venous duplex scan - venous duplex scan findings of non-compressible
findings of non-compressible popliteal vein? popliteal vein. Veins are normally highly compressible.
a. Hesperidin + diosmin - most common symptom: unilateral swelling
b. LMWH - most common sign: Homan's sign ( calf pain at
c. Sequential compression device dorsiflexion of the foot)
d. Streptokinase

UST-FMS Section B 2021 23 of 28


CARDIOLOGY-PULMONOLOGY SHIFTING EXAM (SECTION A)

83. A 60 year old female admitted for breast mass surgery D Massive pulmonary embolism is defined as an acute PE
developed desaturations and difficulty of breathing. with systemic arterial hypotension (SBP < 90 mmHg for
Which finding will classify the patient as massive at least 15 mins.)
pulmonary embolism?
A. Hemoptysis of 200 mL
B. Tachycardia > 100
C. Tachypnea of 30
D. Sustained BP < 90/60

84. Which treatment is recommended for a patient with CT A Consider massive pulmonary embolism because of the
angiography findings of wedge-shaped defect and imaging findings and sustained bradycardia
thrombus on the pulmonary artery and sustained
bradycardia? Gold standard treatment for PE if there is no
A. Alteplase contraindication to fibrinolytics would be to give IV
B. Clopidogrel thrombolytics (Alteplase IV 100 mg over 2 hours)
C. Dabigatran
D. Enoxaparin

85. A patient complained of fever, cough and significant B In pleural effusion, there is (-) Mirror artifact - replaced
pleuritic pain. Ultrasound of the chest was done and this by an echo free space
was the result. What is the next best option? The anechoic (black) region is fluid above the diaphragm
The liver is visualized below the diaphragm

Diagnosis:
Pleural effusion
-excess quantity of fluid in the pleural space
-May develop where there is excess pleural fluid
formation or decreased fluid removal by the lymphatics

Management: VATS

A. Thoracentesis
B. VATS
C. Pleurodesis
D. Chest tube insertion

86. What ultrasound finding is associated with B Barcode sign or Stratosphere sign - Signify
pneumothorax? Pneumothorax
A. A lines
B. Bar code A-line: Air in the Lungs
C. Echo-free space between chest wall and lung
D. Lung Sliding Lung Sliding: Seen in Bat Sign

87. 60 year old COPD and Diabetic has sudden dyspnea. B B lines are perpendicular lines to the parietal pleura
PE showed crackles wheezes over both lung fields. (comet tail artifacts)
Chest UTZ reveals more than 2 zones with > 3 B lines.
What is the best management? Too many B lines mean too much fluid in the interstitium
A. Vasopressor = congestive heart failure
B. Diuretic
C. Steroid
D. Beta agonist

88. A morbidly obese 50 year old male came in due to B Hypercapnia during wakefulness (PaCO2 >45 mmHg)
headache, uncontrolled hypertension, and loud and hypoxemia during wakefulness (SaO2 <90%) are
snoring. His daytime ABG showed PaCO2 of 49 and aspects of Obesity Hypoventilation (Pickwinian)
PaO2 of 79. Which is your consideration? Syndrome. It is differentiated from OSA in that
A. Obstructive sleep apnea hypercapnia subsides during wakefulness in cases of
B. Obesity hypoventilation syndrome OSA.
C. Untreated apnea
D. Acute respiratory failure

UST-FMS Section B 2021 24 of 28


CARDIOLOGY-PULMONOLOGY SHIFTING EXAM (SECTION A)

89. Which of the following findings will be most consistent D STOP BANG Criteria:
with OSA? ● Snoring
A. BMI 23 ● Tiredness
B. Neck circumference 40cm ● Observed Apnea
C. Stable BP ● Pressure (Hypertensive)
D. Snoring ● BMI (>30) - Patient BMI is 23
● Age (>50 y/o)
● Neck Circumference - >40cm (16 in for
females, 17 in for males)
● Gender (Male>Female)

90. A 39 year old male was diagnosed with OSA. His BMI C STOP BANG Criteria:
is 26 with neck circumference of 46 cm. As an ● Snoring
alternative to CPAP, what will you consider? ● Tiredness
A. BIPAP ● Observed Apnea
B. Surgery ● Pressure (Hypertensive)
C. Oral appliance ● BMI (>30) - Patient’s BMI is 26 (Lower BMI)
D. Alcohol intake at night ● Age (>50 y/o) - Patient’s age is 39 (Younger
Age)
● Neck Circumference (16 in for females, 17 in for
males)
● Gender (Male>Female)

91. Case (91-94): A 60-year old chronic smoker presents B ● Patient has risk factors (60 y/o, chronic smoker)
with 10kg weight loss, intermittent cough, and ● Weight loss is present
blood streaked sputum. Chest radiograph showed a ● Blood-streaked sputum could mean that the
4x5 cm opacification over the right paratracheal area lung mass is central (mass could have
and right hilar area with irregular borders. What is the neovascularization near the airways resulting to
most likely diagnosis? hemoptysis whenever the patient coughs.)
A. Bronchiectasis
B. Lung malignancy
C. MRSA pneumonia
D. Pulmonary hypertension

92. If the chest CT showed an upper lobe lesion, which D Features suggestive of Malignant Lesions:
feature of the lesion would point to a possible - spiculated border
malignancy? - presence of corona radiata pattern
A. Popcorn Calcification - a relatively large lesion, lack of asymmetric
B. Central Calcification calcification, chest symptoms, associated
C. Laminated Edges atelectasis, pneumonitis or growth of the lesion
D. Spiculated Borders revealed by comparison with an old x-ray or CT
scan or a positive PET scan

UST-FMS Section B 2021 25 of 28


CARDIOLOGY-PULMONOLOGY SHIFTING EXAM (SECTION A)

93. On imaging, the pulmonary mass is located on the B In general, central lesions such as squamous cell
central area (right hilar area), which is the carcinomas, small-cell carcinomas, or endobronchial
recommended diagnostic procedure? lesions are more readily diagnosed by bronchoscopic
A. CT scan guided biopsy examination.
B. Fiberoptic bronchoscopy with biopsy
C. Ultrasound guided biopsy Peripheral lesions such as adenocarcinomas and large-
D. Open lung biopsy cell carcinomas are more amenable to transthoracic
biopsy

94. What is the stage of the patient if the tumor size is 5cm C TNM Staging System for Lung Cancer (8th Edition)
on the right upper lobe, with multiple ipsilateral carinal
lymph node and metastasis to the pleura? T - T2b
A. Stage III B N - N2
B. Stage III C M - M1a
C. Stage IV A
D. Stage IV B

UST-FMS Section B 2021 26 of 28


CARDIOLOGY-PULMONOLOGY SHIFTING EXAM (SECTION A)

95. How would one characterize the symptom of dyspnea C Asthma- episodic, variability in timing, intensity and
in a 45 year old male smoker diagnosed with COPD? airflow limitation; controlled by medications but cannot
A. Episodic be cured
B. Persistent
C. Worse with exercise PE- most common symptom is unexplained
D. Relieved with coughing breathlessness; pulmonary artery is blocked persistent

COPD- three most common symptoms in COPD are


cough, sputum production and exertional dyspnea.

VTE, COPD Bronchial asthma trans. Harrison’s pg 2036

96. A 65 year old chronic smoker consults at the clinic due C Modified MRC = breathlessness
to difficulty of breathing. He has two exacerbations this COPD Assessment Test (CAT) = symptoms
year and an MMRC score of 2, CAT of 10. Which
medications should be prescribed?

A. SABA
B. ICS
C. ICS+LABA/LAMA
D. LABA or LAMA

Source: 2020 GOLD Report

97. Which of the following is an essential non- D


pharmacologic management for COPD in patient group
B to D?
A. Flu and pneumococcal vaccine
B. Smoking cessation
C. Physical activity limitation
D. Pulmonary rehabilitation
Wala sa trans. Taken from 2018 GOLD guidelines

Group A = Smoking cessation (may include


pharmacologic treatment); essential management

Group B-D = smoking cessation (may include


pharmacologic treatment) OR Pulmonary
rehabilitation; essential management

Generally:
Smoking cessation = Groups A-D
Pulmonary Rehabilitation = Groups B-D

Flu and pneumococcal vaccine = relative management,


depending on the local guidelines

UST-FMS Section B 2021 27 of 28


CARDIOLOGY-PULMONOLOGY SHIFTING EXAM (SECTION A)

98. What is the GOLD classification of severity in a patient B


with FEV1/FVC of <70%; and FEV1 of 50% predicted?
A. GOLD 1 Mild
B. GOLD 2 Moderate
C. GOLD 3 Severe
D. GOLD 4 Very Severe

99. A 56-year-old farmer presents with chronic cough. On D Bronchiectasis


further questioning, he describes the chronic cough to -chronic productive cough w/ daily sputum production
be productive particularly with large volume and thick - chest radiograph finding: low set diaphragm with
sputum . bullous lesions (honeycomb appearance)

Differential diagnosis:
COPD
- cough: worse in the morning w/ small amount of
colorless sputum

Based on history and chest x-ray, what is the diagnosis?

A. COPD
B. Interstitial fibrosis
C. Heart failure
D. Bronchiectasis

100. What is the most cited mechanism of infective A


bronchiectasis?
A. Vicious cycle hypothesis
B. Protease-antiprotease hypothesis
C. Homozygous alpha; deficiency
D. Immune-mediated reaction

Cole’s Hypothesis on Vicious Cycle of Bronchiectasis

Reference: MED2 16 Bronchiectasis pg. 2

-- END --

Goodluck and God bless, Doctors! ~B2021

UST-FMS Section B 2021 28 of 28

You might also like